Final Final Final

Ace your homework & exams now with Quizwiz!

During a routine yearly physical an older adult says to a nurse, "I haven't had sex lately because I can't get an erection anymore!" What should the nurse's initial response be?

"Let's discuss this concern a little more."

Which client statement supports the diagnosis of somatic delusions?

"My heart stopped beating 3 days ago, and now my lungs are rotting away."

A client is hospitalized with a tentative diagnosis of pancreatic cancer. On admission the client asks the nurse, "Do you think I have anything serious, like cancer?" What is the nurse's best reply?

"I don't know if you do; let's talk about it."

A client with bipolar disorder is dealing with a variety of other chronic health and social issues. In light of the information provided by the client, the nurse addresses the priority safety concern by asking what?

"What do you mean when you say that you don't want to see another day?"

A nurse is caring for a client with the diagnosis of dementia. What should the nurse ask the client to best ascertain orientation to place?

"Where are you?"

A depressed client tells a nurse, "I want to die." What is the most therapeutic response by the nurse?

"You would rather not live."

What is the purpose of a mental health treatment plan (pg.93)?

(1)It is a guide for planning and implementing clients care ,(2) the plan serves as a vehicle for monitoring the client's progress and the effectiveness of therapeutic interventions, (3) the plan serves as a mean for communicating and coordinating clients care

A nurse needs to obtain consent for the medical treatment of a child whose parent is a minor. What appropriate step should the nurse take to obtain consent? 1 The nurse should ask the minor to give consent. 2 The nurse should wait for the consent of the court. 3 The nurse should ask any adult siblings of the minor to give consent. 4 The nurse should ask a legal guardian of the minor to give consent.

1

A nurse works in a crisis intervention center. A woman who has experienced sexual abuse comes in and says, "I've got to talk to someone or I'll go crazy. I shouldn't have dated him." What is most important for the nurse to identify after initially assessing the client's physical condition? 1 Support system 2 Sexual background 3 Ability to relay the facts 4 Knowledge of sexual assault terminology

1

What effect of anxiety makes it particularly important for the nurse to reduce the anxiety of a client with heart failure? 1 Increases the cardiac workload 2 Interferes with usual respirations 3 Produces an elevation in temperature 4 Decreases the amount of oxygen used

1 Increases the cardiac workload Irritability and restlessness associated with anxiety increase the metabolic rate, heart rate, and blood pressure; these complicate heart failure. Anxiety does not directly interfere with respirations; an increase in cardiac workload will increase respirations. Anxiety alone usually does not elevate the body temperature. Anxiety can cause an increase in the amount of oxygen used and leads to an increased respiratory rate.

A client who recently has had an abdominoperineal resection and colostomy accuses the nurse of being uncomfortable during a dressing change because the "wound looks terrible." The nurse identifies that the client is using the defense mechanism known as: 1 Projection 2 Sublimation 3 Compensation 4 Intellectualization

1 Projection Projection is the attribution of unacceptable feelings and emotions to others. Sublimation is the substitution of socially acceptable feelings or instincts to replace those that are threatening to the ego. Compensation is overachievement in a more comfortable area, thereby covering up a weakness. Intellectualization is the use of mental reasoning processes to deny facing emotions and feelings involved in a situation.

When intimate partner violence (IPV) is suspected, the nurse plays an important role as an advocate for the victim. The advocate role includes what important components? Select all that apply. 1 Planning for future safety 2 Normalizing victimization 3 Validating the experiences 4 Promoting access to community services 5 Providing housing for the victim

1, 3, 4

A nurse is assessing a toddler and the dynamics of the child's family, in which abuse is suspected. What behaviors are expected? Select all that apply. 1 The child cringes when approached. 2 The child has unexplained healed injuries. 3 The parents are overly affectionate toward the child. 4 The child lies still while surveying the environment. 5 The parents give detailed accounts of the child's injuries.

1,2,4

A nurse is teaching a client about false imprisonment. What information should the nurse provide? Select all that apply. 1 "False imprisonment is an example of an intentional tort." 2 "False imprisonment refers to any intentional touching without consent." 3 "False imprisonment involves restraining a person unjustly without any legal warrant." 4 "A falsely imprisoned client should be aware of his or her confinement." 5 "An unconscious client in restraints is considered to be falsely imprisoned."

1,3,4

Which initiatives for leaders and managers are set forth in the Institute of Medicine's core relevance report "The Future of Nursing?" Select all that apply. 1 Created state coalitions focused on nursing 2 Reinforced the disparities that occur within healthcare 3 Supported the concept of shared governance in nursing 4 Focused on outcomes, accreditation processes, and nurse's roles 5 Provided a framework for considering how nurses could determine staffing requirements

1,5

What statement by a 45-year-old woman scheduled for an abdominal hysterectomy and bilateral salpingo-oophorectomy should alert the nurse to the potential for postoperative coping difficulties? 1. "I'm not the least bit worried." 2. "I don't want any more children." 3. "I get along very well with my husband." 4. "I'll be glad not to have any more periods."

1. "I'm not the least bit worried." **Not being worried indicates potential denial and possible failure to address the problem emotionally

An older female client is concerned about maintaining her independent living status. What initial intervention strategy is of primary importance? 1. Reinforcing routines and supporting her usual habits 2. Helping her secure assistance with cleaning and shopping 3. Writing down and repeating important information for her use 4. Setting clear goals and time limitations for her visits with the nurse

1. Reinforcing routines and supporting her usual habits

The nurse is completing a mental status examination for a newly admitted patient. In which part of the nursing process is the nurse functioning? 1. assessment 2. planning 3. implementation 4. evaluation

1. assessment

After collecting data the nurse is concerned that a patient is at risk for developing a mental illness. What findings caused the nurse to come to this conclusion? SELECT ALL THAT APPLY 1. difficulty thinking 2. discussed the use of daily exercise to deal with stress 3. made statements indicating being out of touch with reality 4. expressed remorse in having to leave a pet dog with a neighbor 5. explaining a lack of friends because everyone is jealous of the patient

1. difficulty thinking 3. made statements indicating being out of touch with reality 5. explaining a lack of friends because everyone is jealous of the patient

The nurse is assisting in the preparation of an educational seminaron anxiety disorders. Which anxiety disorder should the nurse make sure are included in this presentation?SELECT ALL THAT APPLY 1. phobia 2. panic disorder 3. schizophrenia 4. unipolar depression 5. PTSD 6. OCD

1. phobia 2. panic disorder 5. PTSD 6. OCD

After being admitted for a surgical procedure a patient with a history of schizophrenia becomes extremely agitated when other patients are around and believes that others want to cause the patient harm. Which intervention should the nurse use to provide a therapeutic milieu for the patient? 1. place the patient in a private room 2. request an order for antipsychotic medication 3. help the patient clarify the meaning of the feelings 4. suggest the patient be scheduled for a psychotherapy session

1. place the patient in a private room

A patient with extreme anxiety is arriving for out patient chemotherapy. What should the nurse do to help reduce the patient's anxiety during this current situation? 1. play a CD with nature sounds 2. select a TV station with a sporting event 3. close the door to the room during the treatment 4. remind the patient that anxiety is not going to make the treatment effective

1. play a CD with nature sounds

The nurse is assisting with teaching a patient who is to begin taking a monoamine oxidase inhibitor (MAOI). Which foods should the nurse teach the patient to avoid? SELECT ALL THAT APPLY 1. fish 2. wine 3. bread 4. pastas 5. aged cheese

2. wine 5. aged cheese

A nurse teaches dietary guidelines to a client who will be receiving tranylcypromine sulfate (Parnate), a monoamine oxidase inhibitor (MAOI). The client compiles a list of foods to avoid. Which foods included on the list indicate that the teaching has been effective? (Select all that apply.) 1.French fries 2.Pepperoni pizza 3.Bologna sandwich 4.Hamburger on a bun 5.Hash brown potatoes

2.Pepperoni pizza 3.Bologna sandwich

A client sits huddled in a chair and leaves it only to assume the fetal position in a corner. The nurse, observing this, identifies the behavior as: 1.Reactive 2.Regressive 3.Dissociative 4.Hallucinatory

2.Regressive

A client asks the nurse how psychotropic medications work. The nurse correctly replies: 1. "These medications decrease the metabolic needs of your brain." 2. "These medications increase the production of healthy nervous tissue." 3. "These medications affect the chemicals used in communication between nerve cells." 4. "These medications regulate the sensory input received from the external environment."

3. "These medications affect the chemicals used in communication between nerve cells."

The nurse is caring for a client experiencing a crisis. What role is most important for the nurse to assume when providing therapeutic crisis intervention? 1. Passive listener 2. Friendly adviser 3. Active participant 4. Participant observer

3. Active participant

One morning a client tells the nurse, "My legs are turning to rubber because I have an incurable disease called schizophrenia." The nurse identifies that this as an example of: 1. Hallucinations 2. Paranoid thinking 3. Depersonalization 4. Autistic verbalization

3. Depersonalization

A health care provider refers a 52-year-old man to the mental health clinic. The history reveals that the man lost his wife to colon cancer 6 months ago and that since that time he has seen his health care provider seven times with the concern that he has colon cancer. All tests have had negative results. Recently the client stopped seeing friends, dropped his hobbies, and stayed home to rest. Which disorder should the nurse identify as consistent with the client's preoccupation with the fear of having a serious disease? 1. Conversion disorder 2. Somatization disorder 3. Hypochondriac disorder 4. Body dysmorphic disorder

3. Hypochondriac disorder **Preoccupation with fears of getting or having a serious disease is called hypochondriasis

A nurse is caring for an adolescent who has anorexia nervosa. The nutritional treatment of anorexia is composed of several guidelines. Which guidelines should the nurse emphasize? (Select all that apply.) 1. Increasing high-fiber foods 2. Eating just three meals a day 3. Increasing food intake gradually 4. Limiting mealtime to half an hour 5. Providing privileges for goal achievement

3. Increasing food intake gradually 4. Limiting mealtime to half an hour 5. Providing privileges for goal achievement

A 19 year-old, arrested for assault and robbery, has a history of truancy and prostitution but is unconcerned that her behavior has caused emotional distress to others. The diagnosis of antisocial personality disorder is made. According to psychoanalytical theory, the client's lack of remorse and repetitive behavior probably are related to an underdeveloped: 1. Id 2. Ego 3. Superego 4. Limbic system

3. Superego

A client with depression is to be given fluoxetine (Prozac). What precaution should the nurse consider when initiating treatment with this drug? 1. It must be given with milk and crackers to avoid hyperacidity and discomfort. 2. Eating cheese or pickled herring or drinking wine may cause a hypertensive crisis. 3. The blood level may not be sufficient to cause noticeable improvement for 2 to 4 weeks. 4. The blood level should be checked weekly for 3 months to monitor for an appropriate level.

3. The blood level may not be sufficient to cause noticeable improvement for 2 to 4 weeks. **Fluoxetine (Prozac) does not produce an immediate effect; nursing measures must be continued to reduce the risk for suicide

The nurse is assisting with data collection on a patient newly diagnosed with schizophrenia. Which observation should the nurse consider as being positive symptoms of schizophrenia? SELECT ALL THAT APPLY 1. alogia 2. apathy 3. delusions 4. hallucinations 5. social isolation 6. disorganized behavior

3. delusions 4. hallucinations 6. disorganized behavio

A patient recovering from electroconvulsive therapy (ECT) for sever depression is confused and wants to know what has been done. What action by the nurse is best? 1. administer a sedative to help calm the patient 2. call the physician and report the patients response 3. explain that the patient is in the hospital and has just had a ECT 4. encourage the patient to go back to sleep until the pre-procedure medication has worn off

3. explain that the patient is in the hospital and has just had a ECT

A patient who has schizophrenia has a dull facial expression and speaks in a monotone voice, even though a visitor is making an effort to be jovial. What terminology should the nurse use to document this observation? 1. bored 2. depressed 3. flat affect 4. ambivalent attitude

3. flat affect

A patient with schizophrenia states, " I'm going to the fribity to see a megnat". What term should the nurse use to describe this language? 1. imagery 2. soliloquy 3. neologism 4. conversions

3. neologism

A student is angry about failing a test at school and accuses the teacher of trying to fail everyone. What type of response is the student demonstrating? 1. anger 2. denial 3. projection 4. repression

3. projection

What developmental task should the nurse consider when caring for toddlers? 1.Trust 2.Industry 3.Autonomy 4.Identification

3.Autonomy

A nurse is caring for a 20-year-old client. According to Erikson's developmental psychosocial theory, what is expected by 20 years of age? 1.Having the capacity for love and a commitment to work 2.Being creative and productive and having concern for others 3.Having a coherent sense of self and plans for self-actualization 4.Accepting the worth, integrity, and uniqueness of one's past and present life

3.Having a coherent sense of self and plans for self-actualization

What should the nurse do to develop a trusting relationship with a disturbed child who acts out? 1.Ask the child's feelings about the parents 2.Implement one-on-one interactions every half hour 3.Offer support and encourage safety during play activities 4.Begin setting limits and explain the rules that must be followed

3.Offer support and encourage safety during play activities

A female client who physically abused her 9-year-old son is undergoing treatment to help her control her behavior. Which statement indicates that the client has developed a safe coping method to help her deescalate? 1 "I promise that I won't get so angry when my son causes trouble again." 2 "If my son gets straightened out, we shouldn't have these kinds of problems." 3 "I think the root of the problem is when my husband comes home after drinking." 4 "If I get angry at my son again, I'm going to need a pillow in the bedroom to punch."

4

What is the best room assignment for a 5-year-old child admitted with injuries that may be related to abuse? 1 In an isolation room 2 With a friendly older child 3 With a child of the same age 4 In a room near the nurses' desk

4

A client with a mood disorder is being discharged from a psychiatric hospital after agreeing to continue follow-up visits with a therapist. During the last interview with the nurse before discharge, the client says, "I've told you a lot about my life and my problems, but there are a few things that bother me that I've told no one." What is the most therapeutic response by the nurse once it has been determined that the client is not at risk for harming herself or others? 1 "The purpose of our getting together is to discuss your problems." 2 "Do you want to work on those during the few minutes we have left?" Incorrect3 "What kind of problem have you not shared with me during our time together?" 4 "One purpose of continuing counseling is to allow you to discuss things that bother you."

4 "One purpose of continuing counseling is to allow you to discuss things that bother you."

How long after the last dose should the nurse schedule to have a client's blood drawn to evaluate the serum lithium level? 1. 2 to 4 hours 2. 4 to 6 hours 3. 6 to 8 hours 4. 8 to 12 hours

4. 8 to 12 hours

A client on the psychiatric unit who has suicidal ideas says to the nurse, "I signed myself in. I'll sign myself out." What concept provides the basis for the nurse's response? 1. Voluntary clients may sign out at any time. 2. Voluntary clients may sign out by following unit procedures. 3. Suicidal clients may sign out if they are able to contract for their safety. 4. Suicidal clients may not sign out even if they voluntarily admitted themselves.

4. Suicidal clients may not sign out even if they voluntarily admitted themselves. **The priority is to keep the client safe; a client admitted on a voluntary basis may be kept involuntarily if professional judgment indicates that the client may harm him- or herself or others.

Adolescents and young adult women account for ______% of Americans affected by eating disorders.

85 to 90

10. In the role of milieu manager, which activity should the nurse prioritize? 1. Setting the schedule for the daily unit activities 2. Evaluating clients for medication effectiveness 3. Conducting therapeutic group sessions 4. Searching newly admitted clients for hazardous objects

ANS: 4 Rationale: The milieu manager should search newly admitted clients for hazardous objects. Safety of the client and others always takes priority. Nurses are responsible for ensuring that the client's safety and physiological needs are met.

Which factors affect the stress that first responders feel after a mass casualty incident (MCI)? Select all that apply.

Age Nature of event Coping mechanisms Psychologic history

Primary caregiver dysfunction

Aggressive intervention can improve the long-term outlook for the children of these parents.

A nurse is caring for a client with dementia. Which clinical manifestations are expected? (Select all that apply.)

Agitation Short attention span Disordered reasoning Impaired motor activities

A male client with the diagnosis of depression is taking a monoamine oxidase inhibitor (MAOI). Which is the most important teaching point the nurse must include in his care plan?

Avoid alcoholic beverages

The nurse recalls that the major defense mechanism used by an individual with a phobic disorder is:

Avoidance

A child with attention deficit-hyperactivity disorder (ADHD) often becomes frustrated and loses control. A nurse uses a variety of graduated techniques to manage disruptive behaviors. List the following interventions in order, from the least invasive to the most invasive.

Avoiding situations that usually precipitate frustration Monitoring behavior for cues of rising anxiety Using a signal to remind the child to use self-control Refocusing the child's behavior with a specific directive Placing the child in a time-out

The __________ constitute a class of drugs that are commonly prescribed for cardiac arrhythmias but also have been found to be effective treatment for social phobias.

Beta-blockers

Somatoform disorder

Common in school-age children

An older client is apprehensive about being hospitalized. The nurse realizes that one of the stresses of hospitalization is the unfamiliarity of the environment and activity. How can the nurse best limit the client's stress?

Explain what the client can expect.

What is the purpose of using the psychiatric assessment tool (pg.95)?

Focuses on obtaining data about the problem, coping behaviors, and resources of clients

After a traumatic event, a client is extremely upset and exhibits pressured and rambling speech. What therapeutic technique can the nurse use when a client's communication rambles?

Focusing

Fear of growing old

Gerontophobia

Imipramine (Tofranil), 75 mg three times per day, is prescribed for a client. What nursing action is appropriate when this medication is being administered?

Having the client checked for increased intraocular pressure and teaching about symptoms of glaucoma

Problems of Childhood

Health care providers who work with children must have an understanding of normal growth and development as well as an awareness of each child's individual pace.

What is the most appropriate way for the nurse to help a severely depressed adolescent client accept the realities of daily living?

Helping the client fulfill personal hygiene needs

Humanistic theories are important to health care because these theories serve as the foundation for the concept of:

Holistic care

According to Freud's theory, a baby who is crying in response to wanting to be held by his mother is an example of which part of the personality's control over behavior?

Id

What are some data collecting methods used by care providers for mentally ill patients (pg.94)?

Interviews, observational techniques, and rating scales and inventories

A nurse is evaluating a young adult for evidence of achievement of the age-related developmental stage set forth in Erikson's developmental theory. What developmental crisis is associated with this age group?

Intimacy versus isolation

General interventions are focused on the following:

Meeting basic needs, Providing opportunities, Encouraging self-care activities

Medication

Must be monitored closely by the nurse

What diagnoses of a mentally ill patient relates to the client's problems affecting his or her ability to carry out the activities of daily living (pg.93)?

Nursing diagnoses

Data collection is divided into what two groups (pg.94)?

Objective data and subjective data

The health care provider prescribes donepezil (Aricept) 5 mg by mouth once a day for a client exhibiting initial signs of dementia of the Alzheimer type. The client is already taking digoxin (Lanoxin) 0.125 mg in the morning and alprazolam (Xanax) 0.5 mg twice a day. What should the nurse teach the client's spouse to do?

Prefill a weekly drug box with the medications for the spouse to self-administer

Children grow in

Proximal-distal (near-far) pattern

Freud believed that an individual's personality developed through stages of sexual instinct from birth to adulthood. This is known as the __________ theory of personality development.

Psychosexual

Therapeutic interventions

Reduce the problems that are causing depression.

school avoidance

Repeated absence from school of a child who is physically healthy.

A male client who has a diagnosis of generalized anxiety disorder (GAD) is seen in the emergency room with complaints of chest pain, shortness of breath, and inability to concentrate, along with feelings of overwhelming anxiety. The nurse uses Maslow's theory to triage the client's complaints, knowing that which complaint must be addressed first?

Shortness of breath

Wakens with strong verbal stimuli; falls asleep if left undisturbed

Somnolent level of consciousness

Expressive language disorder

Speaking at a rapid or slow rate, with strange rhythms and word use

During an admission assessment, a male client states that he has been having auditory hallucinations and difficulty concentrating at work. This type of data refers to:

Subjective

What is the biggest risk factor for a mentally ill patient?

Suicide

Betty Neuman developed the nursing theory that promotes nursing interventions to assist individuals in reaching and maintaining the highest level of wellness possible. What is this theory known as?

Systems model

Primary caregiver dysfunction

When a parent is unable to meet the needs of a child, a disturbance in the parent-child interaction exists.

The nurse knows that the major concept of the systems theory is that individuals are viewed as functioning:

Within a set of interacting and related units

What is important when observing a mentally ill client during the intervention phase (pg.93)?

Work to keep an open mind and remember that many reactions are culturally determined.

hallucination drug therapy

antipyschotics and neuroleptic drugs

common problems of childhood

colic feeding and sleeping temper tamtrums breathe holding spells

Attempt to overcome feelings of inferiority or make up for deficiency. A girl who thinks she cannot sing studies to become an expert pianist.

compensation

conduct disorder

defiant of authority

devoted to the study and treatment of behaviors related to brain functioning.

neurobiological

What is the purpose of the interview process (pg.94)?

obtaining or exchanging information

Dr. Sigmund Freud believed that a person's unconscious thoughts and emotions affect his or her behavior. The now well-known therapy that he developed to explore an individual's unconscious thoughts is referred to as ____________.

psychoanalysis

Study of biochemical foundations of thought, mood, emotion, and behavior.

psychobiological

What is the purpose of using the DSM or DSM-5 (PGS.93)?

to aide in diagnosis and help guide clinical practice

What is the purpose of a formal interview (pg.94)?

usually documented on a standardized form

Cephalocaudal

where growth occurs from head to tail

A male client with the diagnosis of a bipolar disorder, depressed episode, is found lying on the floor in his room in the psychiatric unit. He states, "I don't deserve a comfortable bed; give it to someone else." The best response response by the nurse is:

"Everyone has a bed. This one is yours."

A client with schizophrenia is experiencing auditory hallucinations. A nurse makes the following statements when interacting with this client. Place these statements in the order in which they should be made.

"Hearing voices must be frightening." "The voices you hear are part of your illness." "I don't hear any voices." "Come with me for a walk." "Let's play cards with another client in the recreation room."

A nurse moves into the working phase of a therapeutic relationship with a depressed client who has a history of suicide attempts. What question should the nurse ask the client when exploring alternative coping strategies?

"How will you manage the next time your problems start piling up?"

A 56-year-old man is admitted to the inpatient unit after family members report that he seems to be experiencing auditory hallucinations. The man has a history of schizophrenia and has had several previous admissions. Which statement indicates to the nurse that the client is experiencing auditory hallucinations? "Get these horrible snakes out of my room!" "I am not the devil! Stop calling me those names!" "The food on this plate has poison in it, so take it away—I won't eat it." "I did see an alien spaceship last night outside in my yard, and I've felt worse ever since."

"I am not the devil! Stop calling me those names!" The client is responding to messages that he is hearing, which are auditory hallucinations. The responses regarding the snakes and the spaceship are examples of visual hallucinations because they describe what the client sees. The accusation of poisoning is the statement of a client who is suspicious and paranoid but not hallucinating.

When a nurse sits next to a depressed client and begins to talk, the client responds, "I'm stupid and useless. Talk with the other people who are more important." Which response is most therapeutic?

"I want to talk with you because you are important to me."

A client with schizophrenia reports having ongoing auditory hallucinations that he describes as "voices telling me that I'm a bad person" to the nurse. What is the best response by the nurse?

"Try to ignore the voices."

A client asks the nurse, "Should I tell my partner that I just found out I'm human immunodeficiency virus (HIV) positive?" What is the nurse's most appropriate response?

"You are having difficulty deciding what to say."

After a therapy session with a health care provider in the mental health clinic, a client tells the nurse that the therapist is uncaring and impersonal. What is the best response by the nurse?

"You have strong feelings about your therapy session and your therapist." The use of reflection assists the client in expressing feelings, which is the major goal of therapy. Telling the client that the therapist is good is a defensive response by the nurse that may cut off communication and limit the expression of feelings. Telling the client to try to cooperate avoids discussion of the client's feelings about the therapist.

When a client who has had a mastectomy sees her incision for the first time, she exclaims, "I look horrible! Will it ever look better?" What is the nurse's best response?

"You seem shocked by the way you look now."

lithium lab value

0.5-1.5

A client at 16 weeks' gestation arrives at the prenatal clinic for a routine visit. During the examination the nurse notes bruises on the client's face and abdomen. There are no bruises on her legs and arms. Further assessment is required to confirm what? 1 Domestic abuse 2 Hydatidiform mole 3 Excessive exercise 4 Thrombocytopenic purpura

1

A mother and her three young children arrive at the mental health clinic. The woman says that she is seeking help in leaving her husband. She reports that he has been beating her for years but just started hitting the children. What is the best initial action by the nurse? 1 Arranging for a staff member to watch the children so the mother and nurse can talk 2 Calling a facility where the mother and her children will be safe until the crisis is resolved 3 Determining whether the mother is ambivalent about this decision before making permanent plans 4 Suggesting that the mother and her husband return for couples counseling so the marriage can be saved

1

A nurse is teaching a group of parents about child abuse. What definition of assault should the nurse include in the teaching plan? 1 Assault is a threat to do bodily harm to another person. 2 Assault is a legal wrong committed by one person against the property of another. 3 Assault is a legal wrong committed against the public that is punishable by federal law. 4 Assault is the application of force to another person without lawful justification.

1

Sildenafil (Viagra) is prescribed for a man with erectile dysfunction. A nurse teaches the client about common side effects of this drug. (Select all that apply.) 1. Flushing 2. Headache 3. Dyspepsia 4. Constipation 5. Hypertension

1. Flushing 2. Headache 3. Dyspepsia

A psychiatric nurse is hired to work in the psychiatric emergency department of a large teaching hospital. While reviewing the manuals, the nurse reads, "People with mental health emergencies shall be triaged within 5 minutes of entering the emergency department." What does the nurse consider this statement to represent? 1. Hospital policy 2. Standard of care 3. Hospital procedure 4. Mental Health Bill of Rights

1. Hospital policy

An adult who has been in a gay relationship for 3 years arrives at the emergency department in a state of near-panic. The client says, "My partner just left me. I'm a wreck." What should the nurse do to help the client cope with this loss? (Select all that apply.) 1. Identify the client's support systems 2. Explore the client's psychotic thoughts 3. Reinforce the client's current self-image 4. Encourage the client to talk about the situation 5 . Suggest that the client explore personal sexual attitudes

1. Identify the client's support systems 4. Encourage the client to talk about the situation

A college student is brought to the mental health clinic by his parents. The diagnosis is borderline personality disorder. Which factors in the client's history support this diagnosis? (Select all that apply.) 1. Impulsiveness 2. Lability of mood 3. Ritualistic behavior 4. Psychomotor retardation 5. Self-destructive behavior

1. Impulsiveness 2. Lability of mood 5. Self-destructive behavior

A client who had to be cut out of a car after a motor vehicle collision has no visible physical effects from the ordeal. The client responds to the emergency department nurse's questions factually in a soft voice with a composed manner. This behavior may indicate that the client: 1. Is controlling the expression of feelings 2. Has repressed the details of the accident 3. Has blocked out the events of the last few hours 4. Is experiencing the reorganization phase of the trauma experience

1. Is controlling the expression of feelings

A primary nurse notes that a client has become jaundiced after 2 weeks of antipsychotic drug therapy. The nurse continues to administer the antipsychotic until the health care provider can be consulted. What does the nurse manager conclude about this situation? 1. Jaundice is sufficient reason to discontinue the antipsychotic. 2. Jaundice is a benign side effect of antipsychotic agents that has little significance. 3. The blood level of an antipsychotic drug must be maintained once it has been established. 4 .The prescribed dosage for the antipsychotic agent should have been reduced by the nurse.

1. Jaundice is sufficient reason to discontinue the antipsychotic.

The nurse anticipates that the medication that will be used to prevent symptoms of withdrawal in clients with a long history of alcohol abuse is: 1. Lorazepam (Ativan) 2. Phenobarbital (Luminal) 3. Chlorpromazine (Thorazine) 4. Methadone hydrochloride (Methadone)

1. Lorazepam (Ativan) **Lorazepam (Ativan) is most effective in preventing the signs and symptoms associated with withdrawal from alcohol.

Which nursing intervention involves providing, structuring, and maintaining a safe and therapeutic environment in collaboration with patients, families, and other health care providers? 1. Milieu therapy 2. Coordination of care 3. Health teaching and health maintenance 4. Pharmacologic, biologic, and integrative therapies

1. Milieu therapy **Milieu therapy involves providing, structuring, and maintaining a safe and therapeutic environment in collaboration with psychiatric patients, families, and other health care providers.

A young client who has become a mother for the first time is anxious about her new parenting role. With the nurse's encouragement, she joins the new mothers' support group at the local YMCA. What kind of prevention does this activity reflect? 1. Primary prevention 2. Tertiary prevention 3. Secondary prevention 4. Therapeutic prevention

1. Primary prevention

A client with the diagnosis of schizophrenia, paranoid type, has been receiving a phenothiazine drug. The daycare center is planning a fishing trip. It is important that the nurse: 1. Provide the client with sunscreen 2. Caution the client to limit exertion during the trip 3. Give the client an extra dose of medication to take after lunch 4. Take the client's blood pressure before allowing him to participate in the outing

1. Provide the client with sunscreen

A client begins fighting and biting other clients. The practitioner prescribes a stat injection of haloperidol (Haldol). How should the nurse implement this prescription? 1. Quickly, with an attitude of concern 2. Before the client realizes what is happening 3. After the client agrees to receive the injection 4. Quietly, without any explanation of the reason for it

1. Quickly, with an attitude of concern **Quickness is used for safety; an attitude of concern may help reduce the client's anxiety.

What should a nurse ensure when creating an environment that is conducive to psychological safety? 1. Realistic limits are set. 2 Passive acceptance is promoted. 3 The client's physical needs are met. 4 The physical environment is kept in order

1. Realistic limits are set.

A client is to begin lithium carbonate therapy. The nurse should ensure that before the drug's administration the client has baseline: 1. Renal studies 2. Liver enzyme studies 3. Adrenal function studies 4. Pulmonary function studies 5. New Choice 6. New Choice 7. New Choice

1. Renal studies

A depressed client often sleeps past the expected time of awakening and spends excessive time resting and sleeping. Which nursing intervention is appropriate for this client? 1. Restricting the client's access to the bedroom 2. Offering the client a series of relaxation tapes 3. Rescheduling the client's bedtime to an earlier hour 4. Suggesting that the client exercise before going to bed

1. Restricting the client's access to the bedroom **The goal is 6 to 8 hours of rest at night; too much time spent sleeping in the daytime will defeat the goal of adequate rest at night.

The nurse refers a client to a self-help group. What does the nurse anticipate that a self-help groups such as Alcoholics Anonymous (AA) will help its members learn? 1. That their problems are not unique 2. That they do not need a crutch to lean on 3. That their problems are caused by alcohol 4. That the group can stop them from drinking

1. That their problems are not unique **Sharing problems with others who have similar problems can help one explore feelings and begin to enhance coping abilities.

According to Erikson, a child's increased vulnerability to anxiety in response to separation or pending separation from significant others results from failure to complete a developmental stage. What does the nurse call this stage? 1. Trust 2. Identity 3. Initiative 4. Autonomy

1. Trust

For which adverse effect should the nurse continually observe a client who is receiving valproic acid (Depakene)? 1. Yellow sclerae 2. Motor restlessness 3. Ringing in the ears 4. Torsion of the neck

1. Yellow sclerae **Yellow sclerae are a sign of jaundice; pancreatitis and hepatic failure are life-threatening adverse effects of valproic acid (Depakene).

A patient with schizophrenia is returning from a CT scan of the brain followed by an electroencephalogram. Which diagnostic test finding should the nurse identify as supporting this patient's diagnosis?SELECT ALL THAT APPLY 1. enlarged ventricles 2. reduced amount of gray matter 3. areas of nerve de-myelinization aneurysms of the cerebral vessels 4. diminished prefrontal cortex activity

1. enlarged ventricles 2. reduced amount of gray matter 4. diminished prefrontal cortex activity

The nurse is assisting in planning care for a patient with extreme anxiety. Which interventions should the nurse include in this patient's plan of care? SELECT ALL THAT APPLY 1. maintain a calm environment 2. encourage verbalization of feeling 3. model and encourage positive self talk 4. encourage participation in competitive activities 5. permit the patient to have time alone during acute anxiety events

1. maintain a calm environment 2. encourage verbalization of feeling 3. model and encourage positive self talk

The nurse is reviewing potential patient teaching needs. For what prescribed medications should the nurse plan to instruct patients to follow a tyramine-free diet? SELECT ALL THAT APPLY 1. phenelzine (Nardil) 2. vuspirone (Buspar) 3.isocarboxazid (Marplan) 4. valproic acid (Depakote) 5.lithium carbonate (Eskalith)

1. phenelzine (Nardil) 3. isocarboxazid (Marplan)

The nurse is assisting a patient identifying coping mechanisms. What general definition of coping is the nurse most likely using with this patient ? 1. the way one adapts to a stressor 2. the adaption to mental health problems 3. the use of specific mechanisms to reduce anxiety 4. the development of unconscious behaviors to reduce psychological distress

1. the way one adapts to a stressor

Lithium levels are considered toxic when they become higher than __________ mEq/L

1.5

An 18-month-old toddler is admitted to the pediatric unit. The child has never been separated from the mother before this admission. What behavior does the nurse expect from the toddler? 1 Sitting quietly, uninterested in playing 2 Crying relentlessly and consoled by no one but a parent 3 Looking sad but becoming content after meeting roommates 4 Screaming when people enter the room but smiling after several minutes

2 Crying relentlessly and consoled by no one but a parent The first stage of separation anxiety is protest, which is characterized by loud crying, rejection of all strangers, and inconsolable grief. Despair is indicative of the second stage of separation anxiety. Toddlers do not socialize well with peers. Smiling is not expected because an 18-month-old child is not easily consoled when separated from his or her parents.

A nurse administers cephulac (Lactulose) to a client with cirrhosis of the liver. Which response leads the nurse to determine that the cephulac is effective? 1 Decreased amylase Correct2 Decreased ammonia 3 Increased potassium 4 Increased hemoglobin

2 Decreased ammonia Cephulac destroys intestinal flora that break down protein and in the process give off ammonia. In clients with cirrhosis, ammonia is inadequately detoxified by the liver and can build to toxic levels. Amylase levels are associated with pancreatic problems. Potassium is not directly affected by neomycin; it may increase if nephrotoxicity occurs. Hemoglobin is not influenced by neomycin.

A client with emotional problems is being discharged from a psychiatric unit. What should the nurse encourage the client to do? 1 Go back to regular activities. 2 Enroll in an aftercare program. 3 Call the unit whenever she is upset. 4 Find a group that has similar problems.

2 Enroll in an aftercare program.

A nurse is implementing interventions to assist an aggressive client in deescalating the agitated behavior. Select all that apply. 1 Physical contact with the client to show caring 2 Encouraging the client to express perceived needs 3 Avoiding verbal struggles in an attempt to demonstrate authority 4 Providing the client with clear options to the unacceptable behavior 5 Referring to the client in an authoritarian manner to demonstrate control of the situation 6 Explaining the expected outcomes if the client is unable to control the unacceptable behavior

2,3,4,6

A client with a diagnosis of paranoid schizophrenia throws a chair across the room and starts screaming at the other clients. Several of these clients have frightened expressions, one starts to cry, and another begins to pace. A nurse removes the agitated client from the room. What should the nurse remaining in the room do next? 1. Continue the unit's activities as if nothing has happened 2. Arrange a unit meeting to discuss what has just happened 3. Refocus clients' negative comments to more positive topics 4. Have a private talk with the clients who cried and started to pace

2. Arrange a unit meeting to discuss what has just happened

A woman who is emotionally and physically abused by her husband calls a crisis hotline for help. The nurse works with the client to develop a plan for safety. What should be included in the safety plan? (Select all that apply.) 1. Limiting contact with the abuser 2. Determining a safe place to go in an emergency 3. Memorizing the domestic violence hotline number 4. Obtaining a bank loan to finance leaving the abuser 5. Arranging for a family member to assist her in leaving

2. Determining a safe place to go in an emergency 3. Memorizing the domestic violence hotline number

Neuroleptic malignant syndrome is a potentially fatal reaction to antipsychotic therapy. What signs and symptoms of this syndrome should the nurse identify? (Select all that apply.) 1. Jaundice 2. Diaphoresis 3. Hyperrigidity 4. Hyperthermia 5. Photosensitivity

2. Diaphoresis 3. Hyperrigidity 4. Hyperthermia **Diaphoresis, hyperrigidity, and hyperthermia occur with neuroleptic malignant syndrome as a result of dopamine blockade in the hypothalamus.

A client who is being treated in a mental health clinic is to be discharged after several months of therapy. The client anxiously tells the nurse, "I don't know what I'll do when I can't see you anymore." The nurse determines that the client is: 1. Expressing thanks to the nurse 2. Reacting to the planned discharge 3. Attempting to manipulate the nurse 4. Indicating a need for further treatment

2. Reacting to the planned discharge

A health care provider writes a prescription of "Restraints PRN" for a client who has a history of violent behavior. What is the nurse's responsibility in regard to this order? 1. Asking that the order indicate the type of restraint 2. Recognizing that PRN orders for restraints are unacceptable 3. Implementing the restraint order when the client begins to act out 4. Ensuring that the entire staff is aware of the order for the restraints

2. Recognizing that PRN orders for restraints are unacceptable

After an automobile collision involving a fatality and a subsequent arrest for speeding, a client has amnesia regarding the events surrounding the accident. Which defense mechanism is being used by the client? 1. Projection 2. Repression 3. Suppression 4. Rationalization

2. Repression

A secretary in a home health agency gossips about coworkers and then writes them notes to tell them how valuable they are to the organization and how much she likes working with them. What defense mechanism is being used by the secretary? 1.Denial 2.Undoing 3.Displacement 4.Intellectualization

2.Undoing

A client is diagnosed with chronic pancreatitis. When providing dietary teaching it is most important that the nurse instruct the client to: 1 Eat a low fat, low protein diet 2 Avoid foods high in carbohydrates Correct3 Avoid ingesting alcoholic beverages 4 Eat a bland diet of six small meals a day

3 Avoid ingesting alcoholic beverages Alcohol increases pancreatic secretions, which cause pancreatic cell destruction. Although the diet should be low in fat, it should be high in protein; also, it should be high in carbohydrates. The client should be consuming a sufficient amount of complex carbohydrates each day to maintain weight and promote tissue repair. A bland diet is not necessary, but large, heavy meals should be avoided.

A client is admitted to a psychiatric hospital with the diagnosis of schizoid personality disorder. Which initial nursing intervention is a priority for this client? 1 Helping the client enter into group recreational activities 2 Convincing the client that the hospital staff is trying to help 3 Helping the client learn to trust the staff through selected experiences 4 Arranging the client's contact with others so it is limited while she is in the hospital

3 Helping the client learn to trust the staff through selected experiences

A nurse is preparing a community health program for senior citizens. The nurse teaches the group that the physical findings that are typical in older people include: 1 A loss of skin elasticity and a decrease in libido 2 Impaired fat digestion and increased salivary secretions Correct3 Increased blood pressure and decreased hormone production 4 An increase in body warmth and some swallowing difficulties

3 Increased blood pressure and decreased hormone production With aging, narrowing of the arteries causes some increase in the systolic and diastolic blood pressures; hormone production decreases after menopause. There may or may not be changes in libido; there is a loss of skin elasticity. Salivary secretions decrease, not increase, causing more difficulty with swallowing; there is some impairment of fat digestion. There may be a decrease in subcutaneous fat and decreasing body warmth; some swallowing difficulties occur because of decreased oral secretions.

A male college student who is smaller than average and unable to participate in sports becomes the life of the party and a stylish dresser. What defense mechanism does the nurse determine that the client is using? 1. Introjection 2. Sublimation 3. Compensation 4. Reaction formation

3. Compensation **By developing skills in one area, the individual compensates for a real or imagined deficiency in another, thereby maintaining a positive self-image.

A nurse encourages a client to attend Alcoholics Anonymous (AA) meetings after discharge. What do self-help groups such as AA help their members do? 1. Set long-term goals 2. Limit excessive drinking 3. Foster changes in behavior 4. Identify underlying causes of behavior

3. Foster changes in behavior

A client has been on the psychiatric unit for several days. The client arouses anxiety and frustration in the staff and manipulates them so well that staff members are afraid to approach the client. One morning the client shouts at the nurse, "You've worked it so I can't go for a walk with the group today. You're as cunning as a fox. I hate you! Get out, or I'll hit you!" What is the best response by the nurse? 1."Tell me what I did to upset you." 2"Go ahead and try to hit me if you need to." 3."I don't like hearing your threats, but tell me more about your feelings." 4"You're being rude and your behavior is stopping me from wanting to be with you."

3."I don't like hearing your threats, but tell me more about your feelings."

A family member brings a relative to the local community hospital because the relative "has been acting strange." Which statements meet involuntary hospitalization criteria? Select all that apply. 1."I cry all the time, I am so sad." 2."Since I retired I have been so depressed." 3."I would like to end it all with sleeping pills." 4."Voices say it is okay for me to kill all prostitutes." 5."My boss makes me so angry by always picking on me."

3."I would like to end it all with sleeping pills." 4."Voices say it is okay for me to kill all prostitutes." Rationale: 3. This statement indicates a suicide threat; it is a direct expression of intent but without action. 4. The threat to harm others must be heeded; the client must be protected from self harm as well as harming others.

An 8-year-old girl visits the school nurse frequently with vague ailments. The nurse spends time listening to the girl, takes her temperature, and always sends her back to class. One Thursday the girl tells the nurse that she no longer wants to visit her grandfather in his home because he "hugs me too tight and touches me down there" (pointing to her genitals). She has told her mother that she does not want to spend the weekend with her grandparents, but her mother says that she has no choice. What is the most appropriate action by the nurse? 1 Planning a home visit to discuss with the mother what her child has shared with the nurse 2 Advising the child to tell her mother why she does not want to go to her grandfather's house 3 Arranging a meeting with the principal and the mother to discuss the possibility of child molestation 4 Reporting the alleged abuse to the local child protective agency and encouraging an investigation before the weekend visit

4

Risk for assaultive behavior is highest in the mental health client who does what? 1 Uses profane language 2 Touches people excessively 3 Exhibits a sudden withdrawal 4 Experiences command hallucinations

4

The client is scheduled for an abdominal hysterectomy with a bilateral oophorectomy. As the nurse prepares to have the client sign the informed consent, the client asks how long she should wait to become pregnant. Which action should the nurse take? 1 Have the client sign the informed consent form. 2 Ask the client if she understands what the surgery entails. 3 Tell the client that she will not be able to get pregnant after the surgery. 4 Call the primary healthcare provider immediately and hold preoperative medications

4

Which right of delegation refers to the giving of clear, concise descriptions of a task to the delegatee? 1 Right task 2 Right person 3 Right supervision 4 Right communication

4

A client with diabetes mellitus is able to discuss in detail the diabetic metabolic process while eating a piece of chocolate cake. What defense mechanism does the nurse identify when evaluating this behavior? 1. Projection 2. Dissociation 3. Displacement 4. Intellectualization

4. Intellectualization Rationale: 4. Intellectualization occurs when a painful emotion is avoided by means of a rational explanation that removes the event from any personal significance.

A client with schizophrenia is started on an antipsychotic/neuroleptic medication. The nurse explains to a family member that this drug primarily is used to: 1. Keep the client quiet and relaxed 2. Control the client's behavior and reduce stress 3. Reduce the client's need for physical restraints 4. Make the client more receptive to psychotherapy

4. Make the client more receptive to psychotherapy

A patient who is withdrawing from alcohol is restless and reports seeing snakes on the ceiling. Vital signs are bp 180/100, pulse 92 beats per minute, and respiration 22/breaths per min. What should the nurse do first? 1. teach the patient a relaxation technique 2. administer a dose of lorazepma ( Ativan) 3. search the patient's room for hidden alcohol 4. administer an antihypertensive agent as ordered

4. administer an antihypertensive agent as ordered

Staff on a mental health unit routinely uses positive and negative reinforcement when caring for patients with mental health disorders. What needs to be done before the staff can implement these approaches? 1. have a health care provider order to use the approaches 2. ensure all patients have medication orders for sedatives 3. place a set of 4 point restraints within each patients room 4. have the patient sign a consent form agreeing to this approach

4. have the patient sign a consent form agreeing to this approach

A patient with schizophrenia has not bathed recently and a family member states that the patient has not been out of the house for 10 days. The patient tells the nurse, "They are trying to hurt me; don't let them hurt me" Which symptom is the patient demonstrating? 1. paranoid delusions 2. grandiose delusions 3. auditory delusions 4. persecutory hallucinations

4. persecutory hallucinations

C, D

A 2-year-old child is admitted with multiple fractures and bruises, and abuse is suspected. Which nursing assessment findings support this suspicion? Select all that apply. A: Bedwetting B: Thumb-sucking C: Difficulty consoling D: Underdevelopment for age E: Demands for physical closeness

B, D

A 2.5-year-old child is admitted for treatment of injuries supposedly sustained in a fall down a flight of stairs. Child abuse is suspected. What statements might the nurse expect from a parent who engages in child abuse? Select all that apply. A: "Kids have to learn to be careful on the stairs." B: "Every time I turn around the kid is falling over something." C: "This child tends to be adventurous and doesn't understand about getting hurt on the stairs." D: "I can't understand it. This child didn't have a problem using the stairs without my help before this." E: "I try to keep an eye on my child, but little kids are always on the go and I just can't keep running after the kid."

D

A client with a history of violence is becoming increasingly agitated. Which nursing intervention will most likely increase the risk of acting-out behavior? A: Being assertive B: Responding early C: Providing choices D: Teaching relaxation

A nurse in a community therapeutic recreation program is working with a client with dysthymia. The treatment plan suggests group activities when possible for this client. What is the priority rationale for this intervention?

A group can offer increased support.

The nurse observes the client as he paces the floor and talks to himself. The nurse documents, "client confused and agitated." This is an example of

A judgment

C

A nurse in the emergency department is assessing a client who has been physically and sexually assaulted. What is the nurse's priority during assessment? A: The family's feelings about the attack B: The client's feelings of social isolation C: The client's ability to cope with the situation D: Disturbance in the client's thought processes

C

A nurse may best assist abusive parents in altering behavior toward their abused 2-year-old child by helping them do what? A: Recognize what behavior is appropriate for a toddler. B: Learn appropriate ways of punishing a toddler's inappropriate behavior. C: Identify the specific ways in which the toddler's behavior provokes frustration. D: Ignore the toddler's negative nondestructive behavior while supporting acceptable behavior.

B

A nurse on the pediatric unit is assigned to care for a 2-year-old child with a history of physical abuse. What does the nurse expect the child to do? A: Smile readily at anyone who enters the room. B: Be wary(adj. 谨慎的) of physical contact initiated by anyone. C: Begin to scream when the nurse nears the bedside. D:Pay little attention to the nurse standing at the bedside

A male client with a diagnosis of bipolar disorder is admitted to an inpatient unit during severe manic episode. As a result of guidelines implemented by the Health Care Financing Administration in 1983, the client's medicare will pay for his stay in the unit for:

A pre-determined length of time based on the diagnosis.

13. A scientific structuring of the environment in order to effect behavioral changes and to improve the psychological health and functioning of the individual is defined as ________________________ therapy.

ANS: milieu Rationale: Milieu therapy is defined as a scientific structuring of the environment in order to effect behavioral changes and to improve the psychological health and functioning of the individual. The goal of milieu therapy is to manipulate the environment so that all aspects of the client's hospital experience are considered therapeutic.

A nurse is working in a daycare center with clients who have cognitive impairments. What does the nurse expect of a client in the middle stages of dementia?

Able to recall events from the past

By the ages of 14 to 17, teens are able to demonstrate problem solving skills using concepts, generalizations, and being flexible in planning actions and goals. What is this an example of?

Abstract thinking

Abuse and neglect

Abuse is causing harm to or maltreating another.

What is most important for the nurse to do when caring for a client who is in an alcohol detoxification program?

Accept the client as a worthwhile person

When working with a client who is in an alcohol detoxification program, it is most important for the nurse to:

Address the client's holistic needs

Shortly after the death of her husband after a long illness, a woman visits the mental health clinic complaining of malaise, lethargy, and insomnia. The nurse, knowing that it is most important to help the wife cope with her husband's death, should attempt to determine which information?

Adequacy of the wife's support system

Signs and symptoms of schizophrenia often are first seen in:

Adolescence

How should a nurse characterize a sudden terrorist act that causes the deaths of thousands of adults and children and negatively affects their families, friends, communities, and the nation?

Adventitious An adventitious crisis is a crisis or disaster that is unplanned and accidental; its subcategories include national disasters and crimes of violence.

__________ is how the client displays his or her emotions through facial, vocal, or gestural behavior.

Affect

temper tantrum

An outburst of anger, especially a childish display of rage or bad temper

A 22-year-old male client with the diagnosis of schizophrenia has been in a mental health facility for approximately 2 weeks. After his parents visit he is seen pacing in the hall, talking loudly to himself. What should the nurse's initial intervention be? Obtaining a prescription for a tranquilizer Asking the client about the events of his day Calling the parents to find out what happened Assigning a nursing assistant to remain with the client

Asking the client about the events of his day A broad opening encourages communication that may elicit the client's perception of the day's events. Obtaining a prescription for a tranquilizer is premature. What is most important is the client's, not the parents', perception of what has occurred. Assigning a nursing assistant to remain with the client is premature; there are no data to indicate that the client may harm himself or others.

People who experience psychotic disorders lose: A) The will to continue B) Contact with reality C) The ability to comply with treatment D) Contact with intellectual functions

B. Contact with reality

What is the main issue for adolescents with anorexia? A) Anxiety B) Control C) Body image D) Appropriate behavior

B. Control

A nurse on the psychiatric unit is assigned to work with a male client who appears reclusive and distrustful of everyone. The nurse can help the client develop trust by:

Being prompt(신속한) for their scheduled meetings

What should the nurse identify as the foremost basis for the development of schizophrenia?

Biological perspective

The 39-year-old daughter of a client with a terminal illness tells the nurse that she thinks something is wrong with her because she frequently cries, is often sad, and can't imagine losing her mother. The nurse assures the daughter that these are normal feelings associated with: A) Bereavement-related depression B) Complicated grief C) Anticipatory grief D) Caregiver grief

C. Anticipatory Grief

The nurse is answering questions from a client and his family regarding a recent diagnosis of Alzheimer's disease. The client asks how effective medication is in treating the disease. What is the nurse's best response? A) "There is no cure or treatment for Alzheimer's disease." B) "Medications have shown little improvement in symptoms." C) "Medications for the disease have been found to improve thinking abilities, behavior, and daily functioning in some clients." D) "Alternative therapies, such as co-enzyme Q-10 and Ginkgo biloba, are more effective than any of the prescription medications used to treat the symptoms."

C. Medications for the disease have been found to improve thinking abilities...

Which are key features of anorexia nervosa? Select all that apply. A) Excessive laxative use B) Purging C) Severe weight loss D) Introverted personality E) Hunger is denied

C. Severe Weight loss, D. Introverted personality and E. Hunger is denied

Posttraumatic stress disorder

Can develop following an extremely traumatic event that involves injury or threat to the child

What should a nurse recognize that a client who uses the defense mechanism of sublimation is doing?

Channeling unacceptable impulses into socially approved behavior

A nurse is teaching a client about side effects of medications. Which drug will cause a heightened skin reaction to sunlight?

Chlorpromazine Clients taking chlorpromazine should be instructed to stay out of the sun. Photosensitivity makes the skin more susceptible to burning.

A nurse suggests a crisis intervention group to a client experiencing a developmental crisis. The nurse knows that these groups are successful because the:

Client is assisted in investigating alternative approaches to solving the identified problem

A recently married 22-year-old woman is brought to the trauma center by the police. She has been robbed, beaten, and sexually assaulted. The client, although anxious and tearful, appears to be in control. The health care provider prescribes alprazolam (Xanax) 0.25 mg for agitation. The nurse should administer this medication when the:

Client requests something to calm her

What client's, may a care provider utilize a risk factor assessment tool (pg.95)?

Client's who may pose a risk for violence toward themselves and others

A client is admitted for treatment of obsessive-compulsive disorder that is interfering with activities of daily living. Which medication should the nurse anticipate the health care provider will prescribe?

Clomipramine (Anafranil) Clomipramine (Anafranil) potentiates the effects of serotonin (antiobsessional effect) and norepinephrine in the central nervous system; it diminishes obsessive-compulsive behaviors. Benztropine (Cogentin) is an antiparkinsonian agent, not an antianxiety agent. Amantadine (Symmetrel) is an antiparkinsonian agent, not an antianxiety agent. Diphenhydramine (Benadryl) is an antihistamine, not an antianxiety agent.

The child in early adolescence experiences developmental issues with his or her identity as evidenced by:

Conforming to group norms

A client has been prescribed chlorpromazine (Thorazine) for the management of positive symptoms of schizophrenia. When the client reports difficulty sustaining an erection, the nurse: Reassures him this side effect will resolve in a few weeks Consults with his provider regarding alternative medication therapies Explains that all conventional antipsychotic medications cause impotence Provides additional medication education to explain the medication's side effects in detai

Consults with his provider regarding alternative medication therapies Although erectile dysfunction can result from conventional antipsychotic medication therapy, the provider is often able to prescribe an alternative medication that will help manage the symptoms but is less likely to cause the dysfunction. Education regarding side effects is certainly appropriate, but such information will only confirm that the side effect is not likely to subside with time.

A client has been in an acute care psychiatric unit for 3 days and is receiving haloperidol (Haldol) tablets orally to reduce agitation and preoccupation with auditory hallucinations. There has been no decrease in the client's agitation or preoccupation with auditory hallucinations since the medication was started. What should the nurse's priority intervention be? 1 Asking the health care provider to change the medication 2 Making certain that the client is swallowing the medication 3 Concluding that a therapeutic level of the drug has not been achieved 4 Securing a prescription for as-needed sedation until the client calms down

Correct2 Making certain that the client is swallowing the medication

A nurse is caring for several clients who have severe psychiatric disorders. What is the major reason that a health care provider prescribes an antipsychotic medication for these clients? 1 To improve judgment 2 To promote social skills 3 To diminish neurotic behavior 4 To reduce the positive symptoms of psychosis

Correct4 To reduce the positive symptoms of psychosis

A female nurse has been caring for a depressed 75-year-old woman who reminds her of her grandmother. The nurse spends extra time with her every day and brings her home-baked cookies. The nurse's behavior reflects:

Countertransference

A nurse is working with a couple and their two children. The 14-year-old son has been in trouble at school because of truancy and poor grades. The 16-year-old daughter is quiet and withdrawn and refuses to talk to her parents. The parents have had severe marital problems for the past 10 years. The priority nursing concern at this time is how the:

Couple's marital(결혼,부부 생활)의 problems are affecting their children

A 24-year-old client complains to the nurse in the women's health clinic that her breasts become tender before her menstrual period. What should the nurse recommend that the client do 1 week before an expected menses?

Decrease caffeine intake.

A client with type 1 diabetes is found to have a psychosis and is to receive haloperidol (Haldol). Which response should a nurse anticipate with this drug combination?

Decreased control of the diabetes

One morning a client tells the nurse, "My legs are turning to rubber because I have an incurable disease called schizophrenia." The nurse identifies that this as an example of: Hallucinations Paranoid thinking Depersonalization Autistic verbalization

Depersonalization The state in which the client feels unreal or believes that parts of the body are distorted is known as depersonalization or loss of personal identity. This is not an example of a hallucination; a hallucination is a sensory experience for which there is no external stimulus. The client's statement does not indicate any feelings that others are out to do harm, are responsible for what is happening, or are in control of the situation. The statement is not an example of autistic verbalization.

A female client is admitted to the hospital after attempting suicide. She reveals that her desire for sex has diminished since her child's birth 3 years ago. What is most directly related to the client's loss of interest in sex?

Depression

Depression

Describes a symptom, an emotional state, and a clinical syndrome

The nurse determines that the therapy that has the highest success rate for people with phobias is:

Desensitization involving relaxation techniques

Define "evaluation" of the nursing process (pg.94)

Determines the effectiveness of care ( the final phase)

What is the most difficult initial task in the development of a nurse-client relationship?

Developing an awareness of self and the professional role in the relationship

Secondary enuresis

Develops after child has achieved bladder control

pervasive developmental disorders

Difficulty with language and failure to acquire or early loss of social skills - autism spectrum disorder - rett syndrome asperger s syndrome

A nurse concludes that a client is using displacement. Which behavior has the nurse identified?

Directing pent-up emotions at someone other than the primary source

A male long-distance jumper improves his distance by 3½ inches (7 cm) and earns the praise of his coach, but on another day, when he does not reach his mark, he forcefully kicks the door of his locker. What defense mechanism does his outburst demonstrate?

Displacement

A nurse greets a client who has been experiencing delusions of persecution and auditory hallucinations by saying, "Good evening. How are you?" The client, who has been referring to himself as "the man," answers, "The man is bad." Of what is this an example?

Dissociation

A female client is taking an antipsychotic medication for her schizophrenia. The nurse monitors this client for peripheral nervous system side effects of

Dry mouth, photophobia, and hypotension

Learning disorders Examples:

Dyslexia , ADHD

The major mental health problems of childhood are grouped into seven categories.

Each child is an individual.

Disruptive behavioral conduct disorder

Early diagnosis and treatment are very important if these children are to become productive members of society.

A nurse understands that autism is a form of a pervasive developmental disorder (PDD). Which factor unique to autism differentiates it from other forms of PDD?

Early onset, before 36 months of age

Schizophrenia

Early recognition and treatment are important because schizophrenia is often a long-term disorder.

A disturbed client starts to repeat phrases that others have just said. How should the nurse document this speech?

Echolalia

A nurse recalls that language development in the autistic child resembles:

Echolalia The autistic child repeats sounds or words spoken by others.

How many stages of the human life cycle did Erik Erikson identify?

Eight

A male client with schizophrenia lives in an assisted-living complex for individuals with mental health disorders. He is tired of the Parkinson-like symptoms he experiences with his antipsychotic medication and therefore stops taking his medication after much discussion with his treatment team. He is progressively withdrawing from reality but is not a safety risk at this point to himself or others. What is the best response of the nurse and treatment team?

Ensure that the client and those around him are safe, and monitor for additional symptoms of his schizophrenia while maintaining trust with the client.

During a session with a female client with a diagnosis of social phobia, she talks about how proud she is of herself because she was finally able to shop at the grocery store. The nurse documents the events and knows that this would be considered which phase of the nursing process?

Evaluation

A 15-year-old girl is being admitted to an inpatient mental health clinic with the diagnosis of anorexia nervosa. The nurse knows that the most common personality characteristic of teens affected with this disorder is:

Excessive cooperation

According to Erikson, an individual who fails to master the maturational crisis of adolescence will most often:

Experience role confusion. According to Erikson, adolescents are struggling with identity versus role confusion. Rebellion against parental orders reflects part of the struggle for independence; it does not indicate failure to resolve the conflicts of adolescence. Adolescents tend to be group oriented, not isolated; they struggle to belong, not to escape. Adolescents may experiment with drug and alcohol use, but most of them do not become abusers.

Risk for assaultive behavior is highest in the mental health client who:

Experiences command hallucinations

Adolescents who become chemically dependent progress through four stages. Place these stages in order of occurrence.

Experimentation Active seeking Preoccupation Burnout

_____________ side effects can occur when antipsychotic medications are taken that manifest as abnormal movements such as akathisia

Extrapyramidal

A nurse is caring for a client with a bipolar disorder depressive episode. What should the nurse's objective for this client be?

Feeling comfortable with the nurse

The grieving spouse of a client who has just died says to the nurse, "We should have spent more time together. I always felt that my work came first." What should the nurse conclude that the spouse is experiencing?

Feelings of guilt

Criterion for inpatient admission

Feels that he is no longer able to cope with life stressors or maintain control of behavior

The nurse explains to a nursing assistant that behavior usually is viewed and accepted as normal if it:

Fits within standards accepted by one's society

During an interview of a client with a diagnosis of bipolar I disorder, manic episode, the nurse expects the client to demonstrate:

Flight of ideas

During an interview of a client with a diagnosis of bipolar I disorder, manic episode, the nurse expects the client to demonstrate:

Flight of ideas Flight of ideas is a fragmented, pressured, nonsequential pattern of speech typically used during a manic episode.

A client with a diagnosis of schizophrenia is discharged from the hospital. At home the client forgets to take the medication, is unable to function, and must be rehospitalized. What medication may be prescribed that can be administered on an outpatient basis every 2 to 3 weeks?

Fluphenazine

After a traumatic event, a client is extremely upset and exhibits pressured and rambling speech. What therapeutic technique can the nurse use when a client's communication rambles?

Focusing Focusing is indicated when communication is vague; the nurse attempts to concentrate or focus the client's communication on one specific aspect. Touch invades the client's space and will not help focus the client's communication. Silence prolongs the rambling communication; the client needs to be focused. Until the concern is identified and explored, summarizing is impossible.

What is the best nursing intervention to encourage a socially withdrawn client to talk?

Focusing on nonthreatening subjects

What interview method serves as a starting point for building the therapeutic relationship with the client and includes obtaining information (pg.94)?

Formal interview

What information are gathered during the holistic assessment (pg.94)?

Gather information about the physical, intellectual, emotional, social, cultural and spiritual aspects of each client.

According to Erikson, a person's adjustment to the period of senescence 노쇠 will depend largely on the adjustment the individual made to the earlier developmental stage of:

Generativity versus stagnation

Psychobiology theory considers the causes of mental illness to be which of the following? (Select all that apply.)

Genetics Neurotransmitter activity Immune system dysfunction

A 4-year-old client in a pediatric unit is imitating the actions of the nurse. The nurse knows, according to Erik Erikson's theory, that this child is displaying a characteristic seen during which developmental stage?

Genital-locomotor

A client is admitted to the psychiatric hospital after many self-inflicted nonlethal injuries over the preceding month. Of which level of suicidal behavior is the client's behavior reflective?

Gestures

A nurse is planning activities for a withdrawn client who is hallucinating. Which activity will be most therapeutic for the client?

Going for a walk with the nurse Going for a walk with the nurse facilitates one-on-one interaction and the development of a trusting relationship.

Environmental Problems

Growing up is a difficult process for children who are poor, homeless, abused, or neglected.

A client with the diagnosis of schizophrenia refuses to eat meals. Which nursing action is most beneficial for this client? Directing the client repeatedly to eat the food Explaining to the client the importance of eating Waiting and allowing the client to eat whenever the client is ready Having a staff member sit with the client in a quiet area during mealtimes

Having a staff member sit with the client in a quiet area during mealtimes By sitting with the client during mealtimes the nurse can evaluate how much the client is eating; this encourages the client to eat and begins the construction of a trusting relationship. Fewer distractions may help the client focus on eating. The client will not follow directions to eat because of the nature of the illness. Explaining the importance of eating and allowing the client to eat when ready are both unrealistic and will not ensure adequate intake.

A client on the psychiatric unit is undergoing a pretreatment evaluation for electroconvulsive therapy (ECT). Because of the client's profoundly depressed behavior, the nurse doubts that the client can provide informed consent. What should the nurse's initial intervention be?

Having the client verbalize her understanding and the outcomes of the procedure

Without assessment of six specific aspects of an individuals being, the mental health nurses scope of care is narrow and limited in effectiveness. These aspects include social, physical, cultural, intellectual, emotional, and spiritual areas of a persons life, known as a(n) __________ assessment.

Holistic

Which feelings should a nurse anticipate a client with bulimia nervosa to report experiencing during an episode of binge eating? (Select all that apply.)

Hopelessness Powerlessness

A psychiatric nurse is hired to work in the psychiatric emergency department of a large teaching hospital. While reviewing the manuals, the nurse reads, "People with mental health emergencies shall be triaged within 5 minutes of entering the emergency department." What does the nurse consider this statement to represent?

Hospital policy

When educating a client being treated with lithium, which item(s) in his or her diet should be monitored or avoided?

Hot dogs and ham

A husband is upset that his wife's alcohol withdrawal delirium has persisted for a second day. What is the most appropriate initial response by the nurse? -"I see that you're worried. We're using medication to ease your wife's discomfort." -"This is expected. I suggest that you go home because there's nothing you can do to help." -"Are you afraid that your wife will die? I assure you, very few alcoholics die during the detoxification process." -"Are you worried that your wife is uncomfortable while she's going through withdrawal? I'm sure that she's not in pain."

I see that you're worried. We're using medication to ease your wife's discomfort

Freud's psychoanalytical theory states that an adult is more likely to be mentally healthy if there is a balance between which parts of the mind? (Select all that apply.)

Id Ego Super-ego

A female client with low self-esteem tries to dress and act like the nurse who cares for her in an outpatient clinic setting. This behavior is an example of which defense mechanism?

Identification

To therapeutically relate to parents who are known to have maltreated their child, the nurse must first:

Identify personal feelings about child abusers

A young client is admitted to the hospital with a diagnosis of acute schizophrenia. The family reports that one day the client looked at a linen sheet on a clothesline and thought it was a ghost. What is the most appropriate conclusion to make about what the client was experiencing? Illusion Delusion Hallucination Confabulation

Illusion An illusion is a misinterpretation of an actual sensory stimulus. A delusion is a false, fixed belief. A hallucination is a false sensory perception that occurs with no stimulus. Confabulation is a filling in of blanks in memory.

Growth

Increase in physical size

Erik Erikson posited life as a sequence of achievements. Place the levels of development in their order of achievement according to Erikson's theory.

Industry versus inferiority is the conflict of the school-aged child; identity versus role confusion is the conflict associated with adolescence; intimacy versus isolation is the conflict that occurs during young adulthood; generativity versus stagnation is the conflict of adulthood; and integrity versus despair is the conflict that occurs at maturity.

A nurse caring for a pregnant woman determines that she is engaging in the practice of pica. Why should the nurse prepare a teaching plan for this client?

Inedible items are being ingested.

Rumination disorder

Infant regurgitates and rechews food.

Anxiety

Infants and toddlers experience separation anxiety.

An adult female client has been diagnosed recently with mild depression but opts not to take the medication prescribed by her physician after talking with the physician about the benefits, risks, possible outcomes, and side effects. She decides to investigate alternative treatments. This client is making this decision based on the premise of:

Informed consent

The clients on a mental health unit go on a supervised day trip to a baseball game. When returning to the bus, a client with a narcissistic personality disorder insists on leaving the group to get an autograph from a player. What is the most appropriate response by the nurse?

Informing the client in a matter-of-fact tone that everyone must remain with the group

Which suicide method is the least lethal(치명적인)?

Ingesting pills

A client with a history of delusions demonstrates which of the following behaviors?

Insists the government is out to harm them

When having a conversation with a nurse, an older client states, "I've lived a good life. I don't want to die, but I accept it as a part of life." What developmental stage, according to Erikson, has the client completed?

Integrity

A 65-year-old man is admitted to the hospital with a history of depression. The client, who speaks little English and has had few outside interests since retiring, says, "I feel useless and unneeded." The nurse concludes that the client is in Erikson's developmental stage of:

Integrity versus despair is the task of the older adult; the client has difficulty accepting what life is and was, resulting in feelings of despair and disgust. Initiative versus guilt is the task of the preschool-aged child. Intimacy versus isolation is the task of the young adult. Identity versus role confusion is the task of the adolescent.

A client with a history of alcoholism is found to have Wernicke encephalopathy associated with Korsakoff syndrome. What does the nurse anticipate will be prescribed?

Intramuscular injections of thiamine Thiamine is a coenzyme necessary for the production of energy from glucose. If thiamine is not present in adequate amounts, nerve activity is diminished and damage or degeneration of myelin sheaths occurs.

An adult is found to have schizotypal personality disorder. How should a nurse describe the client's behavior?

Introverted and emotionally withdrawn

A client with a history of sleeplessness, lack of interest in eating, and excessive purchases on charge accounts is seen in the mental health clinic. The adaptation that the nurse should expect the client to exhibit is:

Intrusive involvement with environmental activities

Why are physical assessment skills important to the data gathering process (pg.94)?

It is uses to gather data , investigate changes in physical conditions, and evaluate the effectiveness of therapeutic interventions

A client has been taking escitalopram (Lexapro) for treatment of a major depressive episode. On the fifth day of therapy the client refuses the medication, stating, "It doesn't help, so what's the use of taking it?" What is the best response by the nurse?

It usually takes 1 to 4 weeks to attain a therapeutic blood level of escitalopram (Lexapro). Six to 8 weeks is too long. The client needs more time, not an increased dosage, to see an effect of the medication. There is no need for the nurse to notify the health care provider yet.

An agitated, acting-out, delusional client is receiving large doses of haloperidol (Haldol), and the nurse is concerned because this drug can produce untoward side effects. Which clinical manifestations should alert the nurse to stop the drug immediately? (Select all that apply.)

Jaundice Tachycardia

A client comes to the crisis center because her spouse has stated that he wants a divorce. The client states that she is angry and feels rejected. What should the nurse encourage the client to do to cope with this emotional trauma?

Learn to constructively vent her anger.

How can the nurse best minimize psychological stress in an anxious client who has been admitted to the psychiatric unit?

Learn what is of particular importance to the client

Anorexia nervosa is best described as a disorder that is potentially: A) Serious B) Uncommon C) Psychiatric D) Life threatening

Life threatening

What diagnoses of a mentally ill patient relate to the client's physical or mental dysfunctions (pg.93)?

Medical diagnoses

What data is collected for a mentally ill patient (pg.93)?

Medical records are reviewed, a history is obtained, and observations are made; discussions with family members or friends add to the database.

A male adolescent client tells the nurse that he is almost positive that he is homosexual. This realization most likely has occurred during the developmental period of:

Middle adolescence

According to the DSM-IV-TR Axis guidelines, clinical disorders are described as:

Mood disorder, substance abuse, and schizophrenic disorders

A client with schizophrenia who has type II (negative) symptoms is prescribed risperidone (Risperdal). Which outcomes indicate that the medication has minimized these symptoms? (Select all that apply.)

More interest is shown in unit activities. The client performs activities of daily living independently.

A client's admitting history indicates signs of akathisia. What clinical finding should the nurse expect when observing for this condition?

Motor restlessness

Causes:

Neglect

A female client who has had bipolar disorder for several years decides to stop all of her medications because she is tired of the side effects. She also cancels all appointments with her therapist, stating that it is just too difficult to plan the visits in her hectic schedule. This client is considered:

Noncompliant

A group of clients from a psychiatric unit, accompanied by staff members, are going to a professional baseball game. The purpose of visits into the community under the supervision of staff members is:

Observing the clients' abilities to cope with a more complex society

A client seen in the emergency department is noted to be stuporous. Which of the following assessment findings would be of most concern?

Painting furniture in a windowless room

Social development and forming an identity are important tasks for the adolescent. Which of the following is true regarding the function of a peer group? (Select all that apply.)

Provides stability during change Helps loosen ties to family Serves as standard for dress and behavior Helps to define future roles

Treatment

Providing a stable environment and consistently enforced limits

Psychotropic medications can cause a parasympathetic and/or sympathetic response from the autonomic nervous system. Which of the following is considered a sympathetic response?

Pupil dilation

What should a nurse ensure when creating an environment that is conducive to psychological safety?

Realistic limits are set.

A client does not participate in group sessions due to feeling inferior to others in the group. Based on cognitive theory, he would be directed to:

Recognize and change his negative thoughts

What is the best nursing intervention when the language of a client in the manic phase of a bipolar disorder becomes vulgar and profane?

Recognizing that the behavior is part of the illness but setting limits on it

Incidences of child molestation often are revealed years later when the victim is an adult. Which defense mechanism reflects this situation?

Repression is a coping mechanism in which unacceptable feelings are kept out of conscious awareness; later, under stress or anxiety, thoughts or feelings surface and come into one's conscious awareness. Regression is the use of an unconscious coping mechanism through which a person avoids anxiety by returning to an earlier, more satisfying, or comfortable time in life.

The nurse is administering medications to a client with a diagnosis of paranoid schizophrenia. The nurse would expect to see which medication ordered for this client?

Risperdal

The nurse interviews a young female client with anorexia nervosa to obtain information for the nursing history. What will the client's history most likely reveal? (Select all that apply.)

Ritualistic behaviors Desire to improve her self-image

After caring for a terminally ill client for several weeks, a nurse becomes increasingly aware of a need for a respite from this assignment. What is the best initial action by the nurse?

Seeking support from colleagues on the unit

When reviewing the medications for a group of clients on a psychiatric unit, the nurse concludes that the pharmacotherapy for anxiety disorders is moving away from benzodiazepines and moving toward:

Selective serotonin reuptake inhibitors Selective serotonin reuptake inhibitors have better safety profiles and do not carry the risk of substance abuse and tolerance.

During a conversation with a male client, he voices that he really appreciates his family, likes his job, and enjoys groups in which he volunteers. According to Maslow's theory, what is this client experiencing?

Self-actualization

A 16-year-old teenage boy who is bullied at school has recently started staying in his room and not associating with his friends. His grades are dropping and he refuses to eat dinner with his family. What actions should his parents be advised to take?

Set limits with him in a respectful manner and assist him to problem solve.

Childhood disintegrative disorder

Severe regression in many areas after 2 years of normal development

While a client with an abdominal aortic aneurysm is being prepared for surgery, the client complains of feeling light-headed. The client is pale and has a rapid pulse. What does the nurse conclude that the client's symptoms indicate?

Shock

An autistic toddler is sitting in a corner, rocking and spinning a top. How can the nurse be most therapeutic when approaching this toddler?

Sitting with the toddler while watching the spinning top to provide a nonintrusive presence

In a research study of eating disorders, it was found that the most frequent weight loss method used by female high school students was:

Skipping meals

In preparing discharge planning for a client who has been prescribed lithium for the treatment of bipolar disorder, the nurse must be sure that the client demonstrates an understanding of the need to monitor his or her diet for intake of:

Sodium

What happens during the planning phases for a mentally ill patient (pg.93)?

Specific short-term and long-term goals are developed

A young man who is socially inept and rarely dates takes a job assisting a photographer who specializes in photographs of female nude models. What defense mechanism is being used by the assistant photographer?

Sublimation Sublimation is the channeling of instinctual drives into acceptable activities. The photographer is finding a safe outlet for his sexual drive. Denial is defined as an unconscious refusal to admit an unacceptable idea or behavior. Suppression is the voluntary exclusion from awareness of anxiety-producing feelings, ideas, and situations. Displacement is the transfer of feelings related with a particular person, object, or experience to a nonthreatening person, object, or experience.

A client leaves group therapy in the middle of the session. The nurse finds the client obviously upset and crying, and the client tells the nurse that the group's discussion was too much to tolerate. What is the most therapeutic initial nursing action at this time? -Request kindly but firmly that the client return to the group to work out conflicts. -Suggest that the client accompany the nurse to a quiet place so that they can talk about the situation. -Ask the group leader what happened in the group session and base interventions on this additional information. -Respect the client's right to decline therapy at this time and report the incident to the rest of the health team members.

Suggest that the client accompany the nurse to a quiet place so that they can talk about the situation.

A 19 year-old, arrested for assault and robbery, has a history of truancy and prostitution but is unconcerned that her behavior has caused emotional distress to others. The diagnosis of antisocial personality disorder is made. According to psychoanalytical theory, the client's lack of remorse and repetitive behavior probably are related to an underdeveloped:

Superego Lack of remorse 후회 indicates a weak superego, the aspect of personality concerned with prohibitions.

Which client characteristic is an initial concern for the nurse when caring for a client with the diagnosis of paranoid schizophrenia?

Suspicious(의혹을 갖는, 수상쩍어 하는) feelings

A client has been taking prescribed risperidone (Risperdal) 3 mg twice a day for the past 8 days. A friend brings the client to the outpatient clinic. The client reports tremors, shortness of breath, a fever, and sweating. What should the nurse do?

Take the client's vital signs and arrange for immediate transfer to a hospital

A high school adolescent undergoing lithium therapy has not been following the prescribed pharmacological regimen. What should the school nurse do to help promote adherence to the regimen?

Talk with the adolescent about the importance of a consistent lithium blood level in ensuring that the medication will be effective.

An adolescent on the psychiatric unit has an angry outburst toward another client who cut in front of people standing in line to get their mail. Later the nurse conducts a one-on-one therapeutic session with the angry client. What is an appropriate short-term goal for the client to strive for?

Talking about the situation that precipitated the anger

The nurse is developing a teaching plan for a client who has been diagnosed recently with a mental health disorder and has been prescribed a psychotropic medication. Which interventions regarding the medication should the nurse include in the teaching plan? (Select all that apply.)

Teach signs and symptoms of side effects and what to do if these occur Provide written information regarding the purpose, dosage, route, and dosing schedule Ask the client and significant other to verbally explain when it is necessary to contact the physician should side effects occur

A nurse is working with a 15 year old girl and her parents on a treatment plan for her attention-deficit/hyperactivity disorder (ADHD). The nurse should be sure to:

Teach the parents how to structure and enforce limits on their daughter's behavior that appropriate to her condition

Temper tantrums are a common expression of anger and frustration.

Temper tantrums occur in 50% to 80% of children in this age group.

Internal developmental problems are seen as a causative factor for some behavioral and family problems during adolescence. Psychological developmental issues that can lead to problems during late adolescence (17 to 20 years old) include:

Tendency to withdraw when upset

A female client in the psychiatric unit has been monopolizing a group discussion about prenatal care for more than 10 minutes, sharing her feelings about the way in which her husband has treated her. The nurse conducts a quick assessment of the group and finds that about half of the clients are inattentive. Which is the most appropriate nursing intervention?

Thanking the client for sharing her feelings and asking what the other group members have on their minds

A client who is taking lithium arrives at the mental health center for a routine visit. The client has slurred speech, has an ataxic gait, and complains of nausea. The nurse knows that these signs and symptoms are:

The classic signs and symptoms of lithium toxicity include slurred speech, ataxia, nausea, and vomiting. When the lithium level is low the client presents with recurring signs and symptoms of the mood disorder. These are not signs and symptoms of a mood disorder. If the lithium level is within the therapeutic range, the client's mood is more stable; the client may experience gastrointestinal symptoms but will not experience slurred speech or an ataxic gait.

What phase of the nursing process, compares expected outcomes with actual results to note which actions met the goals and which did not (pg.94)?

The evaluation phase

What is the most important information for a nurse to teach to prevent relapse in a client with a psychiatric illness?

The need to follow the prescribed medication regimen Following the prescribed medication regimen is important because side effects and denial of illness may cause clients to stop taking their medications; this is a common cause of relapse or recurrence of symptoms.

A teenage girls boyfriend of 1 year ended their relationship and began to date someone else, resulting in the girls attempting suicide by taking an overdose of her mothers sleeping medication. What is the most likely cause of this girls suicide attempt?

The need to influence others

What is the most important tool a nurse brings to the therapeutic nurse-client relationship?

The nurse brings to a therapeutic relationship the understanding of self and basic principles of therapeutic communication; this is the unique aspect of the helping relationship. Knowledge of psychopathology, advanced communication skills, and years of experience in the field all support the psychotherapeutic management model, but none is the most important tool used by the nurse in a therapeutic relationship.

A teenager with anorexia nervosa is admitted to the adolescent unit of a mental health facility and signs a contract calling for her to gain weight or lose privileges. There is no weight gain after a week. What should the nurse explain to the client?

The prearranged consequences will go into effect.

Therapeutic Actions

The primary goals of therapeutic interventions for children with mental health problems include early identification and treatment.

A client on the psychiatric unit who has suicidal ideas says to the nurse, "I signed myself in. I'll sign myself out." What concept provides the basis for the nurse's response?

The priority is to keep the client safe; a client admitted on a voluntary basis may be kept involuntarily if professional judgment indicates that the client may harm him- or herself or others. Clients who admit themselves voluntarily may leave if they are not suicidal or homicidal and if unit procedures are followed. The nurse has a duty to maintain the client's safety; a suicidal client may not leave under any circumstances.

Which psychotherapeutic theory uses hypnosis, dream interpretation, and free association as methods to release repressed feelings?

The psychoanalytical model studies the unconscious and uses the strategies of hypnosis, dream interpretation, and free association to encourage the release of repressed feelings

Selective serotonin reuptake inhibitors (SSRIs) are most health care providers' drug of choice for the treatment of depression because:

The side effects are more manageable than with other antidepressants

The parents of a 13-year-old girl are concerned that their daughter has a gender identity disorder. Which sign or symptom most indicates that their concern is correct?

Their daughter frequently wears clothes designed for males.

When communicating with a client with a psychiatric diagnosis, the nurse uses silence. When silence is used in therapeutic communication, clients should feel:

There is no hurry to answer. Silence is a tool employed during therapeutic communication that indicates that the nurse is listening and receptive; it allows the client time to collect thoughts, gain control of emotions, or speak without hurrying. Silence should be comfortable and should not create pressure to talk. The client should feel that she has an opportunity to think about the interaction. The client's perception that the nurse expects that further communication is unnecessary will close communication.

A nurse reminds a client that it is time for group therapy. The client responds by shouting, "You're always telling me what to do, just like my father!" What defense mechanism is the client using?

Transference

Posttraumatic stress disorder

Traumatic events are relived repeatedly, and the child goes to great lengths to avoid anything associated with the trauma.

Posttraumatic stress disorder

Treatment focuses on early recognition and emotionally supportive care.

Problems With Eating and Elimination

Treatment focuses on ruling out a cause.

What characteristic uniquely associated with psychophysiological disorders differentiates them from somatoform disorders?

Underlying pathophysiology

A client's hands are raw and bloody from a ritual involving frequent handwashing. Which defense mechanism does the nurse identify?

Undoing

The parents of an adolescent who engages in self-injurious cutting behavior ask the nurse why their child self-mutilates. What should the nurse give as the reason for the cutting?

Way to manage overwhelming feelings

The more important measure is the child's adaptive functioning.

We must define how well the child copes with the demands of life.

A client in the early dementia stage of Alzheimer's disease is admitted to a long-term care facility. Which activities must the nurse initiate? (Select all that apply.)

Weighing the client once a week Having specialized rehabilitation equipment available Establishing a schedule with periods of rest after activities

Primary nocturnal enuresis

Wetting the bed at night

B, C

What behavioral findings correspond to intimate partner violence in young adolescents? Select all that apply. A: Sexually acting out B: Attempting suicide C: Pattern of substance abuse D: Fear of certain people or places E: Preoccupation with others or one's own genitals

A

What important intervention should be included in the nursing care provided immediately after a sexual assault? A: Obtaining the assault history from the client B: Informing the police before the client is examined C: Having the client void a clean-catch urine specimen D: Testing the client's urine for seminal alkaline phosphatase

C, D, E

While assessing an older adult during a regular health checkup, a nurse finds signs of elder abuse. Which physical findings would further confirm the nurse's suspicion? Select all that apply. A: Presence of hyoid bone damage B: Presence of cognitive impairment C: Presence of burns from cigarettes D: Presence of foreign bodies in the rectum E: Presence of unexplained bruises on the wrist(s)

By identifying behaviors commonly exhibited by the client who has a diagnosis of schizophrenia, the nurse can anticipate: Disorientation, forgetfulness, and anxiety Grandiosity, arrogance, and distractibility Withdrawal, regressed behavior, and lack of social skills Slumped posture, pessimistic outlook, and flight of ideas

Withdrawal, regressed behavior, and lack of social skills Withdrawal, regressed behavior, and lack of social skills are classic behaviors exhibited by clients with a diagnosis of schizophrenia. Disorientation, forgetfulness, and anxiety are more commonly associated with dementia. Grandiosity, arrogance, and distractibility are more commonly associated with bipolar disorder, manic phase. Slumped posture, pessimistic outlook, and flight of ideas are more commonly associated with depression.

A client receiving the medication buspirone hydrochloride (Buspar) is admitted to the hospital with the diagnosis of possible hepatitis. The nurse identifies that the client's sclerae look yellow. What should be the nurse's initial action?

Withhold the medication The medication should be stopped immediately because jaundice indicates possible liver damage, which prolongs elimination of the drug and may result in toxic accumulation. Milk does not change the effect of the drug. The drug must be stopped, not reduced. The drug is available only in an oral form; in addition, the route of administration will not influence the occurrence of toxic accumulation.

The nurse is interviewing a client admitted for uncontrolled diabetes after binging on alcohol for the past 2 weeks. The client states, "I am worried about how I am going to pay my bills for my family while I am hospitalized." Which statement by the nurse would best elicit information from the client? -"you are worried about paying your bills?" -Don't worry; your bills will get paid eventually" -"When was the last time you were admitted for hyperglycemia?" -"You really shouldn't be drinking alcohol because of your Dx of diabetes"

You are worried about paying your bills

A client with schizophrenia is actively psychotic, and a new medication regimen is prescribed. A student nurse asks the nurse, "Which of the medications will be the most helpful against the psychotic signs and symptoms?" What response should the nurse give?

Ziprasidone (Geodon) Ziprasidone (Geodon) is a neuroleptic, which will reduce psychosis by affecting the action of both dopamine and serotonin. Citalopram (Celexa) is a selective serotonin reuptake inhibitor antidepressant. Benztropine (Cogentin) is an anticholinergic. Acetaminophen with hydrocodone (Lortab) is an analgesic/opioid.

ADHD is a syndrome—

a cluster of behaviors related to inattention and impulsive actions.

Wernicke-Korsakoff syndrome (WK syndrome)

a degenerative brain disorder due to a thiamine deficiency (Vitamin B1)

In home mental health nurse visits a female client to assess her ability to care for herself at home after discharge from an inpatient setting. Which component of the case management system does this demonstrate?

a. Psychosocial rehabilitation

korsakaffs

amnesic disorder

Tyramine (TIE-ruh-meen)

an amino acid that helps regulate blood pressure.helps relieve depression.

Mydriasis and photophobia are side effects of ?

anticholinergic

The nurse asks the client a series of questions upon entry into a mental health care system. This action is an example of which phase of the nursing process?

assessment

is a disorder that involves an individual's ability to communicate, interact with others, use the imagination, and display appropriate behavior.

autism

The nurse or caregiver needs to have _______ when discussing sexual issues

awareness

breathe holding spells

child is upset,in someway, begin to cry but no sound emerges He stops, turn blue, goes limp become unresponsive

mental retardation

children who function significantly below the average intellectual level of their age group and are limited in abilities

A young client who has just lost her first comes to the mental health clinic very upset and says, "I just start crying without any reason and without any warning." How should the nurse respond initially? -"Do you know what makes you cry?" -"Most of us need to cry from time to time." -"Crying unexpectedly can be very upsetting." -"Are you having any other problems at this time?"

crying unexpectedly care be very upsetting

__________ is a pattern that involves repeated mood swings, alternating between hypomania and depressive symptoms

cyclothymic

What is extremely important when documenting subjective information (pg.94)?

document the subjective information as descriptively and accurately as possible. Do not include interpretive statements (judgements) Document client reactions and behaviors in exact terms. (Ex: the client was pacing about the room while slamming his fist into the wall and swearing)

Many people with personality disorders who suffer from substance abuse or other mental health problems. They are categorized as

dual diagnoses

When is subjective data collected with a mentally ill patient (pg.94)?

during the initial health history interview and during every interaction with the client

pica

eating nonfood items for more than a month

mental health problems in childhood

environmental, problem with parents, emotional,behavioral,eating or elimination problem,developmental problems,pervasive developmental problems

During the mental status examination, the nurse observes that the client rapidly changes from one idea to another related thought. Which disordered thinking process is the client displaying?

flight of ideas

Which therapeutic communication technique is most useful for the nurse to use when the client begins to repeat previously mentioned issues in the same therapeutic conversation? -focusing -clarifying -paraphrasing -summarizing

focusing

A female who is physically abused by her husband arrives in the emergency room following a serious battering incident. During this phase of the domestic abuse cycle, the nurse expects the client to:

have experienced the battering episode a few days prior to the incident

A client with depression is to be given fluoxetine (Prozac). What precaution should the nurse consider when initiating treatment with this drug?

he blood level may not be sufficient to cause noticeable improvement for 2 to 4 weeks. Fluoxetine (Prozac) does not produce an immediate effect; nursing measures must be continued to reduce the risk for suicide. Consuming milk and crackers to help prevent hyperacidity and discomfort is not necessary. Avoiding cheese, pickled herring, and wine is a precaution taken with the monoamine oxidase inhibitors. Weekly blood level checks are not necessary with Fluoxetine.

School avoidance (anxiety-based school refusal)

is a behavioral pattern in which the child refuses to attend school.

Wernicke encephalopathy

is a brain disorder that may occur in clients with chronic alcoholism. It causes damage to the temporal lobes of the brain. Delirium tremens, fetal alcohol syndrome, and alcohol withdrawal syndrome are also associated with alcoholism but may not cause the client brain damage.

A colicky infant

is defined as a healthy, well-fed child who cries for longer than 3 hours every day for more than 3 weeks.

A nurse is preparing to give a 3-year-old toddler an intramuscular injection when the child starts to cry. What is the most therapeutic approach for the nurse to take? - "It'll be OK because it won't hurt a bit." -"It's all right to cry, it will be quick." -"If you act like a grownup, we can be done really quickly." -"I brought another nurse to hold you while I give you the medicine."

it's all right to cry, it will be quick

anergia

lack of energy

which is more severe pdd or mdd?

mdd

Which health insurance government funded and available to persons 65 and older?

medicare

Children at greater risk for committing suicide if they have _______ problems

mental health

akathisia

muscle quivering, restlessness and inability to sit still, sometimesa side effect of antipsychotic or antidepressant medication.a side effect of haloperidol (Haldol), develops early in therapy and is characterized by restlessness and agitation. Torticollis is characterized by a stiff neck (wry neck).

A serious and potentially fatal side effect with unstable vital signs, fever, confusion, muscle rigidity, tremors, incontinence. (ESPE of antipsychotics)

neuroleptic malignant syndrome

A disorder that interrupts normal sleep patterns and is characterized by repeated, brief jerks of the arms and legs that occur every 20 to 60 seconds during the beginning of sleep is called: A) Insomnia B) Narcolepsy C) Hypersomnia D) Nocturnal myoclonus

nocturnal myoclonus

dysthymia

persistant depressive disorder, chronic depression

Eric burne theorized that an individual's three ego states of parent, child, and adult make up ones ____________.

personality

When should the nurse begin preparations for the termination phase of a therapeutic relationship?

prior to the last meeting

dystonia

produce rigidity in the muscles that control gait, posture, and eye movements.

The first priority nursing care measure for adolescent client with a diagnosis of schizophrenia is to

provide care for basic physical and safety needs

What is the purpose of a informal interview (pg.94)?

usually occur casually and provide great opportunities to learn more about clients and their families

A child is admitted to the pediatric intensive care unit after an accident at school. The mother, at the child's bedside, is visibly upset. What is the most therapeutic statement the nurse can make? -you must be full of emotions right now -let me give you a referral for social services -your child will get excellent care at this hospital -it's a shame that your child has become ill so suddenly

you must be full of emotions right now

A client is dying. Hesitatingly, his wife says to the nurse, "I'd like to tell him how much I love him, but I don't want to upset him." Which is the best response by the nurse? -You must keep up a strong appearance for him -i think he'd have difficulty dealing with that now -don't you think he knows that without you telling him? -you should share your feelings with him while you can

you should share your feelings with him while you can

When talking with a client who has been receiving paroxetine (Paxil), the nurse determines that more clarification is needed when the client says:

"I've been on the medication for 8 days now, and I don't feel any better."

A 2½-year-old child is admitted for treatment of injuries supposedly sustained in a fall down a flight of stairs. Child abuse is suspected. What statements might the nurse expect from a parent who engages in child abuse? (Select all that apply.)

"Every time I turn around the kid is falling over something." "I can't understand it. He didn't have a problem using the stairs without my help before this."

Which therapeutic communication technique involves using a coping strategy to help the nurse and client adjust to stress? -sharing hope -sharing humor -sharing empathy -sharing observations

sharing humor

Certain questions are applicable in determining nursing negligence. (Select all that apply.)

"Was reasonable care provided?" "Was there a breach of nursing duty?" "Was there an act of omission that resulted in harm? "Except for the nurse's action, would the injury have occurred?"

During the mental status assessment, the nurse hands the client a piece of paper that reads Please raise your left hand. If the client follows the command, the nurse has just assessed which ability of the client?

reading

development

refers to the increased ability in skills or functions

What characteristic is most essential for the nurse caring for a client undergoing mental health care? 1.Empathy 2.Sympathy 3.Organization 4.Authoritarianism

1.Empathy

A female client calls the clinic for advice after forgetting to take her morning dose of twice-daily lithium 5 hours ago. Which instructions should the nurse give the client?

Eliminate the dose missed, and take the second dose at the normal time.

A client is scheduled for a 6-week electroconvulsive therapy (ECT) treatment program. What intervention is important during the course of treatment?

Elimination of benzodiazepines for nighttime sedation

Emotional Problems

Emotional problems occur in children when they cannot cope successfully with their situations.

Sildenafil (Viagra) is prescribed for a man with erectile dysfunction. A nurse teaches the client about common side effects of this drug. (Select all that apply.)

Flushing Headache Dyspepsia

Sildenafil (Viagra) is prescribed for a man with erectile dysfunction. A nurse teaches the client about common side effects of this drug. (Select all that apply.)

Flushing Headache Dyspepsia

A client is admitted to the psychiatric hospital with a diagnosis of obsessive-compulsive disorder. The client's anxiety level is approaching a panic level, and the client's ritual is interfering with work and daily living. Which selective serotonin reuptake inhibitor (SSRI) should the nurse anticipate that the health care provider may prescribe?

Fluvoxamine (Luvox)

A nurse is caring for a client who uses ritualistic behavior. What common antiobsessional medication does the nurse anticipate will be prescribed?

Fluvoxamine (Luvox)

A frail, depressed client who frequently paces the halls becomes physically tired from the activity. What action should the nurse take to help reduce this activity?

Have the client perform simple, repetitive tasks

A college student is brought to the mental health clinic by his parents. The diagnosis is borderline personality disorder. Which factors in the client's history support this diagnosis? (Select all that apply.)

Impulsiveness Lability of mood Self-destructive behavior

Personality disorders in the adolescent are characterized by:

Impulsivity

A nurse is interviewing an 8-year-old girl who has been admitted to the pediatric unit. Which statement by the child needs to be explored?

"Those boys are so cute. I hope their room's next to mine!" An 8-year-old child should be more concerned with same-gender relationships. A child who demonstrates a strong attraction to opposite-gender relations should be questioned further to explore the possibility of sexual abuse. A statement such as "Wow! This place has bright colors" is not unusual because 8-year-old children are usually attracted to colorful environments. A statement such as "Is my mother allowed to visit me tonight?" or "I'm scared about being here. Can you stay with me awhile?" is not unusual because 8-year-old children will want the support of a trusted person when experiencing stress.

A client at 38 weeks' gestation is scheduled for a nonstress test. The woman asks the nurse, "Do you think this test is necessary?" What is the most appropriate response by the nurse?

"You seem to have doubts about this test."

A client is receiving doxepin (Sinequan). For which most dangerous side effect of tricyclic antidepressants should a nurse monitor the client? 1. Mydriasis 2. Dry mouth 3. Constipation 4. Urine retention

1. Mydriasis

A client is admitted for treatment of obsessive-compulsive disorder that is interfering with activities of daily living. Which medication should the nurse anticipate the health care provider will prescribe?

*Clomipramine (Anafranil) potentiates the effects of serotonin (antiobsessional effect) and norepinephrine in the central nervous system; it diminishes obsessive-compulsive behaviors. Benztropine (Cogentin) is an antiparkinsonian agent, not an antianxiety agent. Amantadine (Symmetrel) is an antiparkinsonian agent, not an antianxiety agent. Diphenhydramine (Benadryl) is an antihistamine, not an antianxiety agent.

Which tool is used to standardize and measure nursing treatments?

-nursing intervention classification

A psychiatric unit uses a behavioral approach to determine a client's level of privileges. Which factor should a nurse use to determine whether an increase in privileges is warranted? 1.)Stating that the depression is lifting 2.)Performing hygiene activities independently 3.)Showing improvement in short-term memory 4.)Verbalizing a desire to change the response to stress

-performing hygiene activities independently

A client with systemic lupus erythematosus is taking prednisone. The nurse anticipates that the steroid may cause hypokalemia. Taking into consideration food preferences, the nurse encourages the client to eat: Correct1 Broccoli 2 Oatmeal 3 Fried rice 4 Cooked carrots

1 Broccoli Potassium is plentiful in green leafy vegetables; broccoli provides 207 mg of potassium per half cup. Oatmeal provides 73 mg of potassium per half cup. Rice provides 29 mg of potassium per half cup. Cooked fresh carrots provide 172 mg of potassium per half cup; canned carrots provide only 93 mg of potassium per half cup.

A nurse is caring for a client with an antisocial personality disorder. What consistent approach should the nurse use with this client? 1 Warm and firm without being punitive 2 Indifferent and detached but nonjudgmental 3 Conditionally acquiescent to client demands 4 Clearly communicative of personal disapproval

1 Warm and firm without being punitive The client needs positive relationships with other adults, but clear, consistent limits must be presented to minimize attempts at manipulation. Being indifferent and detached but nonjudgmental is not a therapeutic approach. Being indifferent and detached gives the impression that the nurse does not care. Conditional acquiescence to client demands is not a therapeutic approach because clear, consistent limits are necessary to prevent manipulation. Being clearly communicative of personal disapproval is a judgmental attitude that should be avoided.

A registered nurse is educating a nursing student about the utilitarian system of ethics. What information should the nurse provide? Select all that apply. 1 "The value of something is decided by its usefulness." 2 "The main emphasis is on the outcome or consequence of the action." 3 "The system examines a situation for the presence of essential right or wrong." 4 "The greatest good for the greatest number of people determines the right actions 5 "The actions can be determined whether right or wrong based on their 'right-making characteristics.'"

1,2,4

Certain questions are applicable in determining nursing negligence. (Select all that apply.) 1. "Was reasonable care provided?" 2. "Was there a breach of nursing duty?" 3. "Was there an act of omission that resulted in harm? 4. "Except for the nurse's action, would the injury have occurred?" 5. "Did the nurse fully understand the actions would result in harm?"

1. "Was reasonable care provided?" 2. "Was there a breach of nursing duty?" 3. "Was there an act of omission that resulted in harm? 4. "Except for the nurse's action, would the injury have occurred?"

What characteristic is most essential for the nurse caring for a client undergoing mental health care? 1. Empathy 2. Sympathy 3. Organization 4. Authoritarianism

1. Empathy **Empathy—understanding and to some extent sharing the emotions of another—encourages the expression of feelings. Empathy is an essential tool in caring for emotionally ill clients.

What is the most appropriate intervention for the nurse to implement after finding a disturbed client in bed in the fetal position? 1. Sitting down in a chair by the client and saying, "I'm here to spend time with you." 2. Touching the client gently on the shoulder and saying, "I'm going to sit with you for a while." 3. Going to the client and saying, "I'll be waiting for you in the community room, so please get up and join me." 4. Leaving the client alone because the behavior demonstrates that the client has regressed too far to benefit from talking with the nurse

1. Sitting down in a chair by the client and saying, "I'm here to spend time with you."

An 8-year-old child who is experiencing a sickle cell pain episode is admitted to the child health unit. What is the most appropriate nursing care during this acute period? 1 Limiting fluids until the crisis ends Correct2 Administering prescribed analgesics 3 Applying cold compresses to painful joints 4 Performing range-of-motion exercises of affected joints

2 Administering prescribed analgesics The priority is pain management; severe pain requires analgesics. Increased hydration is necessary to promote hemodilution, improve circulation, and prevent more sickling. Cold will constrict blood vessels, further depleting oxygenation to affected parts; warmth is preferable. There is too much swelling and pain in the joints during a crisis for the implementation of range-of-motion exercises.

A 23-year-old woman is admitted to a psychiatric unit after several episodes of uncontrolled rage at her parents' home, and borderline personality disorder is diagnosed. While watching a television newscast describing an incident of violence in the home, the client says, "People like that need to be put away before they kill someone." The nurse concludes that the client is using: 1.Denial 2.Projection 3.Introjection 4.Sublimation

2.Projection

Many clients who call a crisis hotline are extremely anxious. The nurse answering the hotline phone considers that the characteristic distinguishing posttraumatic stress disorders from other anxiety disorders is: 1 Lack of interest in family and others 2 Reexperiencing the trauma in dreams and flashbacks 3 Avoidance of situations and activities that resemble the stress 4 Depression and a blunted affect when discussing the traumatic situation

2 Reexperiencing the trauma in dreams and flashbacks Experiencing the actual trauma in dreams or flashbacks is the major symptom that distinguishes posttraumatic stress disorders from other anxiety disorders. Lack of interest in family and others is usually not associated with anxiety disorders. Avoidance of situations and activities that resemble the stress is more common with phobic disorders. Although depression may be generated by discussion of the traumatic situation, the affect is usually exaggerated, not blunted.

The nurse is cautiously avoiding the temptation to take unused or wasted doses of narcotic medications when providing patient care. What percentage of nurses in the USA are chemically impaired? 1. 0%-5% 2. 6%-15% 3.25%-35% 4. 49%-50%

2. 6%-15%

Before a treatment requiring informed consent can be performed, what information must the client be given? (Select all that apply.) 1. The cost of the treatment 2. Alternative treatment options 3. The risks and benefits of the treatment 4.The risks involved in refusing the treatment 5. The nature of the problem requiring the treatment

2. Alternative treatment options 3. The risks and benefits of the treatment 4.The risks involved in refusing the treatment 5. The nature of the problem requiring the treatment

The way individuals cope with an unexpected hospitalization depends on many factors. However, the one that is most significant is: 1. Cognitive age 2. Basic personality 3. Financial resources 4. General physical health

2. Basic personality

A hospitalized 7-year-old boy wakes up crying because he has wet his bed. It is most appropriate for the nurse to: 1. Allow the child to change his bed and pajamas 2. Change the child's bed while he changes his pajamas 3. Take the child to the bathroom and change his pajamas 4. Remind the child to call the nurse next time to avoid the need to change his pajamas

2. Change the child's bed while he changes his pajamas **Changing the child's bed while the child changes his pajamas will not call attention to the accident and will minimize the child's embarrassment.

A recently married 22-year-old woman is brought to the trauma center by the police. She has been robbed, beaten, and sexually assaulted. The client, although anxious and tearful, appears to be in control. The health care provider prescribes alprazolam (Xanax) 0.25 mg for agitation. The nurse should administer this medication when the: 1. Client's crying increases 2. Client requests something to calm her 3. Nurse determines a need to reduce her anxiety 4. Health care provider is getting ready to perform a vaginal examination

2. Client requests something to calm her **Because a sexual assault is a threat to the sense of control over one's life, some control should be given back to the client as soon as possible.

A client has been prescribed chlorpromazine (Thorazine) for the management of positive symptoms of schizophrenia. When the client reports difficulty sustaining an erection, the nurse: 1. Reassures him this side effect will resolve in a few weeks 2. Consults with his provider regarding alternative medication therapies 3. Explains that all conventional antipsychotic medications cause impotence 4. Provides additional medication education to explain the medication's side effects in detail

2. Consults with his provider regarding alternative medication therapies

The parents of a toddler with recently diagnosed moderate cognitive impairment discuss their child's possibility of future independent function. What should the nurse conclude? 1. They accept the child's diagnosis. 2. Denial is being used as a defense. 3. They want to explore their child's limitations. 4. Intellectualization helps them put the diagnosis into perspective.

2. Denial is being used as a defense.

A client is admitted with a conversion disorder. What is the primary nursing intervention? 1. Talking about the physical problems 2. Exploring ways to verbalize feelings 3. Explaining how stress caused the physical symptoms 4. Focusing on the client's concerns regarding the symptoms

2. Exploring ways to verbalize feelings

A client is admitted to the psychiatric hospital with a diagnosis of obsessive-compulsive disorder. The client's anxiety level is approaching a panic level, and the client's ritual is interfering with work and daily living. Which selective serotonin reuptake inhibitor (SSRI) should the nurse anticipate that the health care provider may prescribe? 1. Haloperidol (Haldol) 2. Fluvoxamine (Luvox) 3. Imipramine (Tofranil) 4 .Benztropine (Cogentin)

2. Fluvoxamine (Luvox) **Fluvoxamine (Luvox) inhibits central nervous system neuron uptake of serotonin but not of norepinephrine.

An injured child is brought to the emergency department by the parents. While interviewing the parents, the nurse begins to suspect child abuse. Which parental behaviors might support this conclusion? (Select all that apply.) 1. Demonstrating concern for the injured child 2. Focusing on the child's role in sustaining the injury 3. Changing the story of how the child sustained the injury 4. Asking questions about the injury and the child's prognosis 5. Giving an explanation of how the injury occurred that is not consistent with the injury

2. Focusing on the child's role in sustaining the injury 3. Changing the story of how the child sustained the injury 5. Giving an explanation of how the injury occurred that is not consistent with the injury

An older retired client is visiting the clinic for a regularly scheduled checkup. The client tells the nurse about the great life he has lived and the activities that he enjoys at the senior center. According to Erikson, what developmental conflict has been resolved by this client? 1. Trust versus mistrust 2. Integrity versus despair 3. Generativity versus self-absorption 4. Autonomy versus shame and doubt

2. Integrity versus despair

When a newly admitted client with paranoid ideation tells the nurse about people coming through the doors to commit murder, the nurse should: 1. Ignore the client's stories 2. Listen to what the client is saying 3. Explain that no one can get through the door 4. Ask for an explanation of where the information was obtained

2. Listen to what the client is saying

The nurse should first discuss terminating the nurse-client relationship with a client during the: 1. Working phase, when the client initiates it 2. Orientation phase, when a contract is established 3. Working phase, when the client shows some progress 4.Termination phase, when discharge plans are being made

2. Orientation phase, when a contract is established

A nurse notes that haloperidol (Haldol) is most effective for clients who exhibit behavior that is: 1. Depressed 2. Overactive 3. Withdrawn 4. Manipulative

2. Overactive

A nurse teaches dietary guidelines to a client who will be receiving tranylcypromine sulfate (Parnate), a monoamine oxidase inhibitor (MAOI). The client compiles a list of foods to avoid. Which foods included on the list indicate that the teaching has been effective? (Select all that apply.) 1. French fries 2. Pepperoni pizza 3. Bologna sandwich 4 . Hamburger on a bun 5 . Hash brown potatoes

2. Pepperoni pizza 3. Bologna sandwich

A 23-year-old woman is admitted to a psychiatric unit after several episodes of uncontrolled rage at her parents' home, and borderline personality disorder is diagnosed. While watching a television newscast describing an incident of violence in the home, the client says, "People like that need to be put away before they kill someone." The nurse concludes that the client is using: 1. Denial 2. Projection 3. Introjection 4. Sublimation

2. Projection **Projection is the process of attributing one's thoughts about one's self to others.

A client sits huddled in a chair and leaves it only to assume the fetal position in a corner. The nurse, observing this, identifies the behavior as: 1. Reactive 2. Regressive 3. Dissociative 4. Hallucinatory

2. Regressive **Curling up in a corner reflects the early fetal position; the individual curls up for both protection and security

An older client whose family has been visiting on the psychiatric unit is visibly angry and says to the nurse, "My daughter-in-law says they can't take me home until the doctor lets me go. She doesn't understand how important this is to me; she's not from our culture." What should the nurse do? 1. Ignore the statement for the present 2. Say, "You feel she doesn't want you at home. 3. Reflect on the client's feelings about the cultural differences 4. Respond, "The doctor is the one who makes decisions about discharge."

2. Say, "You feel she doesn't want you at home.

After several interactions with a client, the nurse at the mental health clinic identifies a pattern of withdrawal and nonparticipation in situations requiring communication with others. In which area should the nurse expect the client to have difficulty? 1. Personal identity 2. Social interaction 3. Sensory perception 4. Verbal communication

2. Social interaction

The emergency department nurse is conducting an interview and assisting with the physical examination of a female sexual assault victim. What is most important for the nurse to document on this client's record? 1. Observations about the client's reaction to male staff members 2. Statements by the client about the sexual assault and the rapist 3. Information about the client's previous knowledge of the rapist 4. Summary statement about the client's description of the assault and the rapist

2. Statements by the client about the sexual assault and the rapist

A health care provider prescribes haloperidol (Haldol) for a client. What should the nurse teach the client to avoid while taking this medication? 1. Driving at night 2. Staying in the sun 3. Ingesting aged cheeses 4. Taking medications containing aspirin

2. Staying in the sun **Haloperidol (Haldol) causes photosensitivity. Severe sunburn may occur on exposure to the sun.

An executive, busy at work, receives a phone call from a friend relating bad news. The woman makes a conscious effort to put this information out of her mind and continues to work at the task at hand. The next day she remembers that her friend telephoned her but is unable to recall the message. Which defense mechanism does this behavior represent? 1. Regression 2. Suppression 3. Passive aggression 4. Reaction formation

2. Suppression **Suppression is the voluntary exclusion from awareness of anxiety-producing feelings, ideas, and situations.

What is an initial client objective in relation to anger management? 1. Expressing remorse over aggressive actions 2. Taking responsibility for the hostile behavior 3. Developing alternative methods to release feelings 4. Teaching others how to avoid triggering the angry behavior

2. Taking responsibility for the hostile behavior

The nurse is caring for a patient who is verbalizing concerns related to a difficult relationship.Which response by the nurse will block communication and should be avoided? SELECT ALL THAT APPLY 1. using silence 2. asking why 3. changing the subject 4. agreeing or disagreeing 5. verbalizing the implied

2. asking why 3. changing the subject 4. agreeing or disagreeing

A patient who has been diagnosed with a mental illness tells the nurse about plans to find a voodoo doctor. How should the nurse respond? 1. you know voodoo doctors can't really help you. Don't waste your money 2. be sure to mention your plan to your psychiatrist. Its important to follow up with that treatment plan also 3. research has shown that voodoo can effectively treat any mental health disorders. that decision is up to you 4. i do not think using voodoo is safe. I would recommend you think about it carefully before contacting a voodoo doctor

2. be sure to mention your plan to your psychiatrist. Its important to follow up with that treatment plan also

The nurse is caring for a client in preterm labor who reports that she fell down the stairs. Bruises are apparent on the left part of the client's lower abdomen, the back of each shoulder, and on both wrists. After instituting electronic fetal monitoring, starting tocolytic therapy, and examining the monitor strips, what action should the nurse take next? 1 Ambulating the client to promote circulation 2 Inserting two small-bore intravenous catheters 3 Determining whether the client feels safe at home 4 Ensuring that the client has her glasses to ambulate

3

The rate of attempted suicide in adolescents is _____ times higher with females than with males.

3

A 25-year-old woman is seeking outpatient counseling after thinking about suicide. The nurse realizes that there some factors place individuals at a higher risk for suicide. Which of these factors increases the risk for suicide? (Select all that apply.)

Impulsivity Panic attacks Unemployment Substance abuse

Often seen in children with the following:

Mental retardation, Developmental disorders, Vitamin deficiencies , Mineral deficiencies

A nurse is considering Erikson's stages of psychosocial development while caring for a client. Which behavior is consistent with a problem involving trust versus mistrust? 1 Woman in an abusive relationship who refuses to leave the abuser 2 Man with paranoid schizophrenia who demands placement in a private room 3 Woman whose parents were chronic alcoholics and who has problems making friends 4 Man with borderline personality disorder who has been caught stealing from other clients

3 Woman whose parents were chronic alcoholics and who has problems making friends

A nurse notes that a famous client has received an incorrect dose of medication due to the malfunction of the intravenous (IV) device, but does not inform the primary healthcare provider. Instead the nurse tells a colleague that the medication could not be given due to the client's inappropriate behavior. The nurse then updates media personnel about the client's health status. What legal charges may be brought up against the nurse? Select all that apply. 1 Libel 2 Assault 3 Slander 4 Malpractice 5 Invasion of privacy

3, 4, 5

Which statements made by a terminally ill client address areas of concern regarding end-of-life nursing care? Select all that apply. 1 "I want my children to carry in my casket." 2 "I've prepaid all my funeral expenses so I won't be a burden." 3 "My living will states that I want no heroic measures to prolong my life." 4 "Pain is a concern of mine, so I've discussed that thoroughly with my doctor." 5 "I've made arrangements that will allow me to spend my final days in my own home."

3,4,5

The parents of a child with attention deficit-hyperactivity disorder ask the nurse about using medication. What is the most frequently prescribed medication for this disorder?

Methylphenidate (Ritalin)

In response to a question posed during a group meeting, the nurse explains that the superego is that part of the self that says: 1. "I like what I want." 2. "I want what I want." 3. "I shouldn't want that." 4. "I can wait for what I want."

3. "I shouldn't want that." **Conscience and a sense of right and wrong are expressed in the superego, which acts to counterbalance the id's desire for immediate gratification.

A family member brings a relative to the local community hospital because the relative "has been acting strange." Which statements meet involuntary hospitalization criteria? (Select all that apply.) 1. "I cry all the time; I'm just so sad." 2 . "Since I retired I've been so depressed." 3. "I'd like to end it all with sleeping pills." 4 . "The voices say I should kill all prostitutes." 5 . "My boss makes me so angry—he's always picking on me."

3. "I'd like to end it all with sleeping pills." 4 . "The voices say I should kill all prostitutes." **Rationale: The statement about ending it all is a suicide threat; it is a direct expression of intent without action. Likewise, the threat to harm others must be heeded.

A nurse leads an assertiveness training program for a group of clients. Which statement by a client indicates that the treatment has been effective? 1. "I know that I should put the needs of others before mine." 2. "I won't stand for it, so I told my boss he's a jerk and to get off my back." 3. "It annoys me when people call me 'sweetie,' so I told him not to do it anymore." 4. "It's easier for me to agree up front and then do just enough so that no one notices."

3. "It annoys me when people call me 'sweetie,' so I told him not to do it anymore."

What is the most therapeutic nursing intervention to help a late-middle-aged individual cope with the emotional aspects of aging? 1. Focusing on the individual's past experiences 2. Having the individual attend lectures on aging 3. Assisting the individual with plans for the future 4. Encouraging the individual to focus on his or her career

3. Assisting the individual with plans for the future **Helping an individual maintain an interest in the future is therapeutic. It is forward looking and fosters a positive attitude.

As the nurse is discussing psychiatric care with an older adult client, the client says, "When I was growing up I was taught to accept my lot in life and not complain. I'm proud of the fact that despite my issues I can still function independently. I don't want to be just put away." The nurse understands that the factors that influence the client's mental health are examples of: 1. Setting of care 2. Anxiety disorder 3. Attitudes and beliefs 4. Cultural and ethnic disparities

3. Attitudes and beliefs

What developmental task should the nurse consider when caring for toddlers? 1. Trust 2. Industry 3. Autonomy 4. Identification

3. Autonomy

A parent whose daughter is killed in a school bus accident tearfully tells the nurse, "My daughter was just getting over the chickenpox and didn't want to go to school, but I insisted that she go. It's my fault that she's dead." How should the nurse anticipate that perceiving a death as preventable will likely influence the grieving process? 1. The loss may be easier to understand and accept. 2. The mourner may experience pathological grief. 3. Bereavement may be of greater intensity and duration. 4. The grieving process may progress to a psychiatric illness

3. Bereavement may be of greater intensity and duration.

A client is admitted for treatment of obsessive-compulsive disorder that is interfering with activities of daily living. Which medication should the nurse anticipate the health care provider will prescribe? 1. Benztropine (Cogentin) 2. Amantadine (Symmetrel) 3. Clomipramine (Anafranil) 4. Diphenhydramine (Benadryl)

3. Clomipramine (Anafranil)

What is the most difficult initial task in the development of a nurse-client relationship? 1. Remaining therapeutic and professional 2. Being able to understand and accept a client's behavior 3. Developing an awareness of self and the professional role in the relationship 4. Accepting responsibility for identifying and evaluating the real needs of a client

3. Developing an awareness of self and the professional role in the relationship **The nurse's major tool in mental health nursing is the therapeutic use of self. Mental health nurses must learn to identify their own feelings and understand how they affect the situation.

A client in the mental health clinic who has been seeing a therapist for more than 6 months begins to talk and act like a therapist who is analyzing coworkers. What defense mechanism does the nurse identify? 1. Undoing 2. Projection 3. Introjection 4. Intellectualization

3. Introjection **Introjection is treating something outside the self as if it is actually inside the self; it is unconsciously incorporating the wishes, values, and attitudes of another as if they were one's own.

A client with schizophrenia who has type II (negative) symptoms is prescribed risperidone (Risperdal). Which outcomes indicate that the medication has minimized these symptoms? (Select all that apply.) 1.There is less agitation. 2.There are fewer delusions. 3. More interest is shown in unit activities. 4. The client reports that the hallucinations have stopped. 5. The client performs activities of daily living independently.

3. More interest is shown in unit activities. 5. The client performs activities of daily living independently.

Methylphenidate (Ritalin) is prescribed to treat a 7-year-old child's attention deficit-hyperactivity disorder (ADHD). The nurse understands that methylphenidate is used in the treatment of this disorder in children for its: 1.Diuretic effect 2.Synergistic effect 3.Paradoxical effect 4.Hypotensive effect

3.Paradoxical effect

According to the theorist Erik Erikson, an individual strives to actualize his identity, is most productive, and demonstrates guidance of and concern for others with a core task of caring during which stage of psychosocial development?

Middle adulthood (25-65)

Which tool is used to standardize and measure nursing treatments? 1. Nursing Outcomes Classification (NOC) 2. NANDA-I-Approved Nursing Diagnoses 3. Nursing Interventions Classification (NIC) 4. Diagnostic and Statistical Manual of Mental Disorders (DSM-5)

3. Nursing Interventions Classification (NIC) **Nursing Interventions Classification (NIC) is a tool that helps to define nursing interventions, as well as helps to standardize and measure the nursing care provided.

A group of clients from a psychiatric unit, accompanied by staff members, are going to a professional baseball game. The purpose of visits into the community under the supervision of staff members is: 1. Helping clients adjust to stressors in the community 2. Helping clients return to reality under controlled conditions 3. Observing the clients' abilities to cope with a more complex society 4. Broadening the clients' experiences by providing exposure to cultural activities

3. Observing the clients' abilities to cope with a more complex society

What should the nurse do to develop a trusting relationship with a disturbed child who acts out? 1. Ask the child's feelings about the parents 2. Implement one-on-one interactions every half hour 3. Offer support and encourage safety during play activities 4. Begin setting limits and explain the rules that must be followed

3. Offer support and encourage safety during play activities

What should the nurse do when a client with the diagnosis of schizophrenia talks about being controlled by others? 1. Express disbelief about the client's delusion 2. Divert the client's attention to unit activities 3. React to the feeling tone of the client's delusion 4. Respond to the verbal content of the client's delusion

3. React to the feeling tone of the client's delusion

A nurse is caring for a client who is experiencing a crisis. Which nervous system is primarily responsible for the clinical manifestations that the nurse is likely to identify? 1. Central nervous system 2. Peripheral nervous system 3. Sympathetic nervous system 4. Parasympathetic nervous system

3. Sympathetic nervous system **The sympathetic nervous system reacts to stress by releasing epinephrine, which prepares the body to fight or flee by increasing the heart rate, constricting peripheral vessels, and increasing oxygen supply to muscles.

A patient has been prescribed fluoxetine (Prozac) to treat depression. What should be included in the nurse's teaching about the drug? SELECT ALL THAT APPLY 1. you need to take this drug only once a week 2, take the prescribed dose in the early morinings 3. a decreased interest in sexual activity may occur with this medication 4. you should not consume red wine, aged cheese, or other tyramine-rich foods 5. do not expect immediate results; it usually takes 6 to 8 weeks for therapeutic effects to be felt 6. you may experience some nausea , vomiting, and anorexia, but these side effects will subside in time

3. a decreased interest in sexual activity may occur with this medication 6. you may experience some nausea , vomiting, and anorexia, but these side effects will subside in time

The nurse is assisting with teaching a patient who has been started on fluphenazine (Prolixin). About which side effect should the nurse focus this teaching? 1. weight loss 2. hypoglycemia 3. photo-sensitivity 4. elevated blood pressure

3. photo-sensitivity

The nurse is assisting with medication teaching for a patient who is prescribed lithium carbonate (Eskalith) for bipolar disorder. Which instructions by the nurse is most important? 1. instruct the patient to discontinue other antidepressant agents 2. teach the patient that the lithium will help stabilize mood swings 3. teach the patient side effects to report, such as nausea or weight gain 4. explain to the patient and significant other the importance of regular blood tests

3. teach the patient side effects to report, such as nausea or weight gain

A patient with a suspected mental health disorder having blood drawn to check electrolyte levels ask what blood work has to do with mental illness. Which response by the nurse is correct? 1. electrolyte imbalances are the cause of certain mental health disorders 2. some mental health disorders cause the electrolytes to be out of balance 3. the doctor wants to make sure your symptoms are not caused by a physical problem 4. people with mental illnesses frequently take illicit drugs that can cause electrolyte imbalances

3. the doctor wants to make sure your symptoms are not caused by a physical problem

A nurse is evaluating different situations related to obtaining informed consent. Which situation does the nurse consider to be the most appropriate method of obtaining informed consent? 1 A client consents to a medical procedure after the nurse has explained its risks and benefits in detail. 2 A client provides consent for a surgery after the primary healthcare provider gives the details of the benefits of the surgery. 3 A client consents to a medical procedure after all the details of the procedure have been provided using strictly medical terminology. 4 A client provides consent after the primary healthcare provider has given a detailed explanation of the risks, benefits, and alternatives to the procedure.

4

Which of the following are basic responsibilities of nurses who administer psychotherapeutic drugs? (Select all that apply.)

Monitoring and evaluating the client's response to the medication Continually assessing the client's condition Assisting in the coordination of the client's care Teaching clients about their medications Administering prescribed medications

A carpenter with full-thickness burns of the entire right arm confides, "I'll never be able to use my arm again and I'll be scarred forever" The nurse's best initial response is: 1 "The staff is taking steps to minimize scarring." 2 "Think about how lucky you are. You are alive." 3 "Try not to worry for now. Concentrate on your range-of-motion exercises." 4 "I know you're worried, but it is too early to tell how much scarring will occur."

4 "I know you're worried, but it is too early to tell how much scarring will occur." The response "I know you're worried, but it is too early to tell how much scarring will occur" is a truthful answer and validates the client's feelings. Although true, the response "The staff is taking steps to minimize scarring" shuts off communication and further ventilation of feelings. The response "Think about how lucky you are. You are alive" denies the client's fears. The response "Try not to worry for now. Concentrate on your range-of-motion exercises" denies the client's feelings.

A female nurse has been caring for a depressed 75-year-old woman who reminds her of her grandmother. The nurse spends extra time with her every day and brings her home-baked cookies. The nurse's behavior reflects: 1. Affiliation 2. Displacement 3. Compensation 4. Countertransference

4. Countertransference **With countertransference the professional provider of care exhibits an emotional reaction to a client based on a previous relationship or on unconscious needs or conflicts.

In the process of development the individual strives to maintain, protect, and enhance the integrity of the self. The nurse determines that this is usually accomplished through the use of: 1. Affective reactions 2. Withdrawal patterns 3. Ritualistic behaviors 4. Defense mechanisms

4. Defense mechanisms **When the individual experiences a threat to self-esteem, anxiety increases, and defense mechanisms are used to protect the self

A young adult is being treated in the emergency department for injuries sustained as a result of physical battering by her partner. On learning that there is a history of such abuse, the nurse plans which tertiary nursing interventions? Select all that apply. 1 Contacting family members to provide support 2 Discussing the legal ramifications of not pressing charges 3 Offering to discuss the abusive behavior with the woman's partner 4 Identifying the benefits of attending a support group for battered women 5 Providing her with information regarding local domestic violence shelters

4,5

A nurse concludes that a client is using displacement. Which behavior has the nurse identified? 1. Ignoring unpleasant aspects of reality 2. Resisting any demands made by others 3. Using imaginative activity to escape reality 4. Directing pent-up emotions at someone other than the primary source

4. Directing pent-up emotions at someone other than the primary source

Attention-deficit/hyperactivity disorder (ADHD)

Now the most commonly diagnosed mental health problem in childhood

A client with generalized anxiety disorder says to the nurse, "What can I do to keep myself from overreacting to stress?" What is the best response by the nurse? 1. "Work on problem-solving skills." 2. "Improve your time-management skills." 3. "Ignore situations that you cannot change." 4. "Work on identifying and developing coping strategies."

4. "Work on identifying and developing coping strategies."

A woman who is frequently physically abused tells the nurse in the emergency department that it is her fault that her husband beats her. What is the most therapeutic response by the nurse? 1. "Maybe it was your husband's fault, too." 2. "I can't agree with that—no one should be beaten." 3. "Tell me why you believe that you deserve to be beaten." 4. "You say that it was your fault—help me understand that."

4. "You say that it was your fault—help me understand that."

The CMA is administering an antianxiety medication to a client. Monitoring side effects is the responsibility of which member of the health care team?

Nurse

An older man is widowed suddenly when his wife is killed in an automobile accident. What should the nurse in the emergency department do first to best help the client at this time? 1. Ask a member of the clergy to visit him 2. Have the practitioner prescribe a sedative for him 3. Refer him to a support group that meets near his home 4. Assure him that everything possible was done for his wife

4. Assure him that everything possible was done for his wife

Thirty minutes after administering fluphenazine (Prolixin) to a client, the nurse notes that the client's jaw is rigid, the client is drooling, and her speech is slurred. There are a number of as-needed prescriptions in the client's chart. What should the nurse administer? 1. Diazepam (Valium), 10 mg by mouth 2. Trihexyphenidyl (Artane), 1 mg by mouth 3. Haloperidol (Haldol), 2 mg intramuscularly 4. Benztropine (Cogentin), 2 mg intramuscularly7

4. Benztropine (Cogentin), 2 mg intramuscularly7 **Benztropine (Cogentin) is an anticholinergic, antiparkinsonian drug used to treat drug-induced extrapyramidal symptoms associated with phenothiazine therapy; the intramuscular (IM) route will relieve symptoms more rapidly.

Methylphenidate (Ritalin) is prescribed to treat a 7-year-old child's attention deficit-hyperactivity disorder (ADHD). The nurse understands that methylphenidate is used in the treatment of this disorder in children for its:

Paradoxical effect

Causes:

Parental mental health problems

Individuals with the diagnosis of dissociative disorder often experience chronic low self-esteem. Which nursing intervention most likely would promote the client's sense of self-esteem? A) Have the client list current and past successes and strengths B) Provide external controls and limit setting C) Keep the client in staff view and do observational checks during inpatient stays D) Reorient client as needed to the environment

A. have the client list current and past successes and strengths

Parent-child conflicts

Parents who set limits and enforce them consistently provide stability for children to test their limits in healthy ways.

Causes:

Abuse

A nurse is caring for clients who are undergoing therapy for dependence on alcohol. Which member of the health team has the primary responsibility for their rehabilitation?

According to the philosophy of Alcoholics Anonymous, clients who have problems with alcohol must identify their own need to seek help and therefore become the primary rehabilitators. The nurse can give support but is not the primary rehabilitator. The counselor can give direction but is not the primary rehabilitator. The psychiatrist can give support, but the client is the primary rehabilitator when it comes to coping with alcoholism.

The nurse is caring for a client experiencing a crisis. What role is most important for the nurse to assume when providing therapeutic crisis intervention?

Active participant

S/S of schizophrenia are first seen in

Adolescence

Most common addiction from schizophrenics?

Alcohol

2. 7 y/o inability to make eye contact, inappropriate facial expression, difficulty making friends, showing little emotion

Asperger's

The nurse is working with a teen in whom conduct disorder was diagnosed and his family on developing a plan of care for treatment. What is the nurses first intervention?

Assessing and/or stabilizing the home environment

Following completion of a male clients series of group therapy sessions, the nurse periodically talks with the client to determine whether he has any signs of relapse of his previous problems. This action by the nurse is an example of:

Assessment

What part of the nursing process for a mentally ill patient consist of data being organized into related areas, and problems are identified (pg.93)?

Assessment

The wife of a client who has completed alcohol detoxification relates that she is concerned about her husband's behavior if he starts drinking again. She says, "When the drinking starts it really disrupts my family, and I'm not sure how to handle it." What is the best response by the nurse? -"Include your husband in the family's activities even when he's been drinking." -"Attend Al-Anon meetings and avoid assuming responsibility for your husband's behavior." -"Search the house regularly for hidden alcohol and accompany your husband outside the home." -"Help your husband avoid embarrassment by making excuses for him when it's impossible for him to function."

Attend Al-Anon meetings and avoid assuming responsibility for your husband's behavior

The two disruptive behavioral disorders most commonly encountered are

Attention-deficit/hyperactivity disorder , Conduct disorder

A parent whose daughter is killed in a school bus accident tearfully tells the nurse, "My daughter was just getting over the chickenpox and didn't want to go to school, but I insisted that she go. It's my fault that she's dead." How should the nurse anticipate that perceiving a death as preventable will likely influence the grieving process?

Bereavement may be of greater intensity and duration.

Because of the emotions attached to a suicidal act, the loss of a loved one through suicide compares with the grief reaction of a natural death in that it is ____ stressful. A) Less B) Just as C) Much more D) Not at all

C. Much more

Children grow in

Cephalocaudal (head to tail) direction

A nurse is teaching a client about side effects of medications. Which drug will cause a heightened skin reaction to sunlight?

Clients taking chlorpromazine should be instructed to stay out of the sun. Photosensitivity makes the skin more susceptible to burning. Photosensitivity is not a side effect of lithium, sertraline, or methylphenidate.

Communication disorders

Communication disorders may result from neurological or other medical conditions, but the cause is often unknown.

A client has been prescribed chlorpromazine (Thorazine) for the management of positive symptoms of schizophrenia. When the client reports difficulty sustaining an erection, the nurse:

Consults with his provider regarding alternative medication therapies

When one is developing the care plan for a female adolescent with an eating disorder, the primary issue to consider as the underlying cause is:

Control

The nurse is reviewing information regarding a female client that was obtained with the psychiatric assessment tool. The clients ability to provide food and shelter for herself is included in which area of the assessment?

Coping responses, discharge planning needs

Diurnal enuresis

Daytime wetting

An adolescent with a conduct disorder is undergoing behavioral therapy in an attempt to limit behaviors that violate societal norms. A specific outcome criterion unique to adolescents with this problem is:

Demonstration of respect for the rights of others

Short-term memory loss is seen in which of the following disorders? (Select all that apply.)

Depression Alzheimers disease Anxiety

A nurse is caring for a client with the diagnosis of schizophrenia. What is a common problem for clients with this diagnosis? Chronic confusion Disordered thinking Rigid personal boundaries Violence directed toward others

Disordered thinking The schizophrenic individual has neurobiological changes that cause disorders in thought process and perceiving reality. Chronic confusion and disorientation are not usually associated with this disorder. Illogical thinking and impaired judgment are associated with schizophrenia. Individuals with the diagnosis of schizophrenia often have personal boundary difficulties. They lack a sense of where their bodies end in relation to where others begin. Loss of ego boundaries can result in depersonalization and derealization. Most clients with schizophrenic disorders are not violent.

A client with bipolar disorder is exhibiting accelerating activity and flight of ideas. What is the best nursing intervention to limit the accelerating manic behavior?

Engaging the client in conversation while walking slowly in the hall

Development

Enhanced ability in skills or functions

A client is brought to the emergency department by friends because of increasingly bizarre behavior. Which signs does the nurse identify that indicate that the client was using cocaine? (Select all that apply.)

Euphoria Agitation Hypervigilance Impaired judgment

A colicky infant

Feeding disorders range from overeating to undereating.

Environmental Problems

Homelessness

A client who is experiencing acute alcohol withdrawal delirium appears frightened, points toward the bed, and says, "Bugs are crawling all over me and my bed!" What is the most therapeutic response by the nurse? -"just try to brush them off" -"I don't see any bugs on your or your bed" -"They'll go away when you start feeling better" -"The bugs that you see are just the design on the bedspread"

I don't see any bugs on you or your bed

Major goal for dissociative identity disorder

Therapy begins with assessment and stabilization.

A client with schizophrenia is started on an antipsychotic/neuroleptic medication. The nurse explains to a family member that this drug primarily is used to:

Make the client more receptive to psychotherapy

A nurse determines that a client is pretending to be ill. What does this behavior usually indicate?

Malingering

nfant regurgitates and rechews food.

Malnutrition may occur because food is brought back to the mouth soon after it is eaten.

A health care provider prescribes divalproex (Depakote). What does the nurse consider an appropriate indication for the use of this drug?

Management of manic episodes of bipolar disorder

A client with a history of chronic alcoholism is admitted to the mental health unit. What does the nurse identify as the cause of a client's use of confabulation?

Marked loss of memory

Anxiety

Most children's bouts of anxiety are relieved when they receive reassurance and emotional support.

A woman who was sexually assaulted by a stranger in the elevator of her apartment building is brought by her husband to the emergency department. What is the priority nursing intervention?

Obtaining information about her perception of the incident

A nurse is teaching clients about dietary restrictions during monoamine oxidase inhibitor (MAOI) therapy. What response does the nurse tell them to anticipate if they do not follow these restrictions?

Occipital headaches

Temper tantrums are a common expression of anger and frustration.

Occur in children between 1 and 4 years of age

Attention-deficit/hyperactivity disorder (ADHD)

Occurs more frequently in boys

A nurse understands that when a client is a member of a different ethnic community it is important to:

Offer a therapeutic regimen compatible with the lifestyle of the family

What should the nurse do to develop a trusting relationship with a disturbed child who acts out?

Offer support and encourage safety during play activities Offering support and encouraging safety during play activities sets a foundation for trust because it allows the child to see that the nurse cares.

Stuttering

Pattern of speech in which sounds are repeated frequently

Careful assessment for changes in attitude and suicidal gestures should be monitored in a client taking which medication?

Paxil (paroxetine)

The nurse who feels the most beneficial part of the client's inpatient stay is the establishment of an interpersonal relationship with the nurse is an example of which nursing theorist?

Peplau

When intimate partner violence (IPV) is suspected, the nurse plays an important role as an advocate for the victim. The advocate role includes several important components. (Select all that apply.)

Planning for future safety Validating the experiences Promoting access to community services

What part of the nursing process includes goals and expected outcomes to develop a care plan (pg.93)?

Planning phase

Environmental Problems

Poverty and mental health problems go hand in hand.

A male client with a diagnosis of antisocial personality disorder is admitted to the mental health hospital. What is the priority nursing intervention?

Presenting a united, consistent staff approach

Therapeutic interventions

Provide the child with emotional support to cope effectively.

Treatment

Requires a multidisciplinary approach, Positive reinforcement , Parent and child education, Medication

What should the nurse do when interacting with an adolescent client with the diagnosis of anorexia nervosa?

Set limits

A mother of a 6-year-old boy with the diagnosis of attention deficit-hyperactivity disorder (ADHD) tells the nurse that when she is reading storybooks to her son, about halfway through the story he becomes distracted, fidgets, and stops paying attention. The nurse suggests that the mother:

Shorten the rest of the story

A depressed client arrives at the mental health unit with mild suicidal ideation but no plan of action. Assessment data reveal that the client has many family responsibilities and adequate family support and attends church regularly. What does the nurse determine about this client?

Should be reassessed at intervals regarding suicidal intent

Somatoform disorder

Signs or symptoms of illness without a traceable physical cause

Schizophrenia

Signs, symptoms, and behaviors of children with schizophrenia vary widely.

Therapeutic Actions

Specific goals are developed for each child.

During the first month in a nursing home, an older client with dementia demonstrates numerous disruptive behaviors related to disorientation and cognitive impairment. What should the nurse take into consideration when planning care?

Stressors that appear to precipitate the client's disruptive behavior

Primary caregiver dysfunction

Support and educate parents and help them develop more effective and appropriate child care skills.

Define "Intervention" of the nursing process (pg.93)

The actual delivery of the planned actions

A client with paranoid schizophrenia tells the nurse, "My neighbors are spying on me because they want to rob me and take money." While hospitalized, the client complains of being poisoned by the food and of being given the wrong medication. The nurse evaluates the client's response to medications and therapy. Which assessment finding leads the nurse to conclude that the client's reality testing has improved? The client eats the food provided on the hospital tray. The client discusses his discharge plans with the staff. The client questions each medication when it is administered. The client asks permission to make phone calls to the hospital administration

The client eats the food provided on the hospital tray. Because the client was admitted while complaining that the food was poisoned, eating the food on the tray indicates that the client feels safe. Discussing discharge plans with the staff does not provide adequate behavioral assessment with which the nurse can evaluate reality testing. Questioning each medication when it is administered indicates that the client still does not completely trust the staff. Asking permission to make phone calls to the hospital administration seems to indicate that the client still does not trust the staff and is attempting to intimidate the staff by calling the administration.

In which situation is the use of seclusion contraindicated?

The client has expressed severe suicidal thoughts. Seclusion of a person experiencing severe suicidal thoughts places the client at risk for self-harm and so would be contraindicated. When the criteria for seclusion have been met, seclusion would not be contraindicated for someone who wants to be secluded, has been voluntarily admitted, or showed minimal improvement despite being secluded before.

A client confides to the nurse, "I've been thinking about suicide lately." What conclusion should the nurse make about the client?

The client is fearful of the impulses and is seeking protection from them.

A client in the hyperactive phase of a mood disorder, bipolar type, is receiving lithium. A nurse sees that the client's lithium blood level is 1.8 mEq/L. What is the most appropriate nursing action?

The lithium level should be maintained between 0.5 and 1.5 mEq/L. The lithium level is currently unsafe but does not need to drop to 0.5 mEq/L before being resumed. Continuing the drug and asking the health care provider to prescribe a higher dosage are both unsafe options.

A nurse is evaluating a young adult for evidence of achievement of the age-related developmental stage set forth in Erikson's developmental theory. What developmental crisis is associated with this age group?

The major task of young adulthood is centered on human closeness and sexual fulfillment; lack of love results in isolation. The trust-versus-mistrust stage is associated with infancy. The industry-versus-inferiority stage is associated with middle childhood (school age). The generativity-versus-stagnation stage is associated with middle adulthood.

What is the most important information for a nurse to teach to prevent relapse in a client with a psychiatric illness?

The need to follow the prescribed medication regimen

Define Observation (pg.94)

The process of purposeful looking

Children with conduct disorders are defiant of authority.

They engage in aggressive actions toward other people, refuse to follow society's rules and norms, and violate the rights of others.

A nurse, understanding the possible cause of alcohol-induced amnestic disorder, should take into consideration that the client is probably experiencing:

Thiamine deficiency

What is the dynamic process during evaluation (pg.94)?

Treatment

A nurse is caring for a client with antisocial personality disorder. What client characteristic should the nurse consider when formulating a plan of care?

Usually is unable to postpone gratification(희열) Individuals with antisocial personality disorder tend to be self-centered and impulsive. They lack judgment and self-control and are unable to postpone gratification. Generally they do not suffer from anxiety. These individuals believe that the rules do not apply to them, and they do not profit from their mistakes. These people are too self-centered to have a sense of responsibility to anyone.

C

While questioning a rape victim, the nurse discovers that the victim does not remember anything related to the assault. Of the following, which is the most probable cause of the victim's memory loss? A: The rape victim was using opioids. B: The rape victim was using hallucinogens. C: The rape victim was drugged with flunitrazepam. D: The rape victim was under the influence of alcohol.

A client with schizophrenia is actively psychotic, and a new medication regimen is prescribed. A student nurse asks the nurse, "Which of the medications will be the most helpful against the psychotic signs and symptoms?" What response should the nurse give? Citalopram (Celexa) Ziprasidone (Geodon) Benztropine (Cogentin) Acetaminophen with Hydrocodone (Lortab)

Ziprasidone (Geodon)

Clients with bipolar I, bipolar II, or cyclothymic disorders exhibit different types of: A) Mania B) Anxiety C) Dysthymia D) Regression

a. Mania

1. The concept of recidivism is prevalent among individuals with chronic psychiatric problems. Which is the most accurate description of this concept?

a. Relapse of symptoms of a client's mental health disease, resulting in frequent readmission to facilities.

The wife of a recently deceased client states that she is going to return to work and rejoin her exercise class. She is displaying behavior that occurs at which stage of grieving?

acceptance and recovery

a practice of stereotyping older persons as feeble, dependent, and nonproductive. Look at their attitudes and values about aging.

ageism

inability to recognize familiar objects or people

agnosia

While completing the history portion of an admission assessment of a client with schizophrenia, the nurse notices that the client is continually moving in the chair and frequently stands, then sits back down. The nurse knows that this client most likely is experiencing the side effect of

akathisia

C

client comes to a trauma center reporting that she has been raped. She is disheveled, pale, and staring blankly. The nurse asks the client to describe what happened. What is the nurse's rationale for doing this? A: It will help the nursing staff give legal advice and provide counseling. B: Talking about the assault will help the client see how her behavior may have led to the event. C: It will let the victim put the event in better perspective and help begin the resolution process. D: Discussing the details will keep the victim from concealing the intimate happenings during the assault.

Inability to consider the abstract meaning of a phase; frequently tested by having clients interpret proverbs. Example: a snitch in time saves nine, may mean sew the holes in your clothes to a schizophrenic person.

concrete thinking

A depressed, withdrawn client exhibits sadness through nonverbal behavior. What should the nurse plan to help the client to do? -increase structured physical activity -cope with painful feelings by sharing them -decide which unit activities the client can perform -improve the ability to communicate with significant others

cope with painful feelings by sharing them

79 y/o with dementia remembers procedures and basic nursing care interventions in her past career

crystallized intellgence

1. Confused and thinks the nurse is actually the administrator

delusions

associated with separation and abandonment. People with this problem carry a deep fear of rejection, which expresses itself as the need to be cared for. They become over cooperative and docile to avoid turning people away. They refuse to be responsible for their own actions and unwilling to begin a task alone. Feelings of worthlessness motivate them to seek out overprotective, dominating, or abusive relationships. Men and women are equally diagnosed.

dependent

post-traumatic stress disorder (PTSD)

develops following an extremely traumatic event that involves injury or threat to the child

296 any form of active suicidal behavior, such as threats, gestures, or attempts to end one's life. Individual intends to commit suicide.

direct self-destructive behavior

A male client with schizophrenia begins to have hallucinations during a conversation with the nurse; this prevents him from receiving the message that the nurse is trying to communicate to him. According to Ruesch's theory of communication, this is unsuccessful interaction is called

disturbed communication

incorrect perceptions of caused events as having great or significant meaning. Ideas of media or people are talking about oneself.

ideas of reference

dyslexia

impairment of the ability to read

MAOI

must have low tyramine diet

somatoform disorders

signs or symptoms of illness without a traceable physical cause(school age)

bullying

the repeated use of aggressive behaviors to intentionally intimidate another is very common in schools today.

An adult child of a dying client says to the nurse in the nursing home, "I am so upset because my parent is always angry at me." What is the nurse's best initial response? -"your parent is frightened by impending death" -"your parent is working through acceptance of the situation" -"Your parent is attempting to reduce your need for dependency" -"Your parent is hurt that you will not provide physical care at home"

your parent is working through acceptance of the situation

A health care provider refers a 52-year-old man to the mental health clinic. The history reveals that the man lost his wife to colon cancer 6 months ago and that since that time he has seen his health care provider seven times with the concern that he has colon cancer. All tests have had negative results. Recently the client stopped seeing friends, dropped his hobbies, and stayed home to rest. Which disorder should the nurse identify as consistent with the client's preoccupation with the fear of having a serious disease?

Hypochondriac disorder

A client tells the nurse, "That man on the television is talking only to me." What should the nurse document that the client is exhibiting?

Idea of reference An idea of reference, also called a delusion of reference, is a fixed, false personal belief that public events and people are connected directly to the client. An illusion is a misinterpretation of a sensory stimulus. A hallucination is a perceived experience that occurs in the absence of an actual sensory stimulus. Autistic thinking is a distortion in the thought process that is associated with schizophrenic disorders.

Children and violence

If a child's conduct becomes inappropriate over time, a disruptive behavioral disorder may be diagnosed.

Anxiety

If anxiety does not interfere with relations with family or friends or activities at school, the child is coping effectively.

A nurse in the mental health clinic concludes that a client is using confabulation when:

Imagination is used to fill in memory gaps

An older female client is concerned about maintaining her independent living status. What initial intervention strategy is of primary importance?

Reinforcing routines and supporting her usual habits The client has been able to function well up to this time, and her usual behaviors and routines should be supported. The data presented do not show a need to get the client help with cleaning and shopping, to write down and repeat information, or to set goals and time limits for the client's visits with the nurse.

What characteristic of anxiety is associated with a diagnosis of conversion disorder?

Relieved by the symptom

The nurse refers a client to a self-help group. What does the nurse anticipate that a self-help groups such as Alcoholics Anonymous (AA) will help its members learn?

That their problems are not unique

A client with schizophrenia repeatedly says to the nurse, "No moley, jandu!" The nurse determines that the client is exhibiting:

Neologism

__________ is a side effect that can occur while a client is taking an antipsychotic medication, causing muscle rigidity, high fever, unstable vital signs, confusion, and agitation

Neuroleptic malignant syndrome (NMS)

The nurse is aware that he or she may be administering the new antianxiety medication pregabalin (Lyrica) to clients without an anxiety disorder for the purpose of treating:

Neuropathic pain

Problems with sleep are common to many children.

Night terrors, problems falling asleep, and nighttime awakenings

Clinical findings of depression

Persistent state of unhappiness that interferes with pleasure or productivity

Gross disturbance in social behavior with no impairment in mental state.

psychopath

An older client with a diagnosis of dementia is living in a long-term care facility. The client's daughter, who lives 300 miles away, calls the unit to speak to the nurse about her upcoming visit. What should the nurse say in response to her question about the best time of day to visit?

"Around 2:30 in the afternoon is the best time to visit."

Depression

A prolonged feeling of helplessness, hopelessness, and sadness

Rett syndrome

Development of motor, language, and social problems and loss of previous skills

What is an initial client objective in relation to anger management?

Taking responsibility for the hostile behavior

Colic

consists of a set of behaviors that are seen most commonly in middle-class infants.

neglect

not meeting a child's basic needs for food,clothing, shelter,love and belonging

When reviewing the nursing notes from the previous shift, the nurse notices notations indicating that the client was experiencing a somnolent level of consciousness. The clients behavior would be described as:

"Falling asleep easily and only awakening with strong verbal stimuli"

A client is admitted to the hospital because of incapacitating obsessive-compulsive behavior. Which statement best describes how clients with obsessive-compulsive behavior view this disorder?

"I know there's no reason to do these things, but I can't help myself."

A 17-year-old teenager is found to have leukemia. Which statements by the teenager reflect Piaget's cognitive processes associated with adolescence? (Select all that apply.)

"I'm going to do my best to fight this awful disease." "Now I can't go to the prom because I have this stupid disease." "This illness is serious, but with treatment I think I have a chance to get better."

A nurse approaches a depressed client who is sitting alone in the dayroom. What is best for the nurse to say to the client?

"I'll be sitting with you for a while today."

When talking with a client who has been receiving Paroxetine (Paxil), the nurse determines that more clarification is needed when the client says:

"I've been on the medication for 8 days now, and I don't feel any better."

A client is extremely depressed, and the practitioner prescribes a tricyclic antidepressant, imipramine (Tofranil). The client asks the nurse what the medication will do. The nurse responds:

"It will help increase your appetite and make you feel better."

A client tells the nurse in the mental health clinic that the practitioner said that the cornerstone of therapy used in the clinic is cognitive therapy. The client asks what this therapy entails. What concept should the nurse explain as the basis of cognitive therapy?

"Negative thoughts can precipitate anxiety."

A 6-year-old child is diagnosed with type 1 diabetes. Considering the child's cognitive developmental level, which explanation of the illness is most appropriate? "Diabetes is caused by not having any insulin in your body." "Diabetes will require you to take insulin shots for the rest of your life." "You will be taught how to give yourself insulin now that you have diabetes." "Taking insulin for your diabetes is like getting new batteries for your superhero toys."

"Taking insulin for your diabetes is like getting new batteries for your superhero toys." The child is in Piaget's stage of preoperational thought, which is manifested by magical thinking; therefore, teaching should also use magical thinking.

A client who was forced into early retirement is found to have severe depression. The client says, "I feel useless, and I've got nothing to do." What is the best initial response by the nurse?

"Tell me more about feeling useless."

A nurse is caring for a client with a somatoform disorder. What should the nurse anticipate that this client will do? 1 Redirect the conversation with the nurse to physical symptoms 2 Monopolize conversations about the anxiety being experienced 3 Write down conversations to facilitate the recall of information 4 Start a conversation asking the nurse to recommend palliative care

1 Redirect the conversation with the nurse to physical symptoms Clients with somatoform disorders are preoccupied with the symptoms that are being experienced and usually do not want to talk about their emotions or relate them to their current situation. Clients with somatoform disorders do not seek opportunities to discuss their feelings. Memory problems are not associated with somatoform disorders. These clients want and seek treatment, not palliative care.

One day while shaving, a male client with the diagnosis of bipolar disorder tells the nurse, "I've hidden a razor blade, and tonight I'm going to kill myself." What is the best reply by the nurse? 1. "You're going to kill yourself?" 2. "Things really can't be that bad." 3. "Are you sure you really mean that?" 4. "Killing yourself is not going to solve your problems."

1. "You're going to kill yourself?"

On the psychiatric unit a client has been receiving high doses of haloperidol (Haldol) for 2 weeks. The client says, "I just can't sit still, and I feel jittery." Which side effect does the nurse suspect that the client is experiencing? 1. Akathisia 2. Torticollis 3. Tardive dyskinesia 4. Parkinsonian syndrome

1. Akathisia **Akathisia, a side effect of haloperidol (Haldol), develops early in therapy and is characterized by restlessness and agitation.

One afternoon a nurse sees a client rushing down the hall of the mental health unit, rapidly tapping his fingers against the wall. What is the most appropriate nursing action at this time? 1. Approaching the client in a nonthreatening manner to determine the cause of the agitation 2. Summoning additional staff members to forcefully subdue the client and stop the acting-out behavior 3. Observing the client to see whether the behavior escalates and whether it may pose a risk to other clients or staff 4. Immediately obtaining staff assistance to enable administration of medication prescribed for the client's agitation

1. Approaching the client in a nonthreatening manner to determine the cause of the agitation

When intimate partner violence (IPV) is suspected, the nurse plays an important role as an advocate for the victim. The advocate role includes several important components. (Select all that apply.) 1. Planning for future safety 2. Normalizing victimization 3. Validating the experiences 4 .Promoting access to community services

1. Planning for future safety 3. Validating the experiences 4 .Promoting access to community services

When talking with a client who has alcoholism, the nurse notes that the client becomes irritable, makes excuses, and blames family and friends for the drinking problem. Which defense mechanisms does the nurse conclude that the client is using? (Select all that apply.) 1. Projection 2. Suppression 3. Sublimation 4. Identification 5. Rationalization

1. Projection 5. Rationalization **Projection is the unconscious denial of unacceptable feelings and emotions in one's self while attributing them to others. This defense mechanism commonly is used by clients with alcoholism because it helps make reality more acceptable. Rationalization is making acceptable excuses for behavior; this defense is used by people with alcoholism because it makes reality more acceptable.

A student feels anxious about being unprepared for an upcoming test. Which of the following is a positive response to this anxiety? 1. canceling nonessential activities for 3 days to study 2. staying up all night the night before the test to study 3. borrowing notes from another student who has had the class in the past 4. choosing not to worry about studying because grades have been good before

1. canceling nonessential activities for 3 days to study

The spouse of an older male patient is concerned because since retiring the patient sits around the house, avoids eating, naps, and refuses to participate in sporting activities. Which disorder should the nurse recognize as being associated with these manifestations? 1. depression 2.bipolar disorder 3. conversion disorder 4. post traumatic stress disorder

1. depression

After talking with a patient being evaluated for a mental health disorder, the nurse says, " Its sounds as if you are feeling angry". Which therapeutic communication technique is the nurse employing? 1. restating 2. refelecting 3. giving recongnition 4. offering a general lead

1. restating

A young mother is angry with the mess her son has made in his room. She yells at him and tells him to stay in his room until it is cleaned up. About 20 minutes later, the mother enters the boys room offering him milk and cookies. Which ego defense mechanism is the mother using? 1. restitution 2. regression 3. reaction formation 4. conversion reaction

1. restitution

The nurse is caring for a patient experiencing depression. Which neurotransmitters should the nurse consider being decreased in the patient?SELECT ALL THAT APPLY 1. serotonin 2. dopamine 3. substance P 4. norepinephrine 5. acetylcholine

1. serotonin 2. dopamine 5. acetylcholine

A patient comes into the ER experiencing chest pain and feeling of impending doom. Which assessment finding should the nurse use to determine if this patient is experiencing a panic attack?SELECT ALL THAT APPLY 1. shaking 2. neck pain 3. dissociation 4. vomiting brown emesis 5. occurs a 3 p.m everyday

1. shaking 3. dissociation 5. occurs a 3 p.m everyday

The nurse is providing care for a patient with symptoms of tardive dyskinesia from major tranquilizers. What treatment should the nurse anticipate? 1. use of anticholinergic agents 2. use of muscle relaxant agents 3. discontinuance of the tranquilizers 4. addition of rational emotive therapy to the treatment plan

1. use of anticholinergic agents

A person mowing a lawn is badly disfigured by the lawnmower blade. According to Erikson's theory, which age at the time of injury will be associated with the greatest risk of long-term psychological effects?

11 years An 11-year-old child is generally in Erikson's stage of industry versus inferiority, which involves the mastery of skills; unfortunately, the child did not master the skill of lawnmowing.

Adolescence is defined as beginning at age _____ years and ending at age _____ years.

11, 21

A client is admitted with a conversion disorder. What is the primary nursing intervention? 1 Talking about the physical problems Correct2 Exploring ways to verbalize feelings 3 Explaining how stress caused the physical symptoms 4 Focusing on the client's concerns regarding the symptoms

2 Exploring ways to verbalize feelings The priority is getting the client to express feelings appropriately rather than through the use of physical symptoms. Focusing on symptoms will encourage their use by the client. An expression of feelings, not an intellectual understanding of the cause of the symptoms, is required. Avoidance of feelings resulted in the symptoms. Clients with a conversion disorder are rarely concerned about the associated physical problem; this is known as la belle indifférence.

During client teaching, the nurse must inform the client prescribed a tricyclic antidepressant (TCA) to not expect to see a difference in mood or anxiety level for up to:

2 to 3 weeks

A patient hospitalized for bipolar disorder is sitting in the corner of the room with the lights off, staring into space. Three hours later, the patient is in the same position. What should the nurse say to the patient? 1. cheer up! come on out and join us in a game! 2. come with me. I'd like you to join our group for a while 3. you won't make any progress if you stay in your room all the time 4. what's the matter? don't you know you should be in your group right now?

2. come with me. I'd like you to join our group for a while

A patient with depression is prescribed duloxetine (Cymbalta). What should the nurse instruct the patient about this medication? 1. take with fruit juice 2. do not take with St. John's wort 3. stop the medication if experiencing adverse effects 4. expect blood pressure to drop with this medication

2. do not take with St. John's wort

The nurse is caring for a patient experiencing anxiety. Which techniques should the nurse use when instructing on progressive muscle relaxation therapy? SELECT ALL THAT APPLY 1. start at the head and neck 2. end with the lower extremities 3. play soft music to aid in relaxation 4. systematically tense and relax muscle groups 5. imagining a pleasurable experience from the past

2. end with the lower extremities 4. systematically tense and relax muscle groups 5. imagining a pleasurable experience from the past

A patient is diagnosed has having a phobia. Which fear should the nurse expect to observe in this patient? 1. fear of poisonous spiders 2. fear of leaving the house during the day 3. fear of failing a test that one has not studied for 4. fear that a child playing in the street might get hurt

2. fear of leaving the house during the day

In conjunction with which classification of medication are trihexyphenidyl, biperiden (Akineton), and benztropine (Cogentin) often prescribed? 1. Anxiolytics 2. Barbiturates 3. Antipsychotics 4. Antidepressants

3. Antipsychotics

What should a nurse recognize that a client who uses the defense mechanism of sublimation is doing? 1. Acting out in reverse something already done or thought 2. Returning to an earlier, less mature stage of development 3. Channeling unacceptable impulses into socially approved behavior 4. Excluding from consciousness thoughts that are psychologically disturbing

3. Channeling unacceptable impulses into socially approved behavior

A nurse leads an assertiveness training program for a group of clients. Which statement by a client indicates that the treatment has been effective? 1."I know that I should put the needs of others before mine." 2."I won't stand for it, so I told my boss he's a jerk and to get off my back." 3."It annoys me when people call me 'sweetie,' so I told him not to do it anymore." 4."It's easier for me to agree up front and then do just enough so that no one notices."

3."It annoys me when people call me 'sweetie,' so I told him not to do it anymore."

A woman who is frequently physically abused tells the nurse in the emergency department that it is her fault that her husband beats her. What is the most therapeutic response by the nurse? 1 "Maybe it was your husband's fault, too." 2 "I can't agree with that—no one should be beaten." 3 "Tell me why you believe that you deserve to be beaten." 4 "You say that it was your fault—help me understand that."

4

A nurse is caring for several clients who have severe psychiatric disorders. What is the major reason that a health care provider prescribes an antipsychotic medication for these clients? 1. To improve judgment 2. To promote social skills 3. To diminish neurotic behavior 4. To reduce the positive symptoms of psychosis

4. To reduce the positive symptoms of psychosis **Antipsychotics are used to decrease positive signs and symptoms associated with psychoses, including hallucinations, delusions, paranoia, and disorganized speech.

A 15-year-old female started going to parties earlier in the year where drinking and drug use occurred. Her friends now notice that she actively looks for a party every weekend and attends even when her friends are not invited. If they suggest an alternate activity, she refuses to go with them, and is avoiding them at school. What stage of chemical dependency is she exhibiting?

Actively seeking

are periods of dysfunction accompanied by an increase in the signs, symptoms, and seriousness of a problem.

Exacerbations

The four classes of psychotherapeutic medications include antianxiety agents, antidepressants, antimanics, and __________.

Antipsychotics

In conjunction with which classification of medication are trihexyphenidyl, biperiden (Akineton), and benztropine (Cogentin) often prescribed?

Antipsychotics Antipsychotics are used to control the extrapyramidal (parkinsonian) symptoms that often develop as a side effect of antipsychotic therapy. There is no documented use of anxiolytics with antianxiety agents because they do not have extrapyramidal side effects. Barbiturates do not have extrapyramidal side effects that respond to these drugs. Antiparkinsonian drugs usually are not prescribed in conjunction with antidepressants because antidepressants do not cause parkinsonian symptoms.

In conjunction with which classification of medication are trihexyphenidyl, biperiden (Akineton), and benztropine (Cogentin) often prescribed?

Antipsychotics Antipsychotics are used to control the extrapyramidal (parkinsonian) symptoms that often develop as a side effect of antipsychotic therapy.

What would be some psychiatric assessment examples of "Coping responses, mental status" (pg.95)?

Appearance, speech, motor activity, mood, affect, interactions, perceptions, thought content and process, memory, concentration, calculations, intelligence, insight, judgement

A laboring client reports low back pain. Which intervention should the nurse recommend to the client's coach to promote the most comfort for this client?

Apply pressure to her back during contractions.

What is the nurse's primary outcome goal when managing the care of a client diagnosed with generalized anxiety disorder (GAD)?

Assisting the client with the development of healthy, adaptive coping mechanisms

The nurse should suspect that a client who had a recent myocardial infarction is experiencing denial when the client does what?

Attempts to minimize the illness

As the nurse is discussing psychiatric care with an older adult client, the client says, "When I was growing up I was taught to accept my lot in life and not complain. I'm proud of the fact that despite my issues I can still function independently. I don't want to be just put away." The nurse understands that the factors that influence the client's mental health are examples of:

Attitudes and beliefs Some attitudes and beliefs include reluctance by older people to seek help because of pride in their independence, stoic acceptance of difficulty, unawareness of resources, and fear of being "put away." Although the client mentions "being put away", that is an attitude. He is not talking about all the resources that might be available to him. Anxiety is defined as an unpleasant and unwarranted feeling of apprehension. The client does not mention any cultural or ethnic issues, just his own feelings.

Thirty minutes after administering fluphenazine (Prolixin) to a client, the nurse notes that the client's jaw is rigid, the client is drooling, and her speech is slurred. There are a number of as-needed prescriptions in the client's chart. What should the nurse administer?

Benztropine (Cogentin), 2 mg intramuscularly7

A client has recently started taking a new neuroleptic drug, and the nurse notes extrapyramidal effects. Which drug does the nurse anticipate will be prescribed to limit these side effects?

Benztropine (Cogentin), an anticholinergic, helps balance neurotransmitter activity in the central nervous system (CNS) and helps control extrapyramidal tract symptoms. Zolpidem (Ambien) is a sedative-hypnotic drug used for short-term insomnia. Hydroxyzine (Vistaril) is a sedative that depresses activity in the subcortical areas in the CNS; it is used to reduce anxiety. Dantrolene (Dantrium), a muscle relaxant, has a direct effect on skeletal muscle by acting on the excitation-contraction coupling of muscle fibers and not at the level of the CNS as do most other muscle relaxation drugs.

Episodes of depression altering with episodes of mania

Bipolar I

major episodes of depression followed by hypomania

Bipolar II

Which of the following assessment parameters is a priority in a biobehavioral model of practice?

Blood chemistry

A nurse is aware that after the administration of alprazolam (Xanax) is started, it is important to assess the client for side effects. What is the nurse's initial action?

Check the client's blood pressure. Orthostatic hypotension is a common side effect of alprazolam (Xanax) that occurs early in therapy. Central nervous system depression is not an early side effect, but it may occur after prolonged use. An alteration in urine output is not a common side effect, but it may occur after prolonged use. Distention is not a common side effect, but distention from constipation may occur after prolonged use.

Operational defiant disorder

Children frequently lose their tempers, argue with adults, deliberately annoy other people, and refuse to compromise.

Pervasive Developmental Disorders

Children have difficulty with social interaction skills, communication skills, and learning.

Children and violence

Children may witness violence in the home and adopt it as a method for solving problems or resolving conflict.

Problems of Childhood

Children must master skills that move them from complete dependence to independent functioning in only a few short years.

Problems With Eating and Elimination

Children who have problems with eating are defined as routinely failing to eat adequately.

The nurse suspects the client is experiencing a manic episode based on which of the following observations?

Clothing is very colorful and mismatched, and client cannot sit in chair during interview.

Carl Jung was the founder of analytical psychotherapy; he differed from Freud in that he believed that the mind was divided into three levels: the conscious ego, the personal unconscious, and the:

Collective unconscious

When a person who is nonathletic and uncoordinated is successful in a musical career, it may be related to the defense mechanism of:

Compensation is replacing a weak area or trait with a more desirable one. Sublimation is rechanneling unacceptable desires and drives into those that are socially acceptable. Transference is the unconscious tendency to assign to others in the current environment feelings and attitudes associated with another person. Rationalization is the use of justification to make tolerable certain feelings, behaviors, and motives.

Schizophrenia

Condition associated with disturbing thought patterns and a distorted reality

A client on the psychiatric unit who is receiving high-dosage risperidone (Risperdal) is exhibiting tremors of the hands. What should be the nurse's first intervention? 1 Withholding the medication 2 Telling the client it is transitory 3 Giving the client finger exercises 4 Contacting the health care provider

Contacting the health care provider The health care provider is responsible for prescribing medications but depends on the nurse's observations before making decisions. This is not a severe enough finding to warrant withholding the drug. It is a reaction to the risperidone (Risperdal), and it is not transitory. Giving the client finger exercises will have no effect on the tremors.

A 25-year-old woman with the diagnosis of bipolar disorder, manic episode, is admitted to the psychiatric unit. A nurse on the unit reviews the admission information provided by the client's husband and assesses the client. In light of the information in the chart, what is an appropriate nursing intervention? 1 Assigning the client to a private room 2 Suggesting that the client play cards with several other clients Incorrect3 Encouraging the development of insight through introspection 4 Having the client sit at the communal dining table during meals

Correct1 Assigning the client to a private room

The behavioral state of thoughts, feelings, and activities that follow a loss is called: A) Grief B) Anxiety C) Mourning D) Bereavement

D. Bereavement

The client lost her husband of 50 years 10 months ago. She now sees every day as a gray fog with no light. She has begun to experience changes in eating, sleeping, and activity levels; angry, hostile moods; and an inability to concentrate or complete work tasks. What is the client experiencing? A) Complicated grief B) A normal grief reaction C) Complicated depression D) Bereavement-related depression

D. Bereavement-related depression

Bob drives fast everywhere he goes, especially when he is drinking. Last night, he was arrested for gambling and loud behavior. Bob is engaging in ____ behavior. A) Adaptive B) Suicidal C) Direct self-destructive D) Indirect self-destructive

D. Indirect self-destructive

A female client who is severely incapacitated by obsessive-compulsive behavior has been admitted to the mental health hospital. The client's compulsive ritual involves changing her clothing 8 to 12 times a day. She continually asks the nurse for advice regarding her problems but then ignores it. This is an example of the conflict of:

Dependence versus independence

A nurse is volunteering on the community crisis hotline. What is the final objective of the counseling process?

Developing constructive coping skills

A child in the first grade is murdered, and counseling is planned for the other children in the school. What should a nurse identify first before evaluating a child's response to a crisis?

Developmental level of the child Knowledge of the developmental level is essential to understanding a child's response to a crisis situation; the variety of coping abilities usually increases as the child progresses through the stages of growth and development.

Learning disorders

Diagnosed when a child with normal intelligence falls below other children of the same age and grade on standard reading, mathematics, or written tests

What would be some psychiatric assessment examples of "Previous psychiatric treatment" (pg.95)?

Diagnosis, type of treatment, medications, compliance, psychiatric history in family

A client who is on the third day of detoxification therapy becomes agitated and restless. What are the signs and symptoms that indicate impending alcohol withdrawal delirium? (Select all that apply.)

Diaphoresis Tachycardia Hypertension

A practitioner prescribes Alprazolam (Xanax) 0.25 mg by mouth three times a day for a client with anxiety and physical symptoms related to work pressures. For what most common side effect of this drug should the nurse monitor the client?

Drowsiness

According to Erikson, a person's adjustment to the period of senescence will depend largely on the adjustment the individual made to the earlier developmental stage of:

Erikson theorized that how well people adapt to the current stage depends on how well they adapted to the stage immediately preceding it—in this instance, adulthood. Although Erikson believed that the strengths and weaknesses of each stage are present in some form in all succeeding stages, their influence decreases with time.

A client is admitted to the psychiatric hospital with a diagnosis of obsessive-compulsive disorder. The client's anxiety level is approaching a panic level, and the client's ritual is interfering with work and daily living. Which selective serotonin reuptake inhibitor (SSRI) should the nurse anticipate that the health care provider may prescribe?

Fluvoxamine (Luvox) inhibits central nervous system neuron uptake of serotonin but not of norepinephrine. Haloperidol (Haldol) is not an SSRI; it is an antipsychotic that blocks neurotransmission produced by dopamine at synapses. Imipramine (Tofranil) is a tricyclic antidepressant, not an SSRI. Benztropine (Cogentin) is an antiparkinsonian agent, not an SSRI.

What should the nurse do to achieve a primary objective of providing a therapeutic daycare environment for a client who is withdrawn and reclusive?

Foster a trusting relationship

A client is admitted to the emergency department after ingesting a tricyclic antidepressant in an amount 30 times the daily recommended dose. What is the immediate treatment anticipated by the nurse?

Gastric lavage with charcoal may help decrease the level of tricyclic antidepressant overdose. Supportive measures such as mechanical ventilation may be needed until the medical crisis passes

The nurse is caring for a client with the diagnosis of bipolar disorder, manic episode. Which clinical findings support the diagnosis? (Select all that apply.)

Grandiosity 과장, 떠벌림 Talkativeness Distractibility

Relatives of the victims of a home invasion in which several family members were killed receive crisis intervention services. Which therapy is most beneficial after the immediate event has passed?

Grief

Normal childhood development

Growth is a continuing process.

An older client is transferred to a nursing home from a hospital with a diagnosis of dementia. One morning, after being in the nursing home for several days, the client is going to join a group of residents in recreational therapy. The nurse sees that the client has laid out several outfits on the bed but is still wearing nightclothes. What should the nurse do?

Help the client select appropriate attire and offer to help the client get dressed

One morning, during the working phase of a therapeutic relationship after several sessions in which difficult issues were discussed, the client suddenly becomes very hostile. What is the most appropriate interpretation of this behavior by the nurse?

Hostility is being used as a defense because previous self-disclosure has raised anxiety. Emotional closeness after self-disclosure increases anxiety, which cannot be tolerated; hostility is used to keep the nurse at a distance. Hostility is more extreme than assertiveness and is not an indication of improvement. Although flare-ups often occur even when there is a positive working relationship, the expression of hostility is not a flare-up in this situation. Regressive behavior is the resumption of behavior characteristic of an earlier stage of development; hostility does not fit this definition.

A nurse uses behavior modification to foster toilet-training efforts in a cognitively impaired child. What reward should the nurse provide to reinforce appropriate use of the toilet?

Hug with praise

A score of 1 to 10 on the global assessment functioning (GAF) scale would indicate that a client was at risk for:

Hurting himself or others

The husband of a woman who gave birth to a baby 2 weeks ago calls the postpartum unit at the hospital, seeking assistance for his wife. He reports that he found his wife in bed and that the baby was wet, dirty, and crying in the crib. He says, "She says she just can't do it." What is the best response by the nurse?

Informing him that he should seek emergency intervention for his wife

The nurse explains to the mother of a preschool child that Erikson identified the developmental conflict of children from 3 to 5 years as:

Initiative versus guilt

Jean Piaget's theory of cognitive development identifies an interrelationship between the __________ and the __________ functions in the development of one's personality.

Intellectual; emotional

A client with diabetes mellitus is able to discuss in detail the diabetic metabolic process while eating a piece of chocolate cake. What defense mechanism does the nurse identify when evaluating this behavior?

Intellectualization

To provide appropriate psychosocial support to clients, a nurse must understand development across the life span. What theory is the nurse using in considering relationships and resulting behaviors the central factors that influence development?

Interpersonal theory

The assessment phase of the nursing process refers to the phase when data collection occurs. Which methods does the nurse use to collect data? (Select all that apply.)

Interviewing the client and significant others Observing client behavior Performing physical assessment Reviewing diagnostic testing results

What conflict associated with Erikson's psychosocial stages of development should the nurse remember when caring for a client 30 years of age?

Intimacy versus isolation

dyskinesia

Involuntary abnormal skeletal muscle movements usually seen as jerking motions and sometimes seriously interfere with the client's ability to walk and perform other voluntary movements.

Enuresis

Involuntary urinary incontinence of a child 5 years or older

Asperger's syndrome

Long-lasting impairment in social interactions with repeated patterns of behavior, interest, and activities

The nurse anticipates that the medication that will be used to prevent symptoms of withdrawal in clients with a long history of alcohol abuse is:

Lorazepam (Ativan)

A 70 year old male tells the nurse he is using chelation therapy to prevent alzheimer's disease. Which adverse effect must the client watch for?

Low potassium levels

A client is receiving a monoamine oxidase inhibitor (MAOI). What should the nurse teach the client?

MAOIs interact with many other medications to produce harmful side effects. Clients must be taught to check with the prescribing health care provider before taking any new medications. Photosensitivity has not been reported in clients who are taking MAOIs. Drowsiness is not an expected side effect, but it may occur as an adverse reaction. The therapeutic and toxic levels of the drug are not close for these medications.

A nurse is caring for depressed older adults. What precipitating factors for depression are most common in the older adult without cognitive problems? (Select all that apply.)

Multiple losses Declines in health

Problems with Parent-Child Interactions

One of the most common parent-child problems is conflict.

The treatment team meets with a client for the first time and determines, with the clients input, a nursing diagnosis, goal, and steps to reach this goal. In addition to a nursing diagnosis, the treatment team has completed which phase of the nursing process?

Planning

A young client who has become a mother for the first time is anxious about her new parenting role. With the nurse's encouragement, she joins the new mothers' support group at the local YMCA. What kind of prevention does this activity reflect?

Primary prevention Primary prevention is directed toward health promotion and prevention of problems. Tertiary prevention is focused on rehabilitation and the reduction of residual effects of illness. Secondary prevention is related to early detection and treatment of problems.

A 23-year-old woman is admitted to a psychiatric unit after several episodes of uncontrolled rage at her parents' home, and borderline personality disorder is diagnosed. While watching a television newscast describing an incident of violence in the home, the client says, "People like that need to be put away before they kill someone." The nurse concludes that the client is using:

Projection

A nurse is working with an adolescent client with conduct disorder. Which strategies should the nurse implement while working on the goal of increasing the client's ability to meet personal needs without manipulating others? (Select all that apply.)

Provide physical outlets for aggressive feelings Establish a contract regarding manipulative behavior Develop activities that provide opportunities for success

Which psychotherapeutic theory uses hypnosis, dream interpretation, and free association as methods to release repressed feelings?

Psychoanalytical model The psychoanalytical model studies the unconscious and uses the strategies of hypnosis, dream interpretation, and free association to encourage the release of repressed feelings. The behaviorist model holds that the self and mental symptoms are learned behaviors that persist because they are consciously rewarding to the individual; this model deals with behaviors on a conscious level of awareness. The psychobiological model views emotional and behavioral disturbances as stemming from a physical disease; abnormal behavior is directly attributed to a disease process. This model deals with behaviors on a conscious level of awareness. The social-interpersonal model affirms that crucial social processes are involved in the development and resolution of disturbed behavior; this model deals with behavior on a conscious level of awareness.

Which of the following are common signs and symptoms of the eating disorder bulimia? (Select all that apply.)

Purging after meals Consumption of 5000 to 20,000 calories per day Erosion of tooth enamel

What should the nurse do when a client with the diagnosis of schizophrenia talks about being controlled by others?

React to the feeling tone of the client's delusion

A client who is being treated in a mental health clinic is to be discharged after several months of therapy. The client anxiously tells the nurse, "I don't know what I'll do when I can't see you anymore." The nurse determines that the client is:

Reacting to the planned discharge

A nurse becomes aware of an older client's feeling of loneliness when the client states, "I only have a few friends. My daughter lives in another state and couldn't care less whether I live or die. She doesn't even know I'm in the hospital." The nurse identifies the client's communication as a:

Reflection of depression that is causing feelings of hopelessness

What defense mechanism should the nurse anticipate that a client with the diagnosis of schizophrenia, undifferentiated type, will most often exhibit?

Regression

An older female client is concerned about maintaining her independent living status. What initial intervention strategy is of primary importance?

Reinforcing routines and supporting her usual habits

A neuromuscular blocking agent is administered to a client before electroconvulsive therapy. At this time, the nurse should monitor the client for:

Respiratory difficulties

What is the nurse's specific responsibility when the rights of a client on a mental health unit are restricted by the use of seclusion?

Seclusion and restraints are special procedures for dealing with aggressive acting-out behavior for the protection of the client and others; clear documentation is essential when the client's rights are restricted. Informing the client's family is not necessary because the use of seclusion or restraints is included in the general consent form that is signed on admission. Pharmacological intervention should be monitored for all clients. There is not a typical form; however, documentation is required to justify the need for seclusion or the use of restraints.

A female client is 3-days postoperative and has been receiving meperidine (Demerol) for pain control. The family mentions to the nurse that the client has been taking phenelzine (Nardil) for years for her depression. The client did not list this medication on admission. What signs and symptoms should the nurse look for in case of reaction between these two medications?

Sedation, disorientation, and hallucinations

Olanzapine (Zyprexa) is prescribed for a client with bipolar disorder, manic episode. What cautionary advice should the nurse give the client?

Sit up slowly.

What is the most appropriate intervention for the nurse to implement after finding a disturbed client in bed in the fetal position?

Sitting down in a chair by the client and saying, "I'm here to spend time with you."

What is the most appropriate intervention for the nurse to implement after finding a disturbed client in bed in the fetal position?

Sitting down in a chair by the client and saying, "I'm here to spend time with you." "I'm here to spend time with you" accepts the client at the client's current level and allows the client to set the pace of the relationship. Touching the client may be misinterpreted and may precipitate an aggressive response.

A client who has been hospitalized with schizophrenia tells the nurse, "My heart has stopped and my veins have turned to glass!" What should the nurse conclude that the client is experiencing?

Somatic delusion

As a nurse is assisting a client with the diagnosis of schizophrenia with morning care, the client suddenly throws off the covers and starts shouting, "My body is disintegrating! I'm being pinched." What term best describes the client's behavior? Somatic delusion Paranoid ideation Loose association Ieas of referenc

Somatic delusion A somatic delusion is a false feeling about the physical self that is caused by a loss of reality testing. Paranoid ideations are beliefs that the individual is being singled out for unfair treatment. Loose associations are verbalizations that are difficult to understand because the links between thoughts are not apparent. Ideas of reference are false beliefs that the words and actions of others are concerned with or are directed toward the individual.

A client with schizophrenia is taking benztropine (Cogentin) in conjunction with an antipsychotic. The client tells a nurse, "Sometimes I forget to take the Cogentin." What should the nurse teach the client to do if this happens again? Take 2 pills at the next regularly scheduled dose. Notify the health care provider about the missed dose immediately. Take a dose as soon as possible, up to 2 hours before the next dose. Skip the dose, then take the next regularly scheduled dose 2 hours early.

Take a dose as soon as possible, up to 2 hours before the next dose. Taking a dose as soon as possible is the advised intervention when a dose is missed; interruption of the medication may precipitate signs of withdrawal such as anxiety and tachycardia. Taking 2 pills at the next regularly scheduled dose will provide an excessive amount of the medication at one time. Notifying the health care provider about the missed dose immediately is unnecessary. Skipping a dose is not advised if the next regularly scheduled dose is due within 2 hour

The nurse is working with a 15-year-old girl and her parents on a treatment plan for her diagnosis of attention-deficit/hyperactivity disorder (ADHD). The nurse should be sure to:

Teach the parents how to structure and enforce limits on their daughters behavior that are appropriate to her condition.

During a well-baby visit, the parents complain that their 2-year-old daughter soils herself because she is lazy. The parents plan to make her wear her soiled clothing to teach her a lesson. The nurse is concerned about the potential for child neglect and abuse. Which nursing intervention will be most therapeutic at this time?

Teaching the parents developmental milestones in relation to acceptable discipline methods

Schizophrenia

The core disturbance lies in lack of contact with reality and the child's retreat into his or her own world.

The major mental health problems of childhood are grouped into seven categories.

The diagnosis is less important than the person.

Define Awssessment (pg.94)

The gathering, verifying and communicating of information relative to the client.

Operational defiant disorder

Treatment includes family therapy that stresses limit setting and consistency.

A nurse notes that a client in the detoxification unit is exhibiting early signs of alcohol withdrawal. What clinical manifestations might the nurse have noticed? (Select all that apply.)

Tremors Anorexia

D

Which infection is identified by evaluating the vaginal specimen of an adolescent client who sustained a sexual assault? A: Syphilis B: Chlamydia C: Hepatitis B D: Trichomoniasis

What clinical findings may be expected when a nurse cares for an individual with an anxiety disorder? (Select all that apply.)

Worrying about a variety of issues Converting the anxiety into a physical symptom Displacing the anxiety onto a less threatening object Demonstrating behavior common to an earlier stage of development

For which adverse effect should the nurse continually observe a client who is receiving valproic acid (Depakene)?

Yellow sclerae

feeding disorder

a child's "persistent failure to eat adequately" for a period of at least 1 month, which results in a significant loss of weight or a failure to gain weight

Autism

a disorder of communication social interactions and behavior

Schizophrenia

a psychological disorder characterized by delusions, hallucinations, disorganized speech, and/or diminished, inappropriate emotional expression

anxiety

a vague,uneasy feeling that occurs in response to a threat.

colic

severe abdominal pain

Jumbled set of words that have no connection or relationship to each other. Example: hot happies are spying on me but no men love short feet"

word salad

Random access memory, memory to which one refers. Increases through childhood and peaks during early adulthood, strengthens with use through connecting neurons that occurs with learning.

working memory

Two days after admission to the detoxification program, a client with a long history of alcohol abuse tells the nurse, "I don't know why I came here." What is the most therapeutic response by the nurse? -"You feel that you don't need this program?" -"You realize that you are trying to avoid your problem?" -"I thought that you admitted yourself into the program." -"Don't you remember why you decided to come here in the first place?"

you feel that you don't need this program?

A 67-year-old man with type 2 diabetes sadly confides in the nurse that he has been unable to have an erection for several years. What is the best response by the nurse? - "At your age sex isn't that important." -"Sex isn't everything it's cracked up to be." -"You sound upset about not being able to have an erection." -"Maybe it's time for you to speak to your primary healthcare provider about this."

you sound upset about not being able to have an erection

A client who is dying jokes about the situation even though the client is becoming sicker and weaker. Which is the most therapeutic response by the nurse?

"Does it help to joke about your illness?"

A 2½-year-old child is admitted for treatment of injuries supposedly sustained in a fall down a flight of stairs. Child abuse is suspected. What statements might the nurse expect from a parent who engages in child abuse?

"Every time I turn around the kid is falling over something." "I can't understand it. He didn't have a problem using the stairs without my help before this." Abusive parents often have a poor understanding of the expected growth and development of children and tend to blame the child. Toddlers generally need supervision and some assistance when climbing stairs, but abusive parents have little understanding of toddlers' abilities

The parents of an adolescent girl are upset about their daughter's diagnosis of anorexia nervosa and the treatment plan that has been proposed. What is the best response by the nurse when the client's parents ask to bring food in for the client?

"For now, let the staff handle her food needs."

A nurse uses the CAGE screening test for alcoholism to determine an individual's potential for a drinking problem. What is one of the four questions included on this test?

"Have you ever felt bad or guilty about your drinking?"

A young client with schizophrenia says, "I'm starting to hear voices." What is the nurse's most therapeutic response? "How do you feel about the voices, and what do they mean to you?" "You're the only one hearing the voices. Are you sure you hear them?" "The health team members will observe your behavior. We won't leave you alone." "I understand that you're hearing voices talking to you and that the voices are very real to you. What are the voices saying to you?

"I understand that you're hearing voices talking to you and that the voices are very real to you. What are the voices saying to you? Acknowledging that client is hearing voices talking to him and that the voices are very real to him validates the presence of the client's hallucinations without agreeing with them, which communicates acceptance and can form a foundation for trust; it may help the client return to reality. The nurse also needs to assess the content of the voices to determine the risk of self injury or violence against others. The client's contact with reality is too tenuous to explore what they mean. Saying that the client is the only one hearing the voices and asking whether he is sure that he is hearing demeans the client, which blocks the development of a trusting relationship and future communication. Telling the client that the health team members will observe his behavior and that he won't be left alone is condescending and may impair future communication.

A parent who is visiting a hospitalized adolescent gets into an argument with the adolescent. Leaving the adolescent's room in tears, the parent meets the nurse and relates the argument, saying, "I can't believe I got so angry that I could have hit her." What is the most therapeutic response by the nurse?

"Sometimes we find it difficult to live up to our own expectations of ourselves." The response "Sometimes we find it difficult to live up to our own expectations of ourselves" is the best response because it reflects the feelings being expressed at this time. "Teenagers really can drive you to distraction" avoids the real issue. Telling the parent to bring a surprise for the adolescent on the next visit does not address the real concern; the parent's argument may have been justified, and the child's behavior should not be rewarded. The response "You can't compare yourself to an abusive parent—after all, you didn't beat your child" avoids the issue; the parent may fear that next time control will be lost and abuse will occur.

A client who has been sexually abused tearfully says, "I'm no good now; there's nothing to live for." What is the most therapeutic response by the nurse?

"Tell me more about your feelings." The response "Tell me more about your feelings" is on a feeling level and therefore encourages the exploration of feelings. The statement "I can understand why you feel worthless" supports the negative feelings of worthlessness. The response "Why do you feel that there's nothing to live for?" focuses on negative feelings; "why" questions are difficult and sometimes impossible to answer. The question "Do you feel this way because of what has happened?" will elicit a yes or no response and will not encourage the exploration of feelings.

A 13-year-old who recently was suspended from school for consistently bullying other children is brought to the pediatric mental health clinic by his mother. The child is assessed by the psychiatrist and referred to a psychologist for psychologic testing. The day after the tests are completed, the mother returns to the clinic and asks the nurse for results of the tests. Which is the best response by the nurse? 1 Refer the mother to the psychiatrist. 2 Explain to the mother the results of the tests. 3 Suggest that the mother call the psychologist. 4 Teach the mother about the tests that were administered.

1

A 34-year-old woman who was sexually assaulted is examined in the emergency department within 2 hours of the assault. During assessment she freely discusses the incident, her past psychiatric history, and her past sexual history with the sexual assault nurse examiner (SANE). Which information documented by the nurse indicates that the nurse needs more teaching about appropriate charting? 1 Details of the client's sexual history 2 Verbatim statements made by the client regarding the assault 3 Details of the client's physical trauma documented with a body map or photographs 4 Signs of emotional trauma, including the client's current condition and cooperative behavior

1

A nonviolent client on the psychiatric unit suddenly refuses to take the prescribed antipsychotic medication. What should the nurse do? 1 Honor the client's decision and document the behavior and all interventions. 2 Use an authoritarian approach to induce the client to take the prescribed medication. 3 Call the primary healthcare provider and request that the client be discharged against medical advice. 4 Start proceedings to have the client declared incompetent and seek a court order permitting medication.

1

A nurse in charge in the surgical intensive care unit notes that a number of clients do not seem to be responding to morphine that was administered for pain. Later in the evening the nurse finds a staff nurse dozing in the nurses' lounge. When awakened, the staff nurse appears uncoordinated and drugged, with slurred speech. What should the nurse in charge do? 1 Ask the nurse manager to be present before confronting the staff nurse. 2 Ask other staff members whether they have noticed anything unusual lately. 3 Tell the staff nurse that everyone now knows who has been stealing the morphine. 4 Arrange to secretly observe the staff nurse the next time the staff nurse administers morphine.

1

A registered nurse is educating a nursing student about abortion-related issues. Which statement provided by the nursing student post-teaching needs correction? 1 "If a woman is in her first trimester, she may end her pregnancy according to state regulations." 2 "In the third trimester when the fetus becomes viable, the state's interest is to protect the fetus." 3 "If the fetus is over 28 weeks old, the state requires viability tests before conducting abortions." 4 "In the second trimester, the state enforces regulation regarding the person performing the abortion and the abortion facility."

1

An 18-year-old woman is brought to the emergency department by her two roommates after being found unconscious in the bathroom. Laboratory tests are ordered. The nurse reviewing the findings notes that the urinalysis is positive for flunitrazepam. The nurse knows that flunitrazepam is often used for what? 1 As a date rape drug 2 To control symptoms of psychosis 3 To control symptoms of bipolar mania 4 To treat hangover symptoms after excessive alcohol consumption

1

An older client is treated in the emergency department for soft-tissue injuries that the medical team suspects might be caused by physical abuse. An adult child states that the client is forgetful and confused and falls all the time. A mini-mental examination indicates that the client is oriented to person, place, and time, and the client does not comment when asked directly how the bruises and abrasions occurred. What is the next appropriate nursing action? 1 Interview the client without the presence of family members. 2 Report the abuse to the appropriate state agency for investigation. 3 Accept the adult child's explanation until more data can be collected. 4 Refer the client's clinical record to the hospital ethics committee for review.

1

What professional responsibility does the nurse display as a client's advocate? 1 The nurse protects the client's human and legal rights and provides assistance in asserting said rights. 2 The nurse actively collaborates with other healthcare professionals to follow the best treatment plan for a client. 3 The nurse explains concepts and facts about health, describes the reason for routine care activities, and demonstrates procedures. 4 The nurse establishes an environment for collaborative client-centered care to provide safe, quality care with positive client outcomes.

1

A client who is to undergo a mastectomy for cancer of the breast tells the nurse that she is worried about what she will look like after the surgery. What is the most appropriate initial response by the nurse? 1 "I understand that you'd be concerned." 2 "Try not to think about the surgery now." 3 "Everyone having this surgery feels the same way." 4 "Perhaps you should discuss this with your husband."

1 "I understand that you'd be concerned." Women facing breast surgery often have feelings relating to their sexuality and change in body image; the nurse plays a vital role in helping the client verbalize feelings, and this response keeps channels of communication open. The client's concerns are real, and advising not to think about the surgery will only block further communication. The response "Everyone having this surgery feels the same way" does not focus on the importance of the client as an individual; each person feels differently. The response "Perhaps you should discuss this with your husband" can be interpreted as the nurse's reluctance to listen; the client may not be able to talk with the husband about this.

A nurse is preparing an intramuscular injection to be administered to a 2-year-old child. What approach is the most therapeutic? Correct1 "This might hurt, but it's important that you be very still." 2 "You're afraid of getting a shot because you think it will hurt." 3 "Don't worry—Daddy and Mommy will be back after it's over." 4 "Act like a big child and we can get this done as quickly as possible."

1 "This might hurt, but it's important that you be very still." Telling the child that the shot will hurt but that it's important for the child to remain still is an honest statement that tells the toddler what to expect and expresses the nurse's expectation that the toddler will stay still during the procedure. Emphasizing the child's fears will exacerbate these fears. The parents should be encouraged to stay with the child to provide comfort. Asking the child to be brave puts unrealistic expectations on the child.

A nurse is preparing an intramuscular injection to be administered to a 2-year-old child. What approach is the most therapeutic? 1 "This might hurt, but it's important that you be very still." 2 "You're afraid of getting a shot because you think it will hurt." 3 "Don't worry—Daddy and Mommy will be back after it's over." 4 "Act like a big child and we can get this done as quickly as possible."

1 "This might hurt, but it's important that you be very still." Telling the child that the shot will hurt but that it's important for the child to remain still is an honest statement that tells the toddler what to expect and expresses the nurse's expectation that the toddler will stay still during the procedure. Emphasizing the child's fears will exacerbate these fears. The parents should be encouraged to stay with the child to provide comfort. Asking the child to be brave puts unrealistic expectations on the child.

A nurse is planning to teach a client about self-care. What level of anxiety will best enhance the client's learning abilities? 1 Mild 2 Panic 3 Severe 4 Moderate

1 Mild Mild anxiety motivates one to action, such as learning or making changes. Higher levels of anxiety tend to blur the individual's perceptions and interfere with functioning. Attention is severely reduced by panic. The perceptual field is greatly reduced with severe anxiety and narrowed with moderate anxiety.

A client with a 20-year history of excessive alcohol use is admitted to the hospital with jaundice and ascites. A priority nursing action during the first 48 hours after the client's admission is to: 1 Monitor the client's vital signs 2 Increase the client's fluid intake 3 Improve the client's nutritional status 4 Determine the client's reasons for drinking

1 Monitor the client's vital signs A client's vital signs, especially the pulse and temperature, will increase before the client demonstrates any of the more severe symptoms of withdrawal from alcohol. Increasing intake is contraindicated initially because it may cause cerebral edema. Improving nutritional status becomes a priority after the problems of the withdrawal period have subsided. Determining the client's reasons for drinking is not a priority until after the detoxification process.

The nurse should suspect that a client who had a recent myocardial infarction is experiencing denial when the client: 1. Attempts to minimize the illness 2. Lacks an emotional response to the illness 3. Refuses to discuss the condition with the client's spouse 4. Expresses displeasure with the prescribed activity program

1. Attempts to minimize the illness

A client with rheumatoid arthritis does not want the prescribed cortisone and informs the nurse. Later, when the nurse attempts to administer cortisone, the client asks what the medication is and the nurse gives an evasive answer. The client takes the medication and later discovers that it was cortisone. The client states an intent to sue. What factors in this situation must be considered in a legal action? Select all that apply. 1 Clients have a right to refuse treatment. 2 Nurses are required to answer clients truthfully. 3 The healthcare provider should have been notified. 4 The client had insufficient knowledge to make such a decision. 5 Legally prescribed medications are administered despite a client's objections.

1,2,3

A victim of an automobile crash is brought to the hospital. The nurse notes that the client is in need of cardiopulmonary resuscitation (CPR). What factors should the nurse remember before performing CPR? Select all that apply. 1 Cardiopulmonary resuscitation is an emergency treatment that is provided without a client's consent. 2 Cardiopulmonary resuscitation is not performed on adult clients who have already consented to a do not resuscitate order either verbally or in writing. 3 Cardiopulmonary resuscitation is not to be performed by a primary healthcare provider in violation of a do not resuscitate order under any circumstances. 4 Cardiopulmonary resuscitation is performed on appropriate clients unless a do not resuscitate order has been signed and made part of the client's record. 5 Cardiopulmonary resuscitation is generally performed on any client who requires resuscitation in an emergency, but the client's consent is required.

1,2,4

A parent of four is remanded to the psychiatric unit by the court for observation. The client was arrested and charged with abusing a 2-year-old child, who is in the pediatric intensive care unit in critical condition. The nurse approaches the client for the first time. How should the nurse anticipate that the client will likely respond? Select all that apply. 1 By denying beating the child 2 By avoiding talking about the situation 3 By asking where the other three children are 4 By expressing excessive concern for the child 5 By exhibiting an emotional response that is inconsistent with the degree of injury

1,2,5

A student nurse is listing different scenarios that comply with basic healthcare ethics. Which scenarios mentioned by the student nurse relate to the healthcare ethic of fidelity? Select all that apply. 1 "A nurse monitors a client after providing nonpharmacological measures to relieve anxiety due to hospitalization." 2 "A nurse notes that the pain relief measures provided to that client have been ineffective. The nurse formulates a different plan of care." 3 "A nurse ensures that the client understands the risks and benefits of an experimental treatment before signing the appropriate consent form." 4 "A nurse carefully evaluates the advantages and disadvantages of the client's plan of care to ensure that the risks do not outweigh the benefits." 5 "A nurse is caring for a client who refuses to be touched by people of certain skin color. The nurse continues providing care since other colleagues refuse to attend to the client."

1,2,5

The nurse is working with a client who talks freely about feeling depressed. During the interaction the client states, "Things will never change." What findings support the nurse's conclusion that the client is experiencing hopelessness? (Select all that apply.) 1. Bouts of crying 2. Self-destructive acts 3. Presence of delusions 4. Feelings of worthlessness 5. Intense interpersonal relationships

1. Bouts of crying 2. Self-destructive acts 4. Feelings of worthlessness

A parent of four is remanded to the psychiatric unit by the court for observation. The client was arrested and charged with abusing a 2-year-old son, who is in the pediatric intensive care unit in critical condition. The nurse approaches the client for the first time. How should the nurse anticipate that the client will likely respond? (Select all that apply.) 1. By denying beating the son 2. By avoiding talking about the situation 3. By expressing excessive concern for the son 4. By asking where the other three children are 5. Emotional response is inconsistent with degree of injury

1. By denying beating the son 2. By avoiding talking about the situation 5. Emotional response is inconsistent with degree of injury

A child in the first grade is murdered, and counseling is planned for the other children in the school. What should a nurse identify first before evaluating a child's response to a crisis? 1. Developmental level of the child 2. Quality of the child's peer relationships 3. Child's perception of the crisis situation 4. Child's communication patterns with family members

1. Developmental level of the child **Rationale: Knowledge of the developmental level is essential to understanding a child's response to a crisis situation; the variety of coping abilities usually increases as the child progresses through the stages of growth and development.

The nurse explains to a nursing assistant that behavior usually is viewed and accepted as normal if it: 1. Fits within standards accepted by one's society 2. Helps the person reduce the need for coping skills 3. Expresses the individual's feelings and thoughts accurately 4. Allows achievement of short-term and long-term goals by the individual

1. Fits within standards accepted by one's society

A nurse is teaching clients about dietary restrictions during monoamine oxidase inhibitor (MAOI) therapy. What response does the nurse tell them to anticipate if they do not follow these restrictions? 1. Occipital headaches 2. Generalized urticaria 3. Severe muscle spasms 4. Sudden drop in blood pressure

1. Occipital headaches

Among members of the nursing team, which functions are registered nurses legally permitted to perform in a mental health hospital? Select all that apply. 1.Psychotherapy 2.Health promotion 3.Case management 4.Prescribing medication 5.Treating human responses

2.Health promotion 3.Case management 5.Treating human responses

What should a nurse ensure when creating an environment that is conducive to psychological safety? 1. Realistic limits are set. 2. Passive acceptance is promoted. 3. The client's physical needs are met. 4. The physical environment is kept in order

1. Realistic limits are set.

A 17-year-old client is admitted to the hospital because of weight loss and malnutrition, and the health care provider diagnoses anorexia nervosa. After the client's physical condition is stabilized, the provider, in conjunction with the client and parents, decides to institute a behavior-modification program. What component of behavior modification verbalized by one of the parents leads the nurse to conclude that the parent has an understanding of the therapy? 1. Rewarding positive behavior 2. Deconditioning fear of weight gain 3. Decreasing unnecessary restrictions 4. Reducing anxiety-producing situations

1. Rewarding positive behavior

Antipsychotic drugs can cause extrapyramidal side effects. Which responses should the nurse document as indicating pseudoparkinsonism? (Select all that apply.) 1. Rigidity 2. Tremors 3. Mydriasis 4. Photophobia 5. Bradykinesia

1. Rigidity 2. Tremors 5. Bradykinesia

The nurse interviews a young female client with anorexia nervosa to obtain information for the nursing history. What will the client's history most likely reveal? (Select all that apply.) 1. Ritualistic behaviors 2. Desire to improve her self-image 3. Supportive mother-daughter relationship 4. Low achievement in school and little concern for grades 5. Satisfaction with and a desire to maintain her current weight

1. Ritualistic behaviors 2. Desire to improve her self-image

What is the basic therapeutic tool used by the nurse to foster a client's psychological coping? 1. Self 2. Milieu 3. Helping process 4. Client's intellect

1. Self

A client with a diagnosis of borderline personality disorder (BPD) has negative feelings toward the other clients on the unit and considers them all "bad." Which defense is the client using when identifying the other clients thusly? 1. Splitting 2. Ambivalence 3. Passive aggression 4. Reaction formation

1. Splitting

Amitriptyline (Elavil) is an antidepressant medication used to treat anxiety disorders. Which class of antidepressant medications does it belong to? 1. Tricyclics 2. Monoamine oxidase inhibitors (MAOIs) 3. Selective serotonin reuptake inhibitors (SSRIs) 4. Serotonin-norepinephrine reuptake inhibitors (SNRIs)

1. Tricyclics

The nurse is visiting the home of a patient with a mental health disorder. What should the nurse ensure in during this home visit?SELECT ALL THAT APPLY 1. status of suicide ideation 2. ability to provide self-care safely 3. patients risk of falling in the home 4. number of family members who visit 5. compliance with prescribed medications

1. status of suicide ideation 2. ability to provide self-care safely 3. patients risk of falling in the home 5. compliance with prescribed medications

The nurse assist with admission of a patient to the hospital with pancreatitis and a history of alcohol abuse. Why should the nurse observe the patient for agitation , tremors, and hallucinations? 1. these are symptoms of alcohol withdraw 2. these symptoms indicate possible cirrhosis of the liver 3. the patient may be using alcohol in the hospital setting 4. patients with a history of alcohol abuse are at risk for mental illness

1. these are symptoms of alcohol withdraw

A nurse is interviewing a mother accused of physical child abuse. When speaking with this mother, what does the nurse expect her to do? 1 Attempt to rationally explain her behavior. 2 Reveal the belief that her child needed to be disciplined. 3 Offer a detailed explanation of how her child was injured. 4 Ask how she can arrange to visit her child on the pediatric unit

2

A client who appears dejected, barely responds to questions, and walks very slowly about the mental health unit tells the nurse in a barely audible voice that life is no longer worth living. What is the most therapeutic response to this statement by the nurse? 1."Have you been thinking about suicide?" 2."What could be so bad to make you feel that way?" 3."We'll talk about your feelings after you've rested." 4."Let's talk about something pleasant to make you feel better."

1."Have you been thinking about suicide?"

A nurse is caring for a man who has inoperable cancer of the pancreas. His wife is trying to cope with the diagnosis. Place the wife's statements in order as the woman progresses through the grieving process, from the first stage to the last: 1."He shouldn't have gotten this because he doesn't smoke or drink." 2."I want him to get a second opinion." 3."All I do is cry, because I can't live without him." 4."If he can't be cured, I just want him to be comfortable." 5."His grandchildren need to get to know him."

1."I want him to get a second opinion." 2."He shouldn't have gotten this because he doesn't smoke or drink." 3."His grandchildren need to get to know him." 4."All I do is cry, because I can't live without him." 5."If he can't be cured, I just want him to be comfortable." Tip: Denial,Anger,Bargaining,Depression,Acceptance

A client on the psychiatric unit sits alone most of the day. The nurse approaches the client. As the nurse gets approximately 3 feet away, the client lets out a string of profanity and shouts, "Leave me alone; I don't want to talk to you!" What is the most appropriate response by the nurse? 1."I'll leave for now, but I'll be back later." 2."Why do you feel the need to greet me like that?" 3."Don't talk to me like that—I'm here to spend time with you." 4."I don't like it when you talk like that—are you trying to push me away?"

1."I'll leave for now, but I'll be back later."

A client has been taking escitalopram (Lexapro) for treatment of a major depressive episode. On the fifth day of therapy the client refuses the medication, stating, "It doesn't help, so what's the use of taking it?" What is the best response by the nurse? 1."It can take 1 to 4 weeks to see an improvement." 2."It takes 6 to 8 weeks for this medication to have an effect." 3."I'll talk to your doctor about increasing the dosage. That may help." 4."You should have felt a response by now. I'll notify the doctor right now."

1."It can take 1 to 4 weeks to see an improvement."

A client who has recently been found to be infected with HIV comments to the nurse, "There are so many terrible people around. Why couldn't one of them get HIV instead of me?" What is the best response by the nurse? 1."It seems unfair that you should have this disease." 2."I'm sure you really don't wish this on someone else." 3."It might be good for you to speak with your religious leader." 4."I'm sure you know that HIV infection is now considered a chronic illness."

1."It seems unfair that you should have this disease."

A client who had to be cut out of a car after a motor vehicle collision has no visible physical effects from the ordeal. The client responds to the emergency department nurse's questions factually in a soft voice with a composed manner. This behavior may indicate that the client: 1.)Is controlling the expression of feelings 2.)Has repressed the details of the accident 3.)Has blocked out the events of the last few hours 4.)Is experiencing the reorganization phase of the trauma experience

1.) Is controlling the expression of feelings

A resident in a nursing home recently immigrated to the United States from Italy. How does the nurse plan to provide emotional support? 1.)By offering choices consistent with the client's heritage 2.)By ensuring that the client understands American beliefs 3.) By assisting the client in adjusting to the American culture 4.)By correcting the client's misconceptions about appropriate health practices

1.) by offering choices consistent with the client's heritage

Which approaches should a nurse use during crisis intervention? Select all that apply. 1.Active 2.Passive 3.Reflective 4.Interpretative 5.Goal-directed

1.Active 5.Goal-directed 1. The nurse should assume an active role in assessing the current situation and facilitate the interview with authority. 5. During crisis intervention the nurse should be goal-directed to assist the client with coping with the crisis

The nurse should suspect that a client who had a recent myocardial infarction is experiencing denial when the client: 1.Attempts to minimize the illness 2.Lacks an emotional response to the illness 3.Refuses to discuss the condition with the client's spouse 4.Expresses displeasure with the prescribed activity program

1.Attempts to minimize the illness

A child in the first grade is murdered, and counseling is planned for the other children in the school. What should a nurse identify first before evaluating a child's response to a crisis? 1.Developmental level of the child 2.Quality of the child's peer relationships 3.Child's perception of the crisis situation 4.Child's communication patterns with family members

1.Developmental level of the child

A nurse greets a client who had been experiencing delusions of persecution and auditory hallucinations by saying, "Good evening. How are you?" The client, who has been referring to himself as "man," answers, "The man is bad." Of what is this an example? 1.Dissociation 2.Transference 3.Displacement 4.Identification

1.Dissociation rational 1. Talking in the third person reflects poor ego boundaries and a dissociation from the real self.

A practitioner prescribes Alprazolam (Xanax) 0.25 mg by mouth three times a day for a client with anxiety and physical symptoms related to work pressures. For what most common side effect of this drug should the nurse monitor the client? 1.Drowsiness 2.Bradycardia 3.Agranulocytosis 4.Tardive dyskinesia

1.Drowsiness

In an attempt to remain objective and support a client during a crisis, the nurse uses imagination and determination to project the self into the client's emotions. This technique is known as: 1.Empathy 2.Sympathy 3.Projection 4.Acceptance

1.Empathy

A disturbed client who has been out of touch with reality has been hospitalized for several weeks. One day the nurse notes that the client's hair is dirty and asks whether the client wants to wash it. The client answers, "Yes, and I'd like to shower and change my clothes, too." What can the nurse conclude about the client in relation to this response? 1.He has some feelings of self-worth. 2.He is open to suggestions from others. 3.He may be entering a hyperactive phase. 4.He has a need for social reassurance from others

1.He has some feelings of self-worth.

An adult who has been in a gay relationship for 3 years arrives at the emergency department in a near panic state. The client states, "My partner just left me. I am a wreck." What should the nurse do to help the client cope with this loss? Select all that apply. 1.Identify the client's support systems. 2.Explore the client's psychotic thoughts. 3.Reinforce the client's current self-image. 4.Encourage the client to talk about the situation. 5.Suggest that the client explore personal sexual attitudes.

1.Identify the client's support systems. 4.Encourage the client to talk about the situation. A client in crisis needs to rely on available support systems for assistance; therefore, it is vital for the nurse to identify the client's support system. Talking about the situation helps the individual to put the crisis in perspective.

The nurse anticipates that the medication that will be used to prevent symptoms of withdrawal in clients with a long history of alcohol abuse is: 1.Lorazepam (Ativan) 2.Phenobarbital (Luminal) 3.Chlorpromazine (Thorazine) 4.Methadone hydrochloride (Methadone)

1.Lorazepam (Ativan)

A client is receiving doxepin (Sinequan). For which most dangerous side effect of tricyclic antidepressants should a nurse monitor the client? 1.Mydriasis 2.Dry mouth 3.Constipation 4.Urine retention

1.Mydriasis

A woman who was sexually assaulted by a stranger in the elevator of her apartment building is brought by her husband to the emergency department. What is the priority nursing intervention? 1.Obtaining information about her perception of the incident 2.Notifying legal authorities that a sexual assault has occurred 3.Talking with the husband about his feelings concerning sexual assault 4.Teaching the client how to obtain a midstream clean-catch urine specimen

1.Obtaining information about her perception of the incident

A young client who has become a mother for the first time is anxious about her new parenting role. With the nurse's encouragement, she joins the new mothers' support group at the local YMCA. What kind of prevention does this activity reflect? 1.Primary prevention 2.Tertiary prevention 3.Secondary prevention 4.Therapeutic prevention

1.Primary prevention

A psychiatric client recently admitted to the inpatient unit has a history of angry outbursts. The client's anger appears to be escalating, although the client still appears to be in control. What should the nurse do first to prevent an incident from developing? 1.Set a contract with the client to verbalize frustrations before acting out 2.Establish firm control and use seclusion before the client acts out impulsively 3.Show the client the seclusion room as a method of deterring acting-out behavior 4.Call the health care provider for a prescription for an intramuscular sedative for the client

1.Set a contract with the client to verbalize frustrations before acting out

A client with a diagnosis of borderline personality disorder has negative feelings toward the other clients on the unit and considers them all to be "bad." Which defense was the client using when this statement was made? 1.Splitting 2.Ambivalence 3.Passive aggression 4.Reaction formation

1.Splitting Rationale: 1. Splitting is the compartmentalization of opposite-affect states and failure to integrate the positive and negative aspects of self or others.

In which situation is the use of seclusion contraindicated? 1.The client has expressed severe suicidal thoughts. 2.The client appears to want to be placed in seclusion. 3.The client has been voluntarily admitted for treatment. 4.The client had minimal improvement despite being secluded before

1.The client has expressed severe suicidal thoughts.

A recently hired nurse is caring for several clients on a mental health unit at a local community hospital. The nurse manager is evaluating the nurse's performance. What situation indicates that the nurse-client boundaries of the recently hired nurse are appropriate? 1.The nurse shares with the entire treatment team vital information the client disclosed in a private session. 2.The nurse is often busy doing other tasks when the client and nurse are scheduled for a counseling session. 3.A client enters the therapeutic group late with the nurse's permission even though group rules say that this is not allowed. 4.A client's overall behavior is significantly more independent and demonstrates higher function on the days that the nurse is not working

1.The nurse shares with the entire treatment team vital information the client disclosed in a private session.

Amitriptyline (Elavil) is an antidepressant medication used to treat anxiety disorders. Which class of antidepressant medications does it belong to? 1.Tricyclics 2.Monoamine oxidase inhibitors (MAOIs) 3.Selective serotonin reuptake inhibitors (SSRIs) 4.Serotonin-norepinephrine reuptake inhibitors (SNRIs)

1.Tricyclics

According to Erikson, a child's increased vulnerability to anxiety in response to separation or pending separation from significant others results from failure to complete a developmental stage. What does the nurse call this stage? 1.Trust 2.Identity 3.Initiative 4.Autonomy

1.Trust

A person mowing a lawn is badly disfigured by the lawnmower blade. According to Erikson's theory, which age at the time of injury will be associated with the greatest risk of long-term psychological effects?

11 years An 11-year-old child is generally in Erikson's stage of industry versus inferiority, which involves the mastery of skills; unfortunately, the child did not master the skill of lawnmowing.

A depressed client has been taking Paroxetine (Paxil) 20 mg by mouth once a day for 4 weeks. The practitioner concludes that there is no clinical improvement in the client's condition and increases the daily dose to 30 mg. The medication is supplied in an oral suspension of 10 mg/5 mL. How many milliliters of Paxil solution should the nurse instruct the client to take? Record your answer as a whole number. ___ mL

15mL

A hospitalized psychiatric client with the diagnosis of histrionic personality disorder demands a sleeping pill before going to bed. After being refused the sleeping pill, the client throws a book at the nurse. What does the nurse recognize this behavior to be? 1 Exploitive 2 Acting out 3 Manipulative 4 Reaction formation

2

A nurse in the emergency department notes large welts and scars on the back of a toddler who has been admitted for an asthma attack. What additional information must be included in the nurse's assessment? 1 History of an injury 2 Signs of child abuse 3 Presence of food allergies 4 Recent recovery from chickenpox

2

A nurse's coworker approaches the nurse to inquire about the test results of a friend who is being cared for by the nurse. How should the nurse respond? 1 Answer the questions softly so other people will not hear 2 Decline to discuss the friend's medical condition. 3 Give the coworker the name of the client's primary healthcare provider, so the coworker can contact the provider instead. 4 To provide reassurance, tell the coworker of the friend's test results that are within normal limits.

2

Child maltreatment is suspected in a 3-year-old girl admitted to the hospital with many poorly explained injuries. Which statement by the mother further supports this suspicion? 1 "When I get angry, I take her for a walk." 2 "I have no problems with any of my other children." 3 "When she misbehaves, I send her to her room alone." 4 "I make her stand in the corner when she doesn't eat her dinner."

2

Nurses on a psychiatric unit have secluded a client who has the diagnosis of bipolar I disorder, manic episode, and who has been losing control and throwing objects while in the dayroom. What is the most important intervention for a client who is given an as-needed medication and confined to involuntary seclusion? 1 Continue intensive nursing interactions. 2 Evaluate the client's progress toward self-control. 3 Determine whether any staff member has been injured. 4 Observe the client for side effects of the medication given

2

The leader is devising a strategy to connect individuals, groups, and organizations to attune them toward a common goal. Which political strategy is the leader pursuing? 1 Negotiating 2 Building coalitions 3 Adopting an empowering attitude 4 Fostering collegiality and collaboration

2

To therapeutically relate to parents who are known to have maltreated their child, what must the nurse do first? 1 Develop a trusting relationship with the child. 2 Identify personal feelings about child abusers. 3 Recognize the emotional needs of the parents. 4 Gather information about the child's home environment

2

Two 14-year-old girls are best friends and always eat lunch together at school. One of the girls eats rapidly and then immediately leaves to go to the girls' restroom. After a week or so the other girl begins to suspect that her friend is using self-induced vomiting to keep her weight down. Because the friend is not sure what to do, she speaks with a relative who is a nurse. What should the nurse encourage her to do? 1 Confront her friend with her suspicions. 2 Talk to the school nurse about her concerns. 3 Inform the girl's mother about her daughter's behavior. 4 Watch a while longer before doing anything that might ruin the friendship

2

What is the nurse's priority responsibility when abuse of an 8-year-old child is suspected? 1 Treating the child's traumatic injuries 2 Protecting the child from future abuse 3 Confirming the child's suspected abuse 4 Having the child examined by the healthcare provider

2

A client is admitted to the psychiatric unit of the hospital with a diagnosis of conversion disorder. The client is unable to move either leg. Which finding should the nurse consider consistent with this diagnosis? 1 Feeling depressed 2 Appearing composed 3 Demonstrating free-floating anxiety 4 Exhibiting tension when discussing symptoms

2 Appearing composed

A nurse who is caring for a client after head and neck surgery is concerned with the client's anger and depressive episodes about the effects of surgery. The action that indicates the client is reaching acceptance is: 1 Smiling and becoming more extroverted 2 Performing self-care of the tracheal stoma 3 Ambulating in the hall and sitting in the lounge 4 Allowing a family member to participate in care

2 Performing self-care of the tracheal stoma The best indicator of acceptance is when the client begins to participate in self-care. Smiling and becoming more extroverted does not indicate acceptance and may be an act of pretended courage. Ambulating in the hall and sitting in the lounge does not indicate acceptance and may be an attempt to relieve boredom. Allowing a family member to participate in care does not indicate acceptance and may indicate dependence.

After a subtotal gastrectomy a client is returned to the surgical unit. Which is the best nursing action to prevent pulmonary complications? 1 Ambulating the client to increase respiratory exchange Correct2 Promoting frequent turning and deep breathing to mobilize secretions 3 Maintaining a consistent oxygen flow rate to increase oxygen saturation 4 Keeping a plastic airway in place to ensure patency of the client's airway

2 Promoting frequent turning and deep breathing to mobilize secretions To promote drainage of different lung regions, clients should turn every two hours. Deep breathing inflates the alveoli and promotes fluid drainage. During physical effort, individuals with abdominal incisions often revert to shallow breathing. Oxygen administration is a dependent function and generally is not required unless there is underlying cardiac or respiratory disease. The airway is expelled when the gag reflex returns.

Which nursing intervention is indicated for a client with an anxiety disorder? 1 Encouraging suppression of anger by the client 2 Promoting verbalization of feelings by the client 3 Limiting involvement of the client's family during the acute phase 4 Explaining why the client should accept the psychological factors that are precipitating the anxiety

2 Promoting verbalization of feelings by the client Freedom to express feelings serves as a safety valve to reduce anxiety. Suppression of anger or hostility may add to the client's anxiety. Limiting involvement of the client's family during the acute phase may or may not be helpful; the client's family members may provide support. Explaining why the client should accept the psychological factors that are precipitating the anxiety is not therapeutic; accepting current situational stresses may not be possible.

During initial assessment of mental health status, the nurse asks a patient to interpret a familiar proverb and explain what it means. Which area/aspect of the patients mental status is the nurse assessing? 1. memory 2. judgment 3. mood affect 4. level of awareness and orientation

2. judgment

During a mental status assessment a patient cannot remember the name of a childhood neighborhood. In which aspect of the assessment is the patient demonstrating an alteration? 1. speech 2. memory 3. orientation 4. mood affect

2. memory

A nurse is evaluating different situations that constitute acts of assault and battery. Which situations would the nurse consider to be instances of battery? Select all that apply. 1 A nurse threatens to restrain a client who refuses to consent to having a bath. 2 A nurse force feeds a client who refuses to eat by opening his mouth. 3 A nurse pats an aggressive client to calm him or her down without waiting for the client's consent. 4 A nurse administers an intramuscular injection to a client before obtaining consent for the injection. 5 A nurse locks a client in a private room to prevent him or her from going to the cafeteria without obtaining consent.

2,3,4

In what instances can a minor give consent for himself or herself for medical treatment? Select all that apply. 1 The minor can give consent for his or her siblings. 2 The minor can give consent for any venereal disease. 3 The minor can give consent if he or she is lawfully married. 4 The minor can give consent for a drug or substance abuse. 5 The minor can give consent for an abortion.

2,3,4

An injured child is brought to the emergency department by the parents. While interviewing the parents, the nurse begins to suspect child abuse. Which parental behaviors might support this conclusion? Select all that apply. 1 Demonstrating concern for the injured child 2 Focusing on the child's role in sustaining the injury 3 Changing the story of how the child sustained the injury 4 Asking questions about the injury and the child's prognosis 5 Giving an explanation of how the injury occurred that is not consistent with the injury

2,3,5

A 2½-year-old child is admitted for treatment of injuries supposedly sustained in a fall down a flight of stairs. Child abuse is suspected. What statements might the nurse expect from a parent who engages in child abuse? (Select all that apply.) 1. "Kids have to learn to be careful on the stairs." 2. "Every time I turn around the kid is falling over something." 3. "He tends to be adventurous and doesn't understand about getting hurt on the stairs." 4. "I can't understand it. He didn't have a problem using the stairs without my help before this." 5. "I try to keep an eye on him, but little kids are always on the go and I just can't keep running after him."

2. "Every time I turn around the kid is falling over something." 4. "I can't understand it. He didn't have a problem using the stairs without my help before this."

A 17-year-old teenager is diagnosed with leukemia. Which statements by the teenager reflect Piaget's cognitive processes associated with adolescence? Select all that apply. 1. "My smoking pot probably caused the leukemia." 2. I'm going to do my best to fight this terrible disease." 3. "Now I can't go to the prom because I have this stupid illness." 4. "I know I got sick because I've been causing a lot of problems at home." 5. "This illness is serious, but with treatment I think I will have a chance to get better."

2. "I'm going to do my best to fight this terrible disease." 3. "Now I can't go to the prom because I have this stupid illness." 5. "This illness is serious, but with treatment I think I will have a chance to get better." 1. At 2 to 7 years of age children are in the preoperational stage of cognitive development. They believe that external, unrelated, concrete phenomena cause illness. 2. At 17 years of age the adolescent is in the formal operational stage of cognitive development and therefore is able to understand the seriousness of leukemia. 3. At 17 years of age the adolescent is in the formal operational stage of cognitive development and therefore understands the seriousness of the illness. The statement also reflects an adolescent's preoccupation with peer socialization. 4. At 7 to 10 years of age children are in the concrete operational stage of cognitive development. Because of their egocentrism, they believe that they are responsible for situations, such as illnesses, and are being punished. 5. At 17 years of age the adolescent is in the formal operational stage of cognitive development and therefore is able to comprehend the seriousness of leukemia and the need for treatment.

Oral chlordiazepoxide (Librium) 100 mg/ hr is prescribed for a client with a Clinical Institute Withdrawal Assessment (CIWA) score of 25. The client has had 300 mg in 3 hours but is still displaying acute alcohol withdrawal symptoms. What is the next nursing action? 1. Informing the client that the limit of chlordiazepoxide has been reached 2. Administering chlordiazepoxide as indicated by the client's CIWA score 3. Requesting a prescription for another medication to replace the chlordiazepoxide. 4. Informing the health care provider that the maximum dose of chlordiazepoxide has been reached

2. Administering chlordiazepoxide as indicated by the client's CIWA score **Medication of clients in acute withdrawal from alcohol should be based on withdrawal symptoms, not medication dosage.

A nurse is monitoring a client with the diagnosis of schizophrenia who is experiencing opposing emotions simultaneously. When providing a change-of-shift report, how should the nurse refer to this emotional experience of the client? 1. Double bind 2. Ambivalence 3. Loose association 4. Inappropriate affect

2. Ambivalence

With the client's permission, the nurse should inform the family about what is happening. The main reason for this action is that informed families: 1. Ease the client's anxiety 2. Are better equipped to assist the client 3. Appear more relaxed with the situation 4. Commonly cause fewer nursing problems

2. Are better equipped to assist the client

A client on the psychiatric unit is undergoing a pretreatment evaluation for electroconvulsive therapy (ECT). Because of the client's profoundly depressed behavior, the nurse doubts that the client can provide informed consent. What should the nurse's initial intervention be? 1. Consulting with the hospital's legal staff and following their recommendation 2. Having the client verbalize her understanding and the outcomes of the procedure 3. Asking the client to sign the consent form because the client has not been declared incompetent 4. Suggesting to the health care provider that a family member sign the consent form for the client

2. Having the client verbalize her understanding and the outcomes of the procedure

The nurse is caring for an older patient with a history of depression. Which comments by the patient indicates an immediate need for further assessment? 1. I am so old, all of my friends have died 2. I am useless now; there is no reason to be alive 3. I retire in 6 months , and it will be all downhill from there 4. I am looking forward to seeing my husband in heaven someday

2. I am useless now; there is no reason to be alive

A nurse in the mental health clinic concludes that a client is using confabulation when: 1. The flow of thoughts is interrupted 2. Imagination is used to fill in memory gaps 3. Speech flits from one topic to another with no apparent meaning 4. Connections between statements are so loose that only the speaker understands them

2. Imagination is used to fill in memory gaps **Using imagination to fill in memory gaps is the definition of confabulation; it is a defense mechanism used by people experiencing memory deficits.

Which psychotherapeutic theory uses hypnosis, dream interpretation, and free association as methods to release repressed feelings? 1. Behaviorist model 2. Psychoanalytical model 3. Psychobiological model 4. Social-interpersonal model

2. Psychoanalytical model

A patient diagnosed with a mental illness is being considered for psychotherapy. What should the nurse realize are the goals for this treatment. SELECT ALL THAT APPLY 1. relaxing the body 2. improve social interactions 3. clarify the meaning of events 4. reduce emotional discomfort 5. enhance performance in situations

2. improve social interactions 4. reduce emotional discomfort 5. enhance performance in situation

The nurse is reviewing the causes of anxiety with a patient diagnosed with an anxiety disorder. Which neurotransmitter abnormalities should the nurse include as causing symptoms of anxiety? SELECT ALL THAT APPLY 1. increased substance P 2. increased epinephrine 3. increased somatostatin 4. decreased norepineohrine 5. decreased gamma aminobutyric acid (GABA)

2. increased epinephrine 5. decreased gamma aminobutyric acid (GABA)

While providing care a teenage mother expresses ambivalence about her knew roll as a parent. Which response by the nurse is best? 1. ill call the social worker so you can discuss adoption 2. tell me more about how you feel when you hold the baby 3. parenthood is certainly not for everyone, you shouldn't feel guilty about that 4. im sure your own parents felt the same way when you were small, you can talk to your mom

2. tell me more about how you feel when you hold the baby

The nurse notes that another nurse colleague has been acting differently lately. The nurse often has red watery eyes and a runny nose. Today, the nurse was unhappy with the patient assignment ans screamed. " someone is going to pay for this!" What should the nurse who has observed this behavior do? 1. nothing; all nurses have stressful days sometimes 2. tell the clinical manager exactly what was observed 3. tell the clinical manager that the nurse is abusing drugs 4. confront the nurse with the behavior and provide information about counseling

2. tell the clinical manager exactly what was observed

A patient who experienced injuries for a motor vehicle crash 6 months ago continues to request prescriptions for an opioid analgesic. When assessing this patients for opioid dependency which finding is the nurse least likely to observe? 1.the patient drops out of Saturday night bingo group 2. the patient continues to manage to get to work everyday 3. the patient tried to quit using the opioid but could't stop thinking about it 4. the patient has been to three or four physicians to obtain new prescriptions for the drug

2. the patient continues to manage to get to work everyday

An extremely anxious client enters a crisis center and asks a nurse for help. Which response best reflects the nurse's role in crisis intervention? 1. "Tell me what you have done to help yourself." 2."I will be here for you to help you figure things out." 3."I understand that in the past you have had problems." 4."Tell me about the things that are bothering you the most."

2."I will be here for you to help you figure things out." Clients in crisis need assistance with coping; the nurse must be involved with problem solving.

A nurse is working with a married woman who has come to the emergency department several times with injuries that appear to be related to domestic violence. While talking with the nurse manager, a nurse expresses disgust that the woman returns to the same situation. What is the nurse manager's best response? 1."She must not have the financial resources to leave her husband." 2."Most woman attempt to leave about six times before they are able to do so." 3."There is nothing the staff can do because people are free to choose their own life." 4."These women should be told how foolish they are to remain in their current situation."

2."Most woman attempt to leave about six times before they are able to do so." Nurses who work with clients who are victims of partner abuse need to be supportive and patient. It takes time and several attempts for most victims to be able to leave abusive relationships.

To therapeutically relate to parents who are known to have maltreated their child, the nurse must first: 1.)Recognize the emotional needs of the parents 2.) Identify personal feelings about child abusers 3.)Develop a trusting relationship with the child 4.)Gather information about the child's home environment

2.) Identify personal feelings about child abusers

When having a conversation with a nurse, an older client states, "I've lived a good life. I don't want to die, but I accept it as a part of life." What developmental stage, according to Erikson, has the client completed? 1.)Identity 2.)Integrity 3.)Acceptance 4.)Generativity

2.) Integrity

A client tells the nurse in the mental health clinic that the practitioner said that the cornerstone of therapy used in the clinic is cognitive therapy. The client asks what this therapy entails. What concept should the nurse explain as the basis of cognitive therapy? 1.)Unconscious feelings influence actions." 2.)"Negative thoughts can precipitate anxiety." 3.)"People can act their way into a new way of thinking." 4.)"Maladaptive behaviors will continue as long as they are reinforced."

2.) Negative thoughts can precipitate anxiety

A client is admitted to the psychiatric hospital with a diagnosis of obsessive-compulsive disorder. The client's anxiety level is approaching a panic level, and the client's ritual is interfering with work and daily living. Which selective serotonin reuptake inhibitor (SSRI) should the nurse anticipate that the health care provider may prescribe? 1.)Haloperidol (Haldol) 2.)Fluvoxamine (Luvox) 3.)Imipramine (Tofranil) 4.)Benztropine (Cogentin)

2.) fluvoxamine (Luvox)

A nurse is caring for a client who has abruptly stopped taking a barbiturate. What should the nurse anticipate that the client may experience? 1.) ataxia 2.)seizure 3.) diarrhea 4.) uriticaria

2.) seizure

Oral chlordiazepoxide (Librium) 100 mg/ hr is prescribed for a client with a Clinical Institute Withdrawal Assessment (CIWA) score of 25. The client has had 300 mg in 3 hours but is still displaying acute alcohol withdrawal symptoms. What is the next nursing action? 1.Informing the client that the limit of chlordiazepoxide has been reached 2.Administering chlordiazepoxide as indicated by the client's CIWA score 3.Requesting a prescription for another medication to replace the chlordiazepoxide. 4.Informing the health care provider that the maximum dose of chlordiazepoxide has been reached

2.Administering chlordiazepoxide as indicated by the client's CIWA score

What can the nurse do to help older adult to successfully complete Erikson's major task of this stage? 1.Invest creative energy in promoting social welfare 2.Develop a sense of satisfaction when considering past achievements 3.Develop deep and lasting relationships with other people or institutions 4.Recapture opportunities that the client did not take or those that were not fulfilled

2.Develop a sense of satisfaction when considering past achievements

A nurse in the mental health clinic concludes that a client is using confabulation when: 1.The flow of thoughts is interrupted 2.Imagination is used to fill in memory gaps 3.Speech flits from one topic to another with no apparent meaning 4.Connections between statements are so loose that only the speaker understands them

2.Imagination is used to fill in memory gaps

A client with a history of aggressive, violent behavior is admitted to the psychiatric unit involuntarily. The nurse, who understands the need to use deescalation approaches during the preassaultive stage of the violence cycle, monitors the client's behavior closely for progression of signs of impending violence. List these client behaviors in order of escalating aggression, from the lowest risk to the highest. 1.Pacing in the hall 2.Increasing tension in facial expression 3.Engaging in verbal abuse toward the nurse 4.Pushing another client while waiting in line to the dining room 5.Having difficulty waiting to take turns during a group project

2.Increasing tension in facial expression 5.Having difficulty waiting to take turns during a group project 1.Pacing in the hall 3.Engaging in verbal abuse toward the nurse 4.Pushing another client while waiting in line to the dining room

A 65-year-old man is admitted to the hospital with a history of depression. The client, who speaks little English and has had few outside interests since retiring, says, "I feel useless and unneeded." The nurse concludes that the client is in Erikson's developmental stage of: 1.Initiative versus guilt 2.Integrity versus despair 3.Intimacy versus isolation 4.Identity versus role confusion

2.Integrity versus despair

A nurse is evaluating a young adult for evidence of achievement of the age-related developmental stage set forth in Erikson's developmental theory. What developmental crisis is associated with this age group? 1.Trust versus mistrust 2.Intimacy versus isolation 3.Industry versus inferiority 4.Generativity versus stagnation

2.Intimacy versus isolation

According to Erikson, a young adult must accomplish the tasks associated with the stage known as: 1.Trust versus mistrust 2.Intimacy versus isolation 3.Industry versus inferiority 4.Generativity versus stagnation

2.Intimacy versus isolation

Incidents of child molestation often are revealed years later when the victim is an adult. Which defense mechanism reflects this situation? 1.Isolation 2.Repression 3.Regression 4.Introjection

2.Repression Rationale: 2. Repression is a coping mechanism in which unacceptable feelings are kept out of conscious awareness; later, under stress or anxiety, thoughts or feelings surface and come into one's conscious awareness.

The emergency department nurse is conducting an interview and assisting with the physical examination of a female sexual assault victim. What is most important for the nurse to document on this client's record? 1.Observations about the client's reaction to male staff members 2.Statements by the client about the sexual assault and the rapist 3.Information about the client's previous knowledge of the rapist 4.Summary statement about the client's description of the assault and the rapist

2.Statements by the client about the sexual assault and the rapist

A health care provider prescribes haloperidol (Haldol) for a client. What should the nurse teach the client to avoid while taking this medication? 1.Driving at night 2.Staying in the sun 3.Ingesting aged cheeses 4.Taking medications containing aspirin

2.Staying in the sun

A mother and her 5-year-old daughter have been referred to a child advocacy center for a forensic pediatric sexual examination. Before the child is examined or interviewed, the mother gives a detailed history, relaying her suspicion that the child's maternal grandfather sexually assaulted her. As the interview progresses, the mother suddenly says, "My father sexually molested me when I was a child, but I try not to think about it." What defense mechanism does the nurse recognize that the mother's statement demonstrates? 1.Introjection 2.Suppression 3.Passive aggression 4.Reaction formation

2.Suppression

A client with the diagnosis of schizophrenia is given one of the antipsychotic drugs. The nurse understands that antipsychotic drugs can cause extrapyramidal side effects. Which effect is cause for the greatest concern? 1.Akathisia 2.Tardive dyskinesia 3.Parkinsonian syndrome 4.Acute dystonic reaction

2.Tardive dyskinesia

A nurse plans to use family therapy as a means of helping a family cope with their child's terminal illness. The nurse bases this choice on the principle that: 1.It is more efficient to interact with the whole family together 2.The entire family is involved because what happens to one member affects all 3.The nurse can control manipulation and alliances better by using this mode of intervention 4.It will prevent the parents from deceiving each other about the true nature of their child's condition

2.The entire family is involved because what happens to one member affects all

A nurse reminds a client that it is time for group therapy. The client responds by yelling at the nurse, "You are always telling me what to do, just like my father!" What defense mechanism is the client using? 1.Regression 2.Transference 3.Reaction formation 4.Cognitive distortion

2.Transference With transference a client assigns to someone the feelings and attitudes originally associated with an important significant other.

The nurse is preparing to establish a relationship with a newly admitted patient. On which qualities should the nurse focus when creating this nurse-patient relationship? SELECT ALL THAT APPLY 1. humor 2.respect 3. honesty 4. empathy 5. sympathy 6. friendship

2.respect 3. honesty 4. empathy

Obesity is defined as body weight that is _____ above average, based on the individuals height and build.

20%

A 20-year-old woman is brought to an emergency department after having been raped. She is very anxious and cannot recall any of the circumstances surrounding the assault or provide the police with a description of the rapist. What defense mechanism does the nurse know is being utilized by this woman? 1 Projection 2 Regression 3 Repression 4 Displacement

3

Methylphenidate (Ritalin) is prescribed to treat a 7-year-old child's attention deficit-hyperactivity disorder (ADHD). The nurse understands that methylphenidate is used in the treatment of this disorder in children for its: 1. Diuretic effect 2. Synergistic effect 3. Paradoxical effect 4. Hypotensive effect

3. Paradoxical effect **Methylphenidate (Ritalin), a stimulant, has an opposite effect on hyperactive children; the reason for this action is as yet totally unexplained

A 6-year-old child with a leg fracture of suspicious origin is brought into the emergency department by the mother and the mother's boyfriend. It is the child's first visit to this hospital. After assessing the child, a nurse anticipates that the healthcare provider will order a skeletal survey. Why is a skeletal survey the preferred diagnostic tool? 1 The exact location and extent of the fracture will be pinpointed. 2 Three separate x-ray films of the leg and hip should be ordered, making it more cost-effective. 3 The skeletal history of the current fracture and any previous healing or healed fractures are identified. 4 It is the first step toward a complete assessment before computed tomography and magnetic resonance imaging are done.

3

A child would be demonstrating outwardly focused anger or aggression in an overt manner when engaging in which behavior? 1 Dominating a class discussion 2 Intentionally forgetting to do homework 3 Scribbling on a classmate's art assignment 4 Crying when told he or she must wait his or her turn

3

A client is presented with the treatment option of electroconvulsive therapy (ECT). After discussion with staff members, the client requests that a family member be called to help make the decision about this treatment. What ethical principle does the nurse consider when supporting the client's request? 1 Justice 2 Veracity 3 Autonomy 4 Beneficence

3

A client tells the nurse, "I am so worried about the results of the biopsy they took today." The nurse overhears the nursing assistant reply, "Don't worry. I'm sure everything will come out all right." What does the nurse conclude about the nursing assistant's answer? 1 It shows empathy. 2 It uses distraction. 3 It gives false reassurance. 4 It makes a value judgment.

3

A client who has a hemoglobin of 6 gm/dL (60 mmol/L) is refusing blood because of religious reasons. What is the most appropriate action by the nurse? 1 Call the chaplain to convince the client to receive the blood transfusion. 2 Discuss the case with coworkers. 3 Notify the primary healthcare provider of the client's refusal of blood products. 4 Explain to the client that they will die without the blood transfusion.

3

A married woman is brought to the emergency department of a local hospital. Her eyes are swollen shut, and she has a bruise on her neck. She reports that she is being beaten by her husband. How does the nurse expect the husband to behave when he arrives at the emergency department? 1 Fearful 2 Confused 3 Charming 4 Indifferent

3

A nurse is teaching staff members about the legal terminology used in child abuse. What definition of battery should the nurse include in the teaching? 1 Maligning a person's character while threatening to do bodily harm 2 A legal wrong committed by one person against property of another 3 The application of force to another person without lawful justification 4 Behaving in a way that a reasonable person with the same education would not

3

A nurse who lacks confidence in her performance in a new position is worried about an upcoming review with the nursing director. What type of power does the nursing director hold in this scenario? 1 Expert power 2 Position power 3 Coercive power 4 Referent power

3

A primary nurse completes a nursing assessment of all assigned clients and develops a care plan for each client. Which element of decision-making does the nurse execute in this situation? 1 Authority 2 Autonomy 3 Responsibility 4 Accountability

3

A woman with five children comes to the emergency department with multiple facial injuries. The client says, "My husband is an alcoholic, and he just beat me up." The nurse concludes that the client appears to be a victim of abuse. What should the nurse do next? 1 Discuss birth control with her. 2 Report her experiences to the police. 3 Inquire about her and the children's safety. 4 Discuss the possibility of her and the children leaving her husband.

3

The family of a client infected with human immunodeficiency virus (HIV) wants to see the results of the client's blood tests, unaware that the client is infected. A nurse obliges the family's request without waiting for the client's consent. What legal charge may be brought against the nurse? 1 Slander 2 Negligence 3 Invasion of privacy 4 Defamation of character

3

What is the term used to identify the display of anger in a socially inappropriate manner? 1 Abuse 2 Battery 3 Aggression 4 Defensiveness

3

Which statement accurately demonstrates an act of nursing negligence? 1 A nurse enters false information in the client's electronic health record to prolong treatment. 2 A nurse threatens to initiate intravenous therapy by force because the client refuses to give consent. 3 A nurse instructs the nursing assistive personnel to administer medication through an intravenous line. 4 A nurse informs the client's family about a surgical procedure despite the client's instructions against doing so.

3

Many people control anxiety with the use of ritualistic behavior. What must the nurse do when caring for these clients? 1 Avoid mentioning the ritual 2 Explain the meaning of the ritual 3 Allow them time to carry out the ritual 4 Prevent them from carrying out the ritual

3 Allow them time to carry out the ritual Allowing the client who uses a ritual time to carry out the ritual reduces the client's anxiety. Clients prevented from using ritualistic behavior to control anxiety are being deprived of a defense and will not be able to relieve tension. The client's behavior should never be ignored; it is important to accept and support these clients during this time. Explaining the meaning of the ritual will not decrease the use of the behavior. Preventing a ritualistic behavior will probably increase the client's anxiety.

An 18-month-old toddler requires an intramuscular injection. As the nurse enters the room with the medication, the child begins to scream and flail about on the bed. The father is sitting at the child's bedside and gets up to leave. What action should the nurse take to best handle this situation? 1 Allow the child to say good-bye to the father and then give the injection 2 Tell the father to return and comfort the child after the injection is given 3 Ask the father to stay to comfort the child while the injection is being given 4 Leave the room and ask another nurse to come in to hold the child during the injection

3 Ask the father to stay to comfort the child while the injection is being given Toddlers are extremely fearful of separation, as well as of intrusive procedures; if the parents are present, they should be encouraged to stay and give comfort. Toddlers depend on their parents for comfort and control. Toddlers are dependent on their parents; the parent should be encouraged to participate in care.

In response to a question posed during a group meeting, the nurse explains that the superego is that part of the self that says: 1. "I like what I want." 2. "I want what I want." 3. "I should not want that." 4. "I can wait for what I want."

3. "I should not want that." Rational Conscience and a sense of right and wrong are expressed in the superego, which acts to counterbalance the id's desire for immediate gratification.

A nurse leads an assertiveness training program for a group of clients. Which client statement demonstrates that the treatment has been effective? 1. "I know I should put the needs of others before mine." 2. "I won't stand for it, so I told my boss he's a jerk and to get off my back." 3. "It annoys me when people call me 'Dearie,' so I told him not to do it anymore." 4. "It is easier for me to agree up front and then just do enough so that no one notices."

3. "It annoys me when people call me 'Dearie,' so I told him not to do it anymore." This is an assertive statement; it clearly states what the problem is and sets limits on undesired behavior without being demeaning.

On the first day of the month a practitioner prescribes an antipsychotic medication for a client with schizophrenia. The initial dosage is 25 mg once a day, to be titrated in increments of 25 mg every other day to a desired dosage of 175 mg daily. On what day of the month will the client reach the desired daily dose of 175 mg? 1. Day 7 2. Day 9 3. Day 13 4. Day 15

3. Day 13 **The client will reach the desired dosage of 175 mg on the 13th day of the month; on the first day it is 25 mg, on the third day it is 50 mg, on the fifth day it is 75 mg, on the seventh day it is 100 mg, on the ninth day it is 125 mg, on the 11th day it is 150 mg, and on the 13th day it is 175 mg.

A client has been prescribed lithium. What important nursing intervention must be implemented while this medication is being administered? 1. Restricting the client's daily sodium intake 2. Testing the client's urine specific gravity weekly 3. Regularly testing the level of the drug in the client's blood 4. Withholding the client's other medications for several days

3. Regularly testing the level of the drug in the client's blood

A nurse is interviewing an 8-year-old girl who was admitted to the pediatric unit. Which statement by the child needs to be explored? 1."Wow! This place has bright colors." 2."Is my mother allowed to visit me tonight?" 3."Those boys are so cute. I hope their room is next to mine!" 4."I am scared about being here. Can you stay with me awhile?"

3."Those boys are so cute. I hope their room is next to mine!" 3. A 7-year-old child should be more concerned with same-gender relationships. A child demonstrating a strong attraction to opposite-gender relations should be questioned further regarding the possibility of sexual abuse.

A nurse is caring for a client who is angry and agitated. What is the best approach for the nurse to use with this client? 1.)Confronting the client about the behavior 2.)Turning on the television to distract the client 3.)Maintaining a calm, consistent approach with the client 4.)Explaining to the client why the behavior is unacceptable

3.) Maintaining a calm, consistent approach with the client

A client with depression is to be given fluoxetine (Prozac). What precaution should the nurse consider when initiating treatment with this drug? 1.)It must be given with milk and crackers to avoid hyperacidity and discomfort. 2.)Eating cheese or pickled herring or drinking wine may cause a hypertensive crisis. 3.)The blood level may not be sufficient to cause noticeable improvement for 2 to 4 weeks. 4.)The blood level should be checked weekly for 3 months to monitor for an appropriate level

3.) The blood level may not be sufficient to cause noticeable improvement for 2 to 4 weeks

A nurse is interviewing an 8-year-old girl who has been admitted to the pediatric unit. Which statement by the child needs to be explored? 1.)"Wow! This place has bright colors." 2.) "Is my mother allowed to visit me tonight?" 3.)"Those boys are so cute. I hope their room's next to mine!" 4.)"I'm scared about being here. Can you stay with me awhile?"

3.) Those boys are so cute. I hope their room's next to mine!

A client is scheduled for several diagnostic studies. Which behavior best indicates to the nurse that the client has received adequate preparation? 1.Requests that the tests be reexplained 2.Checks the appointment card repeatedly 3.Arrives early and waits quietly to be called for the tests 4.Paces up and down the hallway the morning of the tests

3.Arrives early and waits quietly to be called for the tests Rationale: 3. The client's early arrival indicates an expected degree of anxiety; the quiet waiting indicates that the client has been told what to expect.

A terminally ill client repeatedly tells the nurse all the details of a daughter's wedding that will take place in 6 months and how important it is for her to attend. What Kübler-Ross stage of grieving does the nurse identify? 1.Anger 2.Denial 3.Bargaining 4.Acceptance

3.Bargaining

A parent whose daughter is killed in a school bus accident tearfully tells the nurse, "My daughter was just getting over the chickenpox and didn't want to go to school, but I insisted that she go. It's my fault that she's dead." How should the nurse anticipate that perceiving a death as preventable will likely influence the grieving process? 1.The loss may be easier to understand and accept. 2.The mourner may experience pathological grief. 3.Bereavement may be of greater intensity and duration. 4.The grieving process may progress to a psychiatric illness

3.Bereavement may be of greater intensity and duration.

When planning care for an older client, the nurse remembers that aging has little effect on a client's: 1.Sense of taste or smell 2.Muscle or motor strength 3.Capacity to handle life's stresses 4.Ability to remember recent events

3.Capacity to handle life's stresses

A client is admitted for treatment of obsessive-compulsive disorder that is interfering with activities of daily living. Which medication should the nurse anticipate the health care provider will prescribe? 1.Benztropine (Cogentin) 2.Amantadine (Symmetrel) 3.Clomipramine (Anafranil) 4.Diphenhydramine (Benadryl)

3.Clomipramine (Anafranil)

On the first day of the month a practitioner prescribes an antipsychotic medication for a client with schizophrenia. The initial dosage is 25 mg once a day, to be titrated in increments of 25 mg every other day to a desired dosage of 175 mg daily. On what day of the month will the client reach the desired daily dose of 175 mg? 1.Day 7 2.Day 9 3.Day 13 4.Day 15

3.Day 13

What is most important for the nurse to do to assist a couple to cope with their feelings about the husband's terminal illness? 1.Referring the husband to a psychotherapist for help in dealing with his anger 2.Placing the couple in a couple's therapy group that addresses terminal illness 3.Helping the couple express to each other their feelings about his terminal illness 4.Encouraging the wife to verbalize her feelings to a therapist during individual therapy sessions

3.Helping the couple express to each other their feelings about his terminal illness

A health care provider refers a 52-year-old man to the mental health clinic. The history reveals that the man lost his wife to colon cancer 6 months ago and that since that time he has seen his health care provider seven times with the concern that he has colon cancer. All tests have had negative results. Recently the client stopped seeing friends, dropped his hobbies, and stayed home to rest. Which disorder should the nurse identify as consistent with the client's preoccupation with the fear of having a serious disease? 1.Conversion disorder 2.Somatization disorder 3.Hypochondriac disorder 4.Body dysmorphic disorder

3.Hypochondriac disorder

A woman with five children comes to the emergency department with multiple facial injuries. The client says, "My husband is an alcoholic, and he just beat me up." The nurse concludes that the client appears to be a victim of abuse. What should the nurse do next? 1.Discuss birth control with her 2.Report her experiences to the police 3.Inquire about her and the children's safety 4.Discuss the possibility of her and the children leaving her husband

3.Inquire about her and the children's safety

A nurse is administering medications to clients on a psychiatric unit. What does the nurse identify as the reason that so many psychiatric clients are given the drug benztropine (Cogentin) or trihexyphenidyl in conjunction with the phenothiazine derivatives neuroleptic medications? 1.It reduces postural hypotension. 2.It potentiates the effects of the other drug. 3.It combats the extrapyramidal side effects of the other drug. 4.It ameliorates the depression that may accompany schizophrenia

3.It combats the extrapyramidal side effects of the other drug.

A client asks the nurse, "Because I'm so comfortable talking with you, can we go out for coffee and a movie after I get discharged?" To maintain the boundaries of a therapeutic relationship, how will the nurse respond? 1 "I'm flattered, but that would be professionally unethical." 2 "You feel connected to me now; that will change once you are discharged." 3 "The attention I've been giving you is directed toward getting you better; it isn't social." 4 "A social life is important, so as your nurse let's talk about how you can form friendships."

4

A client receiving steroid therapy states, "I have difficulty controlling my temper, which is so unlike me, and I don't know why this is happening." What is the nurse's best response? 1 Tell the client it is nothing to worry about. 2 Reassure that everyone does this at times. 3 Instruct the client to attempt to avoid situations that cause irritation. 4 Interview the client to determine whether other mood swings are being experienced

4

A nurse is teaching a class about child abuse. What defense mechanism most often used by the physically abusive individual should the nurse include? 1 Repression 2 Manipulation 3 Transference 4 Displacement

4

A teacher's aide in a kindergarten class informs the school nurse that a male student said that his mother beat him and that he has bruises on the back and shoulders. What is the priority nursing action? 1 Notifying Child Protective Services 2 Reporting this information to the principal 3 Calling the parents to arrange a conference 4 Assessing the child for the presence of bruises

4

A young female client admitted to the trauma center after being sexually assaulted continues to talk about the rape. Toward what goal should the primary nursing intervention be directed? 1 Getting her involved with a rape therapy group 2 Remaining available and supportive to limit destructive anger 3 Exploring her feelings about men to promote future relationships 4 Providing a safe environment that permits the ventilation of feelings

4

The nurse is caring for a newly admitted male client with the diagnosis of bipolar disorder who has a history of hyperactivity and combativeness. Later in the evening, a commotion is heard and the new client is found beating another client. What is the legal interpretation of this situation? 1 The client should have been placed in restraints on admission. 2 A client who is known to have been combative should have been kept sedated. 3 A client with bipolar disorder who is in contact with reality does not require supervision. 4 Because it was known that the client was frequently combative, close observation by the nursing staff was indicated.

4

The school nurse is conducting a teacher's in-service on signs that may indicate that a child is a victim of bullying. Which sign should the nurse include in the teaching session? 1 The child wants to try out for the basketball team. 2 The child asks for extra work to make better grades. 3 The child is participating in several extracurricular activities after school. 4 The child asks to go to the nurse's office frequently with vague complaints.

4

The spouse of a client who is dying tells the primary nurse that the client is asking the nurses to leave the pain medication on the bedside table and fears they are being saved for a suicide attempt. The nurse knows that the staff members have mixed feelings about the client's terminal status and prolonged pain. What is the most ethically appropriate intervention by the nurse? 1 Reporting the information about the medication to the nurse manager 2 Reminding the nurses that they should not leave the medication at the bedside 3 Asking the nurse manager to address the medication problem and the staff's feelings 4 Suggesting a nursing conference to discuss the medication problem and the staff's feelings

4

A client who is taking clozapine (Clozaril) calls the nurse in the psychiatric clinic to report the sudden development of a sore throat and a high fever. What should the nurse instruct the client to do? 1 Stay in bed, drink fluids, take a dose of aspirin, and ask the health care provider to reduce the dosage of clozapine 2 Discontinue the medication immediately and see the health care provider as soon as an appointment becomes available 3 Continue the medication, drink fluids, take aspirin, and see the health care provider in a few days if the symptoms do not improve Correct4 Discontinue the medication and, if the health care provider is unavailable today, go to the emergency department for evaluation

4 Discontinue the medication and, if the health care provider is unavailable today, go to the emergency department for evaluation Symptoms of infection are suggestive of agranulocytosis, an adverse effect that can occur with clozapine therapy and can cause death. Remaining in bed, drinking fluids, taking aspirin, and asking the health care provider to decrease the dose of clozapine is unsafe because agranulocytosis may be developing, and this life-threatening side effect requires immediate treatment. Also, prescribing medications is outside the legal role of the nurse. Only a certified nurse practitioner can prescribe medications. Although discontinuing the medication is acceptable advice, delaying a health care provider's evaluation is unsafe. Continuing the medication, drinking fluids, taking aspirin, and seeing the health care provider in a few days if the condition does not improve is unsafe because agranulocytosis may be developing.

A patient with a mental illness says" I have to go to the bank. The voices are telling me to go there."Which response by the nurse is best? 1. do you need money 2. I will call you a cab later. right now it is time for therapy 3. why do you think the voices are telling you to go to the bank 4. I want to help you focus away from the voices. I am real, they are not

4. I want to help you focus away from the voices. I am real, they are not

The mother of a 13-year-old child with sickle cell anemia tells the nurse that the family is going camping by a lake this summer. She asks what activities are appropriate for her child. Which activity should the nurse suggest? 1 Swimming in the lake 2 Soccer with the family 3 Climbing the mountain trails Correct4 Motorboat rides around the lake

4 Motorboat rides around the lake Motor boating is a relatively passive activity that will not increase the child's oxygen demands, which can precipitate sickling and therefore a painful episode. Mountain lakes are usually cold; temperature extremes can contribute to sickling that may precipitate a painful episode. Playing soccer may lead to increased cellular metabolism and increased tissue hypoxia, which can precipitate sickling that could progress to a painful episode. High altitudes should be avoided because the lower oxygen concentration of the air might trigger a painful episode.

What is the best nursing intervention during the working phase of the therapeutic relationship with which to meet the needs of individuals who demonstrate obsessive-compulsive behavior? 1 Restricting their movements 2 Calling attention to the behavior 3 Keeping them busy to distract them 4 Supporting rituals while setting realistic limits

4 Supporting rituals while setting realistic limits Accepting obsessive-compulsive clients and their symptomatic behavior sets the foundation for the nurse-client relationship. Setting limits provides external controls and helps reduce anxiety. Supporting rituals while setting realistic limits is appropriate during the working phase, not the initial phase, of a therapeutic relationship. Restricting movements will have no effect other than to increase anxiety. Calling attention to the behavior will increase the client's anxiety and increase use of the behavior. Keeping the client busy in an effort to distract him or her is unrealistic.

A nurse determines that confrontation is an appropriate tool for use with a client. What is an example of therapeutic confrontation? 1. "I find that hard to believe." 2. "I noticed that you're not wearing any makeup today." 3. "You feel frustrated because you think your mother doesn't understand you." 4. "You say you're not a good parent, but you were effective when you were talking with your son today."

4. "You say you're not a good parent, but you were effective when you were talking with your son today."

A patient is newly diagnosed with a trauma related disorder. Which medication should the nurse expect to be prescribed for this patient? 1. paraxetine (Paxil) 2. Sertraline ( Zoloft) 3. Buspirone ( Buspar) 4. Alprazolam (xanax)

4. Alprazolam (xanax)

A nurse is teaching a client about side effects of medications. Which drug will cause a heightened skin reaction to sunlight? 1. Lithium 2. Sertraline 3. Methylphenidate 4. Chlorpromazine

4. Chlorpromazine

After speaking with the parents of a child dying of leukemia, the practitioner gives a verbal do-not-resuscitate order but refuses to put it in writing. What should the nurse do? 1. Follow the order as given by the practitioner 2. Refuse to follow the practitioner's order unless the nursing supervisor approves it 3. Ask the practitioner to write the order in pencil on the child's chart before leaving the room 4. Determine whether the family is in accord with the practitioner while following hospital policy

4. Determine whether the family is in accord with the practitioner while following hospital policy

A nurse is teaching a class about child abuse. What defense mechanism most often used by the physically abusive individual should the nurse include? 1.Repression 2.Transference 3.Manipulation 4.Displacement

4. Displacement is a defense mechanism in which one's pent-up feelings toward a threatening person are discharged on less-threatening others.

A client with a diagnosis of schizophrenia is discharged from the hospital. At home the client forgets to take the medication, is unable to function, and must be rehospitalized. What medication may be prescribed that can be administered on an outpatient basis every 2 to 3 weeks? 1. Lithium 2. Diazepam 3. Fluvoxamine 4. Fluphenazine

4. Fluphenazine **Fluphenazine can be given intramuscularly every 2 to 3 weeks to clients who are unreliable about taking oral medications; it allows them to live in the community while keeping the disorder under control.

Imipramine (Tofranil), 75 mg three times per day, is prescribed for a client. What nursing action is appropriate when this medication is being administered? 1. Telling the client that barbiturates and steroids will not be prescribed 2. Warning the client not to eat cheese, fermented products, and chicken liver 3. Monitoring the client for increased tolerance and reporting when the dosage is no longer effective 4. Having the client checked for increased intraocular pressure and teaching about symptoms of glaucoma

4. Having the client checked for increased intraocular pressure and teaching about symptoms of glaucoma **Glaucoma is one of the side effects of imipramine (Tofranil), and the client should be taught the symptoms.

A patient with schizophrenia calls the nurse into the room and says "help me! the books are ion fire!" Which response by the nurse is best? 1. I'll get some water and put it out 2. that's crazy; you know the books are not on fire! 3. you don't have any books ; how could they be on fire? 4. I do not see any fire. Here is your supper; it's time to eat

4. I do not see any fire. Here is your supper; it's time to eat

A patient cannot leave home without checking the coffee pot numerous times. This behavior makes the patient late for many functions. Which anxiety disorder should the nurse suspect the patient is experiencing? 1. phobia 2. generalized anxiety disorder 3. PTSD 4. obsessive compulsive disorder (OCD)

4. obsessive compulsive disorder (OCD)

The husband of a woman who gave birth to a baby 2 weeks ago calls the postpartum unit at the hospital, seeking assistance for his wife. He reports that he found his wife in bed and that the baby was wet, dirty, and crying in the crib. He says, "She says she just can't do it." What is the best response by the nurse? 1. Encouraging him to express his feelings about the situation 2. Telling him to schedule an appointment with the gynecologist 3. Asking whether he can afford a home health aide for several weeks 4. Informing him that he should seek emergency intervention for his wife

4. Informing him that he should seek emergency intervention for his wife **The inability to care for herself or her infant is a significant sign that the wife is depressed and in need of immediate intervention

A young mother of three children, all born 1 year apart, has been hospitalized after trying to hang herself. The client is being treated with milieu therapy. The nurse understands that this therapeutic modality consists of: 1. Providing individual and family therapy 2. Using positive reinforcement to reduce guilt 3. Uncovering unconscious conflicts and fantasies 4. Manipulating the environment to benefit the client

4. Manipulating the environment to benefit the client

A client is receiving a monoamine oxidase inhibitor (MAOI). What should the nurse teach the client? 1. It is necessary to avoid the sun. 2. Drowsiness is an expected side effect of this medication. 3. The therapeutic and toxic levels of the drug are very close. 4. Many prescribed and over-the-counter drugs cannot be taken with this medication

4. Many prescribed and over-the-counter drugs cannot be taken with this medication

A young female client admitted to the trauma center after being sexually assaulted continues to talk about the rape. Toward what goal should the primary nursing intervention be directed? 1. Getting her involved with a rape therapy group 2. Remaining available and supportive to limit destructive anger 3. Exploring her feelings about men to promote future relationships 4. Providing a safe environment that permits the ventilation of feelings

4. Providing a safe environment that permits the ventilation of feelings

The nurse is explaining the Client Bill of Rights to a female client whose psychiatrist has admitted her to an inpatient facility. Her admission is voluntary. The statement that is not a client right is the right to: 1. Personal mail 2. Refuse treatment 3. Written treatment plans 4. Select health team members

4. Select health team members **Clients may not select the members of the health care team when admitted to an inpatient setting that delivers care 24 hours a day, 7 days a week. The other rights are included in the Client Bill of Rights.

Health care providers who work with clients from different cultures must be aware of 1. A history of depression 2. Client's need to reduce anxiety 3. The history of the problem 4. The meaning of the problem for the client

4. The meaning of the problem for the patient

What is the most important information for a nurse to teach to prevent relapse in a client with a psychiatric illness? 1. The need to develop a close support system 2. The need to create a stress-free environment 3. The need to refrain from activities that cause anxiety 4. The need to follow the prescribed medication regimen

4. The need to follow the prescribed medication regimen

A patient who is a war veteran states, " It should have been me that died. I'll never forgive myself for leaving my buddy when he needed me." The nurse recognizes this statement is most associated with which diagnosis? 1. bipolar depression 2. generalized anxiety 3. obsessive- compulsive disorder 4. post traumatic stress disorder (PTSD)

4. post traumatic stress disorder (PTSD)

A physician is admitted to the psychiatric unit of a community hospital. The client, who was restless, loud, aggressive, and resistive during the admission procedure, states, "I will take my own blood pressure." What is the nurse's most therapeutic response? 1."Right now you are just another client." 2."If you would rather, I'm sure you will do it correctly." 3."I will get the attendants to assist me if you do not cooperate." 4."I am sorry, but I cannot allow that because I must take your blood pressure."

4."I am sorry, but I cannot allow that because I must take your blood pressure."

A parent of a 13-year-old adolescent who was recently diagnosed with Hodgkin disease tells a nurse, "I don't want my child to know the diagnosis." How should the nurse respond? 1."It is best if your child knows the diagnosis." 2."Did you know the cure rate for Hodgkin is high?" 3."Would you like someone with Hodgkin to talk with you?" 4."Let's talk about your feeling regarding your child's diagnosis."

4."Let's talk about your feeling regarding your child's diagnosis."

During a phone conversation to a crisis hotline a client states, "I'm falling apart and can't put myself together. This goes on and on." What is the most therapeutic response by the nurse? 1."Is there anyone there with you?" 2."What do you think this means?" 3."How do you usually handle this type of situation?" 4."What's happening right now that prompted you to call?"

4."What's happening right now that prompted you to call?"

As depression begins to lift, a client is asked to join a small discussion group that meets every evening on the unit. The client is reluctant to join because "I have nothing to talk about." What is the best response by the nurse? 1."Maybe tomorrow you will feel more like talking." 2."Could you start off by talking about your family?" 3."A person like you has a great deal to offer the group." 4."You feel you will not be accepted unless you have something to say?"

4."You feel you will not be accepted unless you have something to say?" This reflective statement allows the client to either validate or correct the nurse.

As depression begins to lift, a client is asked to join a small discussion group that meets every evening on the unit. The client is reluctant to join because, she says, "I have nothing to talk about." What is the best response by the nurse? 1."Maybe tomorrow you'll feel more like talking." 2."Could you start off by talking about your family?" 3."A person like you has a great deal to offer the group." 4."You feel you won't be accepted unless you have something to say?"

4."You feel you won't be accepted unless you have something to say?"

A woman who is frequently physically abused tells the nurse in the emergency department that it is her fault that her husband beats her. What is the most therapeutic response by the nurse? 1."Maybe it was your husband's fault, too." 2."I can't agree with that—no one should be beaten." 3."Tell me why you believe that you deserve to be beaten." 4."You say that it was your fault—help me understand that."

4."You say that it was your fault—help me understand that."

A client on the psychiatric unit who is receiving high-dosage risperidone (Risperdal) is exhibiting tremors of the hands. What should be the nurse's first intervention? 1.)Withholding the medication 2.)Telling the client it is transitory 3.)Giving the client finger exercises 4.)Contacting the health care provider

4.) contacting the health care provider

How should a nurse at an assisted living facility encourage a client to effectively complete the tasks of older adulthood? 1.By redefining the resident's role in society 2.By investing the resident's energies into nurturing others 3.By trying to complete missed opportunities with significant others 4.By fostering a sense of contentment when the client looks back on her achievements

4.By fostering a sense of contentment when the client looks back on her achievements

A female nurse has been caring for a depressed 75-year-old woman who reminds her of her grandmother. The nurse spends extra time with her every day and brings her home-baked cookies. The nurse's behavior reflects: 1.Affiliation 2.Displacement 3.Compensation 4.Countertransference

4.Countertransference

A client states, "I get down on myself when I make a mistake." When a cognitive therapy approach is used, which nursing interventions are most appropriate? Select all that apply. 1.Teaching the client relaxation exercises to diminish stress 2.Exploring with the client past experiences that caused distress 3.Providing the client with mastery experiences to boost self-esteem 4.Encouraging the client to replace negative thoughts with positive thoughts 5.Helping the client to modify the belief that anything less than perfection is unacceptable

4.Encouraging the client to replace negative thoughts with positive thoughts 5.Helping the client to modify the belief that anything less than perfection is unacceptable Rationale: 4. Cognitive therapy seeks to find underlying self-defeating beliefs and replace them with more reality-based positive beliefs. 5. Cognitive therapy encourages the use of cognitive restructuring (cognitive reframing) through positive self-talk and rational mindset.

What is the most important information a nurse should teach to prevent relapse in a client with a psychiatric illness? 1.Develop close support systems 2.Create a stress-free environment 3.Refrain from activities that cause anxiety 4.Follow the prescribed medication regimen

4.Follow the prescribed medication regimen This is important because side effects and denial of illness may cause clients to stop taking their medications; this is a common cause of relapse.

Imipramine (Tofranil), 75 mg three times per day, is prescribed for a client. What nursing action is appropriate when this medication is being administered? 1.Telling the client that barbiturates and steroids will not be prescribed 2.Warning the client not to eat cheese, fermented products, and chicken liver 3.Monitoring the client for increased tolerance and reporting when the dosage is no longer effective 4.Having the client checked for increased intraocular pressure and teaching about symptoms of glaucoma

4.Having the client checked for increased intraocular pressure and teaching about symptoms of glaucoma

Which goal specific to a client with impaired verbal communication related to a psychological barrier should be documented in the client's clinical record? 1.Freedom from injury 2.Engaging independently in solitary craft activities 3.Identifying the consequences of acting-out behavior 4.Interacting appropriately with others in the therapeutic milieu

4.Interacting appropriately with others in the therapeutic milieu

A client says, "Since my husband died I've got nothing to live for. I just want to die." The nurse hears the nursing assistant say, "Things will get better soon." The nurse identifies this response as: 1.Offering advice 2.Belittling the client 3.Changing the subject 4.Providing false reassurance

4.Providing false reassurance

A nurse is caring for several clients who have severe psychiatric disorders. What is the major reason that a health care provider prescribes an antipsychotic medication for these clients? 1.To improve judgment 2.To promote social skills 3.To diminish neurotic behavior 4.To reduce the positive symptoms of psychosis

4.To reduce the positive symptoms of psychosis

A nurse administers an antipsychotic medication to a client. For which common manageable side effect should the nurse evaluate the client? 1.Jaundice 2.Melanocytosis 3.Drooping eyelids 4.Unintentional tremor

4.Unintentional tremor

The parents of an adolescent who engages in self-injurious cutting behavior ask the nurse why their child self-mutilates. What should the nurse give as the reason for the cutting? 1.Cry for help 2.Suicide attempt 3.Attention-seeking behavior 4.Way to manage overwhelming feelings

4.Way to manage overwhelming feelings

A client complains to the nurse that he has been fired from his fourth job in 10 months because his bosses and co-workers didnt understand him. While he once had a few close friends, he no longer associates with them for the same reason. His level of functioning on the global assessment of functioning (GAF) scale would be:

41-50; serious symptoms

One of the developmental tasks of adolescence is to establish intimacy and relationships. By the age of 14, what percentage of teens have experienced sexual intercourse?

50%

Environmental problems often lead to mental health problems among adolescents. Approximately _____ million children and adolescents must cope with the issue of having a parent in jail or on parole.

7

The practitioner prescribes valproic acid (Depakene) 750 mg daily to be administered in two divided doses. The medication is supplied as a syrup of 250 mg/5 mL. How many milliliters of solution should the nurse administer per dose? Record the answer using one decimal place. ___ mL

7.5 mL

C

A 3-year-old child is brought to the emergency department by the mother, who reports that her child fell down the stairs and sustained injuries to the right arm and leg. During the physical assessment the nurse identifies a number of old bruises on the child's back, buttocks, and upper arms. What should the nurse say to the child to obtain additional information? A: "Why did you fall down the stairs?" B: "Did you really fall down those stairs?" C: "Show me how you fell down the stairs." D: "Your mommy must have told you to say you fell down the stairs."

Learning disorders

A child has a learning disorder only if the specific problem interferes with academic achievement or activities of daily living.

anxiety disorders

A class of disorders marked by feelings of excessive apprehension and anxiety.

A client on the psychiatric unit asks a nurse about psychiatric advance directives (PADs). What information should form the basis of the nurse's response?

A client is allowed to consent to or refuse potential psychiatric treatments if a future incapacitating mental health crisis occurs.

C

A client is en route to the emergency department after sustaining a gunshot wound to the chest. Which priority nursing action should the nurse take to prepare for the arrival of the client? A: Reserve an operating room. B: Organize equipment for a tracheotomy. C: Prepare equipment for chest tube insertion(胸管放置). D: Arrange for a portable chest x-ray examination

Upon entrance into a mental health care system, clients are thoroughly assessed, and this is followed by the development of a mental health treatment plan. Which of the following are purposes of the treatment plan? (Select all that apply.)

A means of monitoring the clients progress An instrument for communication and coordination of care A guide for planning and implementation of care Evaluating the effectiveness of interventions

A, C, D

A nurse caring for a pregnant client at 28 weeks' gestation and her partner suspects intimate partner violence. Which assessments support this suspicion? Select all that apply. A: The woman has injuries to the breasts and abdomen. B: The partner refuses to come into the examination room. C: The partner answers questions that are asked of the woman. D: The woman has visited the clinic several times in the last month. E: The partner is excessively attentive while the health history is being taken

D

A nurse educates the mother of a four-year-old child about sexual abuse. What behavioral finding explained by the nurse signifies that the child may be a victim of child abuse? A: The child may attempt suicide. B: The child may be verbally aggressive. C: The child may have stress-related concerns. D: The child may show fear of certain people or places.

C

A nurse has learned that infants born to very young mothers are at risk for neglect or abuse, primarily because of what characteristic typical of adolescent mothers? A: Did not plan for her pregnancy B: Cannot anticipate her baby's needs C: Is involved in seeking her own identity D: Becomes resentful of the need to give constant care to the baby

C, E

A nurse provides crisis intervention for a client who recently left her husband because of physical abuse. Which client behaviors indicate to the nurse that the therapy has been successful? Select all that apply. A: Is able to cry B: Sleeps half the day C: Utilizes healthier coping skills D: Refuses a referral to support services E: Describes the current situation realistically

D, E

A nurse suspects sexual child abuse in a preschooler who has come for a routine check-up. What physical findings lead to the nurse's suspicion? Select all that apply. A: The nurse finds signs of immersion burns. B: The nurse finds hematomas and bruises at various stages of resolution. C: The nurse finds injuries and trauma inconsistent with reported cause. D: The nurse finds that the preschooler has difficulty walking and sitting. E: The nurse finds that the child has pain, itching, or unusual odor in genital area.

A, B, E

A parent of four is remanded to the psychiatric unit by the court for observation. The client was arrested and charged with abusing a 2-year-old child, who is in the pediatric intensive care unit in critical condition. The nurse approaches the client for the first time. How should the nurse anticipate that the client will likely respond? Select all that apply. A: By denying beating the child B: By avoiding talking about the situation C: By asking where the other three children are D: By expressing excessive concern for the child E: By exhibiting an emotional response that is inconsistent with the degree of injury

B

A primary healthcare provider writes a prescription of "Restraints PRN" for a client who has a history of violent behavior. What is the nurse's responsibility in regard to this prescription? A: Asking that the prescription indicate the type of restraint B: Recognizing that PRN prescriptions for restraints are unacceptable C: Implementing the restraint prescription when the client begins to act out D: Ensuring that the entire staff is aware of the prescription for the restraints

B, C

A woman who is emotionally and physically abused by her husband calls a crisis hotline for help. The nurse works with the client to develop a plan for safety. What should be included in the safety plan? Select all that apply. A: Limiting contact with the abuser B: Determining a safe place to go in an emergency C: Memorizing the domestic violence hotline number D: Obtaining a bank loan to finance leaving the abuser E: Arranging for a family member to assist her in leaving

A

A young woman tells the nurse, "My partner prevents me from taking my medications." What should the nurse do to deal with the situation? A: Conduct an interview with the client alone, when the partner is not around. B: Notify the primary healthcare provider to conduct an interview with the client. C: Collaborate with multiple community resources to obtain adequate health care. D: Evaluate the client's and the family's cultural beliefs, values, and practices to determine their specific needs.

A male client with a dissociative disorder copes with his low self-esteem by displaying behaviors associated with an exaggerated sense of self-importance. Which behaviors would this client most likely exhibit? Select all that apply. A) Bragging about special abilities B) Setting unrealistic goals C) Having unrealistic dreams D) Having a view of life that everything is either right or wrong

A. Bragging about special abilities, B. Setting unrealistic goals, C. Having unrealistic dreams

The set of emotional reactions that accompany a loss is called: A) Grief B) Anxiety C) Mourning D) Bereavement

A. Grief

Paranoid personality disorders are diagnosed more often in: A) Men B) Women C) Children D) Adolescents

A. Men

During assessment of a client with a dissociative disorder, the nurse notices that the client has been cutting herself on both arms. After talking with the client, the nurse, along with other members of the treatment team, decides that the best intervention at this time to prevent further self-destructive behavior would be: A) Establishing a signed contract with the client to tell a team member when she is having self-destructive thoughts B) Isolating the client from all other clients and activities until she is no longer having self-destructive thoughts C) Administering medications that will reduce the client's anxiety levels D) Involving the client in activities as a diversion from self-destructive thoughts

A. establishing a signed contract with the client to tel a team member when she is having self-destructive thoughts

3. A newly admitted client asks, "Why do we need a unit schedule? I'm not going to these groups. I'm here to get some rest." Which is the most appropriate nursing response? 1. "The purpose of group therapy is to learn and practice new coping skills." 2. "Group therapy is mandatory. All clients must attend." 3. "Group therapy is optional. You can go if you find the topic helpful and interesting." 4. "Group therapy is an economical way of providing therapy to many clients concurrently."

ANS: 1 Rationale: The nurse should explain to the client that the purpose of group therapy is to learn and practice new coping skills. The client owns his or her environment and can make decisions to attend group or not.

5. To promote self-reliance, how should a psychiatric nurse best conduct medication administration? 1. Encourage clients to request their medications at the appropriate times. 2. Refuse to administer medications unless clients request them at the appropriate times. 3. Allow the clients to determine appropriate medication times. 4. Take medications to the clients' bedside at the appropriate times.

ANS: 1 Rationale: The psychiatric nurse promoting self-reliance would encourage clients to request their medications at the appropriate times. Nurses are responsible for the management of medication administration on inpatient psychiatric units, but nurses must work with clients to encourage self-reliance and responsibility, which may result in independent decision-making, leading to medication adherence.

4. A client diagnosed with schizophrenia functions well and is bright, spontaneous, and interactive during hospitalization but then decompensates after discharge. What does the milieu provide that may be missing in the home environment? 1. Peer pressure 2. Structured programming 3. Visitor restrictions 4. Mandated activities

ANS: 2 Rationale: The milieu, or therapeutic community, provides the client with structured programming that may be missing in the home environment. The therapeutic community provides a structured schedule of activities in which interpersonal interaction and communication with others are emphasized. Time is also devoted to personal problems and focus groups.

7. What is the best rationale for including family in the client's therapy within the inpatient milieu? 1. To structure a program of social and work-related activities 2. To facilitate discharge from hospitalization 3. To provide a concrete demonstration of caring 4. To encourage the family to model positive behaviors

ANS: 2 Rationale: The nurse should include the client's family in therapy within the inpatient milieu to facilitate discharge from the hospital. Family members are invited to participate in some therapy groups and to share meals with the client in the communal dining room. Family involvement may also serve to prevent the client from becoming too dependent on the therapeutic environment.

11. A nurse attends an interdisciplinary team meeting regarding a newly admitted client. Which of the following individuals are typically included as members of the interdisciplinary treatment team in psychiatry? (Select all that apply.) 1. Respiratory therapist and psychiatrist 2. Occupational therapist and psychologist 3. Recreational therapist and art therapist. 4. Social worker and hospital volunteer 5. Mental health technician and chaplain

ANS: 2, 3, 5 Rationale: The interdisciplinary treatment team in psychiatry consists of a psychologist, occupational therapist, recreational therapist, art therapist, mental health technician, and chaplain. In addition, a psychiatrist, psychiatric nurse, psychiatric social worker, music therapist, psychodramatist, and dietitian also participate in the interdisciplinary treatment team. Respiratory therapists and hospital volunteers are not included in the interdisciplinary treatment team in psychiatry.

12. Which of the following conditions promote a therapeutic community? (Select all that apply.) 1. The unit schedule includes unlimited free time for personal reflection. 2. Unit responsibilities are assigned according to client capabilities. 3. A flexible schedule is determined by client needs. 4. The individual is the sole focus of therapy. 5. A democratic form of government exists.

ANS: 2, 5 Rationale: A therapeutic community is promoted when unit responsibilities are assigned according to client capability and a democratic form of government exists. Therapeutic communities are structured and provide therapeutic interventions that focus on communication and relationship-development skills.

8. How does a democratic form of self-government in the milieu contribute to client therapy? 1. By setting punishments for clients who violate the community rules 2. By dealing with inappropriate behaviors as they occur 3. By setting expectations wherein all clients are treated on an equal basis 4. By interacting with professional staff members to learn about therapeutic interventions

ANS: 3 Rationale: A democratic form of self-government in the milieu contributes to client therapy by setting the expectation that all clients should be treated on an equal basis. Clients participate in the decision-making and problem-solving aspects that affect treatment setting. The norms, rules, and behavioral limits are established by the staff and clients. All individuals have input.

9. A client is to undergo psychological testing. Which member of the interdisciplinary team should a nurse consult for this purpose? 1. The psychiatrist 2. The psychiatric social worker 3. The clinical psychologist 4. The clinical nurse specialist

ANS: 3 Rationale: The nurse should consult with the clinical psychologist to obtain psychological testing for the client. Clinical psychologists can administer, interpret, and evaluate psychological tests to assist in the diagnostic process.

1. An angry client on an inpatient unit approaches a nurse stating, "Someone took my lunch! People need to respect others, and you need to do something about this now!" The nurse's response should be guided by which basic assumption of milieu therapy? 1. Conflict should be avoided at all costs on inpatient psychiatric units. 2. Conflict should be resolved by the nursing staff. 3. On inpatient units, every interaction is an opportunity for therapeutic intervention. 4. Conflict resolution should only be addressed during group therapy.

ANS: 3 Rationale: The nurse's response should be guided by the basic assumption that every interaction is an opportunity for therapeutic intervention. The nurse can use milieu therapy to effect behavioral change and improve psychological health and functioning.

2. A client on an inpatient unit angrily says to a nurse, "Peter is not cleaning up after himself in the community bathroom. You need to address this problem." Which is the appropriate nursing response? 1. "I'll talk to Peter and present your concerns." 2. "Why are you overreacting to this issue?" 3. "You should bring this to the attention of your treatment team." 4. "I can see that you are angry. Let's discuss ways to approach Peter with your concerns."

ANS: 4 Rationale: The most appropriate nursing response involves restating the client's feeling and developing a plan with the client to solve the problem. According to Skinner, every interaction is an opportunity for therapeutic intervention to improve communication and relationship-development skills.

6. A nurse working on an inpatient psychiatric unit is assigned to conduct a 45-minute education group. What should the nurse identify as an appropriate group topic? 1. Dream analysis 2. Creative cooking 3. Paint by number 4. Stress management

ANS: 4 Rationale: The nurse should identify that teaching clients about stress management is an appropriate education group topic. Nurses should be able to perform the role of client teacher in the psychiatric area. Nurses need to be able to assess a client's learning readiness. Other topics for education groups include medical diagnoses, side effects of medications, and the importance of medication adherence.

The nurse manager is observing the performance of a nursing assistant. What behavior by the nursing assistant toward a client reflects a boundary violation?

Accepting a gift from the client A boundary violation occurs when a provider of care goes beyond the established therapeutic relationship standard and enters into a personal or social relationship with a client. Offering advice to the client is an example not of a boundary violation but rather of inadequate communication skills; advice should not be given because it undermines the client's ability to solve problems and may precipitate dependency and helplessness. False reassurance is an example not of a boundary violation but rather of inadequate communication skills; false reassurance should not be given because it is not based in reality and may close off communication. Engaging in excessive probing is an example not of a boundary violation but rather of inadequate communication skills; inappropriate probing is not therapeutic and may be done by the provider of care to avoid uncomfortable silences or obtain information because of curiosity.

Children and violence

Acts of aggression are common in schools where children may be bullied or intimidated.

Oral chlordiazepoxide (Librium) 100 mg/ hr is prescribed for a client with a Clinical Institute Withdrawal Assessment (CIWA) score of 25. The client has had 300 mg in 3 hours but is still displaying acute alcohol withdrawal symptoms. What is the next nursing action?

Administering chlordiazepoxide as indicated by the client's CIWA score

A client who is addicted to opioids undergoes emergency surgery. During the postoperative period the health care provider decreases the previously prescribed methadone dosage. For what clinical manifestations should the nurse monitor the client?

Agitation and attempts to escape from the hospital When the methadone dosage is reduced, a craving for opioids may occur, anxiety will increase, and the client will become agitated and may try to leave the hospital to secure drugs.

A male client who is taking clozapine (Clozaril) is seen by the nurse in the outpatient mental health clinic. The nurse interviews the client, sends a venous blood specimen to the laboratory, obtains the vital signs, and finally reviews all the collected information. Which complication associated with clozapine does the nurse suspect that the client is experiencing?

Agranulocytosis Clozapine (Clozaril) can cause bone marrow suppression. The expected white blood cell (WBC) value for an adult is 4500 to 10,000 mm3. The client has a reduction in WBCs, making him vulnerable to infection. A fever with complaints of a sore throat and weakness supports the conclusion that the client may have an infection. The red blood cell (RBC) count does not indicate anemia. The expected range of RBCs for an adult male is 4.6 to 6.2 ´ 106/mL3. The small change in the blood pressure from standing to sitting does not support the conclusion of orthostatic hypotension. Labile hypertension is associated with neuroleptic malignant syndrome. There are insufficient data to support the conclusion that the client is experiencing neuroleptic malignant syndrome. Although tachycardia and tachypnea are associated with neuroleptic malignant syndrome, the client's fever would be more than 100.6° F (38.1° C). Additional characteristics of neuroleptic malignant syndrome include labile hypertension, diaphoresis, drooling, increased muscle tone, and decreased level of consciousness.

While completing the history portion of an admission assessment of a client with schizophrenia, the nurse notices that the client is continually moving in the chair and frequently stands and then sits back down. The nurse knows that this client most likely is experiencing the side effect of:

Akathisia

A client is found to have paranoid schizophrenia, and the practitioner prescribes a typical antipsychotic medication. After a 1-month hospitalization the client is discharged home with instructions to continue the antipsychotic and a referral for weekly mental health counseling. The picture illustrates the client's physical status as observed by the nurse on the client's first visit to the community mental health clinic. What extrapyramidal side effect has developed?

Akathisia Akathisia, an extrapyramidal side effect of typical antipsychotics, is motor restlessness. The client is unable to sit or stand still and feels the need to move, pace, rock, swing the legs, or tap the feet. The condition occurs within 5 to 90 days of the initiation of therapy. Dystonia is muscle spasms of the face, tongue, head, neck, jaw, or back, usually resulting in exaggerated posturing. This extrapyramidal side effect of typical antipsychotics occurs within 1 hour to 1 week of the initiation of therapy. Tardive dyskinesia is facial, ocular, oral/buccal, lingual/masticatory, and systemic movements. This extrapyramidal side effect of typical antipsychotics may occur 6 months or more after the initiation of therapy. Pseudoparkinsonism has characteristics similar to those of Parkinson's disease (e.g., shuffling gait, tremors, rigidity, bradykinesia). This extrapyramidal side effect of typical antipsychotics may occur anytime after the initiation of therapy.

On the psychiatric unit a client has been receiving high doses of haloperidol (Haldol) for 2 weeks. The client says, "I just can't sit still, and I feel jittery(초조한)." Which side effect does the nurse suspect that the client is experiencing?

Akathisia(정좌 불능) Akathisia, a side effect of haloperidol (Haldol), develops early in therapy and is characterized by restlessness and agitation. Torticollis is characterized by a stiff neck (wry neck). Tardive dyskinesia is characterized by gross involuntary movements of the extremities, tongue, and facial muscles that develop after prolonged therapy. Pseudoparkinsonism is characterized by motor retardation, rigidity, and tremors; the reaction resembles Parkinson's syndrome but usually responds to decreasing the dose, the administration of an antidyskinetic medication, or discontinuation of the haloperidol.

What is the primary reason that the nurse encourages the family of an alcoholic to become involved in the treatment program?

Alcoholism involves the entire family.

A nurse is planning care for a depressed client. Which approach is most therapeutic?

Allowing the client time to complete activities

During group therapy, certain group change mechanisms may be observed. When an individual engages in helping others, resulting in improvement in his or her own self-esteem, which group change mechanism has the individual experienced?

Altruism

A nurse is monitoring a client with the diagnosis of schizophrenia who is experiencing opposing emotions simultaneously. When providing a change-of-shift report, how should the nurse refer to this emotional experience of the client? Double bind Ambivalence Loose association Inappropriate affect

Ambivalence Ambivalence is the existence of two conflicting emotions, impulses, or desires. Double bind is two conflicting messages, not emotions, in a single communication. Loose associations are not two conflicting emotions but instead the loosening of connections between thoughts. Inappropriate affect is not two conflicting emotions but instead the inappropriate expression of emotions.

A nurse is caring for a client who has been hospitalized for alcohol withdrawal. The client decides to attend an Alcoholics Anonymous meeting. What is a basic principle of this group?

Amends must be made to each person who has been harmed. The eighth step of the 12 steps of Alcoholics Anonymous (AA) is "Made a list of all persons we had harmed, and became willing to make amends to them all."

Amitriptyline (Elavil) is an antidepressant medication used to treat anxiety disorders. Which class of antidepressant medications does it belong to?

Amitriptyline (Elavil) is one of several tricyclic antidepressants used to treat anxiety disorders . It is not an MAOI (e.g., isocarboxazid [Marplan], phenelzine [Nardil], tranylcypromine [Parnate), SSRI (e.g., citalopram [Celexa], Fluoxetine [Prozac], Paroxetine [Paxil]), or SNRI (e.g., venlafaxine [Effexor], Duloxetine [Cymbalta], norepinephrine).

What are the "four A's" for which nurses should evaluate clients with suspected Alzheimer disease?

Amnesia, apraxia(운동 불능), agnosia(인지불능증), aphasia(연하 불능)

How should a nurse characterize a sudden terrorist act that causes the deaths of thousands of adults and children and negatively affects their families, friends, communities, and the nation?

An adventitious crisis is a crisis or disaster that is unplanned and accidental; its subcategories include national disasters and crimes of violence. Recurring crisis is not considered a category in crisis theory. A situational crisis results from an external source and involves the loss of self-concept or self-esteem of an individual or family group. A maturational crisis occurs as an individual moves into a new stage of development and prior coping styles are no longer effective; these crises are usually predictable.

A client has just been admitted with the diagnosis of borderline personality disorder. There is a history of suicidal behavior and self-mutilation. The nurse remembers that the main reason that clients use self-mutilation 자해 is to:

Express anger or frustration

__________ side effects can occur when antipsychotic medications are taken that manifest as abnormal movements such as akathisia and pseudo-Parkinson symptoms.

Extrapyramidal

Phonological disorder

Failure to use speech sounds expected for his or her age group

Valium is administered to a client anxious about impending surgery. Which of the following side effects is the client at risk for?

Falls

What should the nurse include when planning activities for an older nursing home resident with a diagnosis of dementia?

Familiar activities that the resident can complete successfully

A nurse is assessing a client with chronic schizophrenia. Which effects will the client most likely exhibit? Select all that apply. Apathy Sadness Flatness Hostility Happiness Depression

Apathy Flatness Apathy (indifference) is common among people with chronic schizophrenia because negative symptoms are more apparent. Flatness, with few extremes of emotion, is common among people with chronic schizophrenia because negative symptoms are more apparent. Extremes in emotions are not associated with chronic schizophrenia. Sadness is related more to mood disorders, such as a depressive episode of bipolar disorder or major depression. Hostility may be seen in some forms of schizophrenia, such as paranoid schizophrenia, but it is rarely seen in the chronic stages. Happiness and elation are associated with manic episodes of bipolar disorder, not chronic schizophrenia. Depression is related to mood disorders, such as a depressive episode of bipolar disorder or major depression.

A client is admitted to the psychiatric unit of the hospital with a diagnosis of conversion disorder. The client is unable to move either leg. Which finding should the nurse consider consistent with this diagnosis?

Appearing composed 침착한, 차분한

With the client's permission, the nurse should inform the family about what is happening. The main reason for this action is that informed families:

Are better equipped to assist the client Families who are informed about the client's status can help with treatment goals and discharge planning. Relief of anxiety, relaxation, and fewer nursing problems may all be secondary gains, but none is the primary purpose.

A nurse recalls that in a conversion disorder, pseudoneurological symptoms such as paralysis or blindness:

Are generally necessary for the client to cope with a stressful situation

An 8-year-old child is found to have oppositional defiant disorder. What behavior noted by the nurse supports this diagnosis?

Argues with adults Oppositional defiant disorder is a repeated pattern of negativistic, disobedient, hostile, defiant behavior toward authority figures, usually exhibited before 8 years of age.

A client with a diagnosis of paranoid schizophrenia throws a chair across the room and starts screaming at the other clients. Several of these clients have frightened expressions, one starts to cry, and another begins to pace. A nurse removes the agitated client from the room. What should the nurse remaining in the room do next?

Arrange a unit meeting to discuss what has just happened

An 18 year-old woman is brought to the emergency department by her two roommates after being found unconscious in the bathroom. Laboratory tests are ordered. The nurse reviewing the findings notes that the urinalysis is positive for flunitrazepam (Rohypnol). The nurse knows that flunitrazepam is often used:

As a date rape drug

A newly admitted male client with schizophrenia appears to be responding to internal stimuli when laughing and talking to himself. What is the best initial response by the nurse? Asking the client whether he is hearing voices Encouraging the client to engage in unit activities Telling the client that the voices he is hearing are not real Giving the client his prescribed PRN antipsychotic medication

Asking the client whether he is hearing voices Because the client is newly admitted, the nurse needs to conduct a thorough assessment before intervening. Encouraging the client to engage in unit activities may eventually be done but is not the priority. Telling the client that the voices he is hearing are not real assumes that the client is hallucinating. The client's behavior does not indicate the need for extra medication at this time. Some clients with schizophrenia have hallucinations throughout their lives.

A client is presented with the treatment option of electroconvulsive therapy (ECT). After discussion with staff members, the client requests that a family member be called to help make the decision about this treatment. What ethical principle does the nurse consider when supporting the client's request?

Autonomy Autonomy is the ethical principle of respecting the independence and right to self-determination of others. In this situation the nurse focuses on helping the client make a choice. Justice is the ethical principle that requires all people to be treated fairly, regardless of sex, age, religion, diagnosis, marital status, or socioeconomic level. Veracity is the ethical principle that requires truthfulness. Beneficence is the duty to do good and promote the welfare of others.

During an interview with a 15-year-old female client admitted for depression, the nurse expresses her disappointment when she to learns that the client recently became pregnant and then had an abortion. The nurse is contradicting the effective interview guideline of:

Avoiding ones personal values that may cloud professional judgment

A 3-year-old child is found to have autism. Which behaviors should the nurse expect when observing this child? (Select all that apply.)

Avoids eye-to-eye contact Performs repetitive activities

The average age for onset of anorexia nervosa is: A) 13 years old B) 17 years old C) 33 years old D) 40 years old

B. 17 years old

The nurse who works in a sleep clinic knows that approximately __________% of adults suffer from insomnia. A) 10 to 20 B) 30 to 40 C) 50 to 60 D) 70 to 80

B. 30 to 40

The main feature of a factitious disorder is that symptoms are purposefully produced to allow the individual to: A) Get out of work B) Assume the sick role C) Assume control of treatment D) Get the attention of health care providers

B. Assume the sick role

Three years after the loss of her husband of 35 years, the wife has a full-time job but finds that she cannot sleep well at night, has frequent mood changes, and attends the couple's night out with friends that she and her husband attended. Upon seeking counseling, she discovers that she is exhibiting symptoms of: A) Bereavement-related depression B) Complicated grief C) Anticipatory grief D) Caregiver grief

B. Complicated Grief

The main focus of medical management for anorexia is to: A) Encourage rapid weight gain B) Encourage the client to eat voluntarily C) Teach more appropriate food choices D) Keep the client from developing additional problems

B. Encourage the client to eat voluntarily

The way individuals cope with an unexpected hospitalization depends on many factors. However, the one that is most significant is:

Basic personality

A nurse is caring for a client with generalized anxiety disorder. Which factor should be evaluated to determine the client's present status?

Behavior

A client is granted permission to watch a favorite television show in the evening because she participated in an occupational therapy activity. Her therapist bases this on ______ theory.

Behavioral

A nurse is caring for clients with a variety of psychiatric illnesses. For which diagnoses is the establishment of a psychiatric advance directive (PAD) most beneficial? (Select all that apply.)

Bipolar disease Paranoid schizophrenia

A nurse is speaking with a client who was sexually abused as a child. The client does not know what constitutes inappropriate touch by another person. What issue will have to be addressed with this client?

Boundary violations

The nurse is working with a client who talks freely about feeling depressed. During the interaction the client states, "Things will never change." What findings support the nurse's conclusion that the client is experiencing hopelessness? (Select all that apply.)

Bouts of crying Self-destructive acts Feelings of worthlessness

Common Behavioral Problems in children

Breath-holding spells occur from 6 months and can last until 5 years of age.

How can the nurse best assist a client with an obsessive-compulsive disorder to decrease the use of ritualistic behavior?

By attempting to limit situations that will worsen the anxiety

A client describes his delusions in minute detail to the nurse. How should the nurse respond?

By changing the topic to reality-based events

How should a nurse at an assisted living facility encourage a client to effectively complete the tasks of older adulthood?

By fostering a sense of contentment when the client looks back on her achievements

A hospitalized client with a borderline personality disorder consistently breaks the unit's rules. How will confronting the client about this behavior help the client?

By fostering self-awareness

A client's methods of coping are maladaptive. How can the nurse best help the client develop healthier coping mechanisms?

By setting realistic limits on the client's maladaptive behavior Setting realistic limits on the client's maladaptive behavior provides structure and helps the client learn acceptable behavior. The client may not be ready for promotion of interpersonal relationships at this time. No environment is stress free. The client may not be ready to assume responsibilities at this time.

A female client is taking a benzodiazepine for her anxiety disorder. She complains of anorexia and nausea since she started taking the medication a few days ago. What is the nurse's best response? A) "Be sure to take the medicine on an empty stomach to avoid these symptoms." B) "It takes a while to get used to the medicine. Give it a couple of weeks." C) "Try taking the medication with food or milk, and see if the symptoms improve." D) "Stop taking the medication immediately, and I will notify your doctor."

C. Try taking the medication with food or milk, and see if the symptoms improve

A nurse is caring for a client with the diagnosis of somatoform disorder, conversion type. What type of affect does the nurse expect this client to exhibit? (Select all that apply.)

Calm Matter-of-fact

Causes:

Can result from repeated unsuccessful attempts to feed an irritable infant

When planning care for an older client, the nurse remembers that aging has little effect on a client's:

Capacity to handle life's stresses An individual's ability to handle stress develops through experience with life; aging does not reduce this ability but often strengthens it. The senses of taste and smell are often diminished in the older individual. Muscle or motor strength is diminished in the older individual. Short-term memory is diminished in the older individual, whereas long-term memory remains strong.

A male college student who is smaller than average and unable to participate in sports becomes the life of the party and a stylish dresser. What defense mechanism should the nurse determine that the client is using? Introjection Sublimation Compensation Reaction formation

Compensation

A hospitalized 7-year-old boy wakes up crying because he has wet his bed. It is most appropriate for the nurse to:

Change the child's bed while he changes his pajamas. Changing the child's bed while the child changes his pajamas will not call attention to the accident and will minimize the child's embarrassment. The child would probably be unable to change the bed without assistance; failure to complete the task might add to his embarrassment. Taking the child to the bathroom to change his pajamas and reminding the child to call a nurse next time will only add to the child's embarrassment.

What should a nurse recognize that a client who uses the defense mechanism of sublimation is doing?

Channeling unacceptable impulses into socially approved behavior The individual using sublimation attempts to fulfill desires by selecting a socially acceptable activity rather than one that is socially unacceptable.

A married woman is brought to the emergency department of a local hospital. Her eyes are swollen shut, and she has a bruise on her neck. She reports that she is being beaten by her husband. How does the nurse expect the husband to behave when he arrives at the emergency department?

Charming

Children are considered mentally retarded only if they have problems in general intellect and adaptive functioning.

Children who repeatedly score lower than 70 on IQ tests are defined as mentally retarded.

Learning disorders

Children with learning disabilities often have low self-esteem and lack the social skills of other children.

Somatoform disorder

Children with somatoform disorders need understanding and reassurance.

Breaks with reality such as those experienced by clients with schizophrenia require the nurse to understand that: Extended institutional care is necessary. Clients believe that what they feel that they are experiencing is real. Electroconvulsive therapy produces remission in most clients with schizophrenia. The clients' families must cooperate in the maintenance of the psychotherapeutic plan.

Clients believe that what they feel that they are experiencing is real. Failure to accept the client and the client's fears is a barrier to effective communication. Today mental health therapy is directed toward returning the client to the community as rapidly as possible. Electroconvulsive therapy is not the treatment of choice for clients with schizophrenia. Family cooperation is helpful but not an absolute necessity.

A client tells the nurse in the mental health clinic that the practitioner said that the cornerstone of therapy used in the clinic is cognitive therapy. The client asks what this therapy entails. What concept should the nurse explain as the basis of cognitive therapy?

Cognitive therapy seeks to discover underlying thoughts that lead to feelings of depression and anxiety; also, it teaches the client to replace these thoughts with more positive, realistic thinking. The response "Unconscious feelings influence actions" reflects a psychoanalytical approach to treatment. The response "People can act their way into a new way of thinking" reflects a behavioral approach to treatment. The response "Maladaptive behaviors will continue as long as they are reinforced" reflects a behavioral approach to treatment. *Negative thoughts can precipitate anxiety

A man is admitted to the psychiatric unit after attempting suicide. The client's history reveals that his first child died of sudden infant death syndrome 2 years ago, that he has been unable to work since the death of the child, and that he has attempted suicide before. When talking with the nurse he says, "I hear my son telling me to come over to the other side." What should the nurse conclude that the client is experiencing?

Command hallucination

An older client whose family has been visiting him in the psychiatric unit is visibly angry and says to the nurse, "My daughter-in-law says they can't take me home until the doctor lets me go. She doesn't understand how important this is to me; she's not from our culture." What should the nurse do? 1 Ignore the statement for the present. 2 Say, "You feel she doesn't want you at home." 3 Reflect on the client's feelings about the cultural differences. 4 Respond, "The doctor is the one who makes decisions about discharge."

Correct2 Say, "You feel she doesn't want you at home"

Medications are used with extreme caution in clients with personality disorders because of their questionable effectiveness. If a client is receiving an antipsychotic medication, it is especially important for the nurse to monitor the client for side effects of: A) Increased thirst and urination, nausea, and anorexia B) Dry mouth, altered taste, sexual dysfunction, and dizziness C) Bone marrow depression, gastrointestinal symptoms, and confusion D) Extrapyramidal movements, dry mouth, blurred vision, and photophobia

D. Extrapyramidal movements, dry mouth, blurred vision, and photophobia

A woman goes to a late movie with her friends and gets home after midnight. She is scheduled to play in a tennis tournament early the next morning and loses all her matches. She blames the line judges for her loss, rather than her poor play. Which coping mechanism best describes her response to losing the match? A) Denial B) Regression C) Displacement D) Rationalization

D. Rationalization

The father of three young children dies. The wife expresses how worried she is about how to raise the children on her own without the support of her husband. She finds herself crying and living through each day without accomplishing anything. In which grieving stage is this behavior typically experienced? A) Denial B) Depression and identification C) Acceptance and recovery D) Yearning

D. Yearning

A "spell" is a culturally defined mental health disorder or a dissociative "state" seen in African Americans, Europeans, and Americans from southern U.S. cultures. This "state" is characterized by: A) Sudden collapsing with eyes open and inability to see, while still hearing and understanding without being able to move B) Seizure activity and coma for up to 12 hours preceded by extreme excitement or irrational behavior C) A state in which spirit possession interferes with daily activities D) Communication with deceased relatives or spirits that occurs during a trancelike state

D. communication with deceased relatives or spirits that occurs during a trancelike state

On the first day of the month a practitioner prescribes an antipsychotic medication for a client with schizophrenia. The initial dosage is 25 mg once a day, to be titrated in increments of 25 mg every other day to a desired dosage of 175 mg daily. On what day of the month will the client reach the desired daily dose of 175 mg?

Day 13 The client will reach the desired dosage of 175 mg on the 13th day of the month; on the first day it is 25 mg, on the third day it is 50 mg, on the fifth day it is 75 mg, on the seventh day it is 100 mg, on the ninth day it is 125 mg, on the 11th day it is 150 mg, and on the 13th day it is 175 mg.

A nurse concludes that a client's withdrawn behavior may temporarily provide a:

Defense against anxiety

In the process of development the individual strives to maintain, protect, and enhance the integrity of the self. The nurse determines that this is usually accomplished through the use of:

Defense mechanisms

In the process of development the individual strives to maintain, protect, and enhance the integrity of the self. The nurse determines that this is usually accomplished through the use of:

Defense mechanisms When the individual experiences a threat to self-esteem, anxiety increases, and defense mechanisms are used to protect the self. Affective reactions are mood disorders. Withdrawal patterns are deviant ways of coping with stress; if carried to an extreme, behavior may become pathological. Ritualistic behaviors are not an aspect of the developmental process.

A nurse is caring for a client who is hospitalized because of injuries sustained in a major automobile collision. As the client is describing the accident to a friend, the client becomes very restless, and pulse and respirations increase sharply. Which factor is most likely related to the client's physical responses?

Delayed psychological response to trauma

maoi diet

Delicatessen meats that are fermented, such as bologna, pepperoni, salami, and sausage, are also high in tyramine and should be avoided by people taking MAOIs. Fresh fish does not need to be avoided by people taking MAOIs; dried, pickled, cured, fermented, and smoked fish should be avoided. Citrus fruits do not need to be avoided by people taking MAOIs. Figs and bananas in large amounts should be avoided.

A client on the psychiatric unit tells the nurse, "I'm a movie star, and the other clients are my audience." What is an appropriate conclusion for the nurse to document about what the client is experiencing?

Delusion of grandeur A delusion of grandeur is a fixed false belief that the person is a powerful, important person.

A client in the mental health clinic tells the nurse, "The FBI is out to kill me." What should the nurse document that the client is experiencing?

Delusion of persecution

Clients diagnosed with Type I-Positive Schizophreniasymptoms respond better to antipsychotic medications. Manifestations of Type I Schizophrenia include which of the following? (Select all that apply.)

Delusions Hallucinations Illusions

An older adult who lives alone tells a nurse at the community health center, "I really don't need anyone to talk to. The TV is my best friend." The nurse identifies the defense mechanism known as:

Denial

The parents of a toddler with recently diagnosed moderate cognitive impairment discuss their child's possibility of future independent function. What should the nurse conclude?

Denial is being used as a defense.

A female client who is severely incapacitated by obsessive-compulsive behavior has been admitted to the mental health hospital. The client's compulsive ritual involves changing her clothing eight to 12 times a day. She continually asks the nurse for advice regarding her problems but then ignores it. What conflict is this an example of?

Dependence versus independence

Neuroleptic malignant syndrome is a potentially fatal reaction to antipsychotic therapy. What signs and symptoms of this syndrome should the nurse identify? (Select all that apply.)

Diaphoresis Hyperrigidity Hyperthermia as a result of dopamine blockade in the hypothalamus.

A client who is taking clozapine (Clozaril) calls the nurse in the psychiatric clinic to report the sudden development of a sore throat and a high fever. What should the nurse instruct the client to do?

Discontinue the medication and, if the health care provider is unavailable today, go to the emergency department for evaluation Symptoms of infection are suggestive of agranulocytosis, an adverse effect that can occur with clozapine therapy and can cause death.

Autism

Disorder of communication, social interactions, and behavior

One afternoon the nurse on the unit overhears a young female client having an argument with her boyfriend. A while later the client complains to the nurse that dinner is always late and the meals are terrible. The nurse identifies the defense mechanism that the client is using as:

Displacement reduces anxiety by transferring the emotions associated with an object or person to another emotionally safer object or person. Projection is the attempt to deal with unacceptable feelings by attributing them to another. Dissociation is an attempt to detach emotional involvement or the self from an interaction or the environment. Intellectualization is the use of facts or other logical reasoning rather than feelings to deal with the emotional effect of a problem; it is a form of denial.

A client who is in a manic phase of bipolar disorder threatens staff and clients on a psychiatric acute care unit. Place the following interventions in priority order, from the least to the most restrictive.

Diversional activities Limit-setting Medication administration Seclusion Restraints

A client's history demonstrates a pervasive pattern of unstable and intense relationships, impulsiveness, inappropriate anger, manipulation, offensive behavior, and hostility. The admitting diagnosis is borderline personality disorder. What does the nurse anticipate that this client may attempt to do?

Divide the staff into opposing factions to gain self-esteem

A parent of a 17-year-old girl who has been hospitalized for extremely disturbed acting-out behavior leaves a gift for the daughter but says, "I'm too busy to visit today." The daughter becomes upset and tearful after being given the message and opening the package. What does the nurse conclude that the parent's actions represent?

Double-bind message

During the sociocultural assessment of a client who is entering a mental health program, the nurse focuses on which information related to the client? (Select all that apply.)

Education Income Ethnicity Age Gender Belief system

What would be some psychiatric assessment examples of "appraisal of healtha/illness" (pg.94)?

Events leading to the problem, definition of the problem, client's goal, regular health care received

A 15 y/o girl is being admitted to an inpatient mental health clinic for anorexia nervosa. The nurse must know that the most common personality characteristic of teens affected with the disorder is

Excessive cooperation

The night nurse reports that a young client with paranoid schizophrenia has been awake for several nights. The day nurse reviews the client's record and finds that this client did not have an interrupted sleep pattern disorder before transfer from a private room to a four-bed room 3 days ago. What factor should the nurse identify as most likely related to the client's sleeplessness? Fear of the other clients Concern about family at home Watching for an opportunity to escape Trying to work out emotional problems

Fear of the other clients Because the client has paranoid feelings that other people are out to do harm, assignment to a four-bed room may be threatening. Concern about family at home seems unlikely because the disruption appears to have started with the transfer to a four-bed room. Watching for an opportunity to escape is possible but unlikely; planning an escape is usually not part of a schizophrenic pattern of behavior. Trying to work out emotional problems is possible but not likely; clients with schizophrenia have difficulty solving problems.

Children who are smaller at birth, have small heads (microcephaly), and fail to develop normally have characteristics of: A) Parents who smoke B) Fetal alcohol syndrome C) Fetal alcohol intoxication D) Fetal accumulation syndrome

Fetal alcohol syndrome

A client with a diagnosis of schizophrenia is discharged from the hospital. At home the client forgets to take the medication, is unable to function, and must be rehospitalized. What medication may be prescribed that can be administered on an outpatient basis every 2 to 3 weeks?

Fluphenazine can be given intramuscularly every 2 to 3 weeks to clients who are unreliable about taking oral medications; it allows them to live in the community while keeping the disorder under control. Lithium is a mood-stabilizing medication that is given to clients with bipolar disorder. This drug is not given for schizophrenia. Diazepam (Valium) is an antianxiety/anticonvulsant/skeletal muscle relaxant that is not given for schizophrenia. Fluvoxamine (Luvox) is a selective serotonin reuptake inhibitor; it is administered for depression, not schizophrenia.

An injured child is brought to the emergency department by the parents. While interviewing the parents, the nurse begins to suspect child abuse. Which parental behaviors might support this conclusion? (Select all that apply.)

Focusing on the child's role in sustaining the injury Changing the story of how the child sustained the injury Giving an explanation of how the injury occurred that is not consistent with the injury

What is the most important information for a nurse to teach to prevent relapse in a client with a psychiatric illness?

Following the prescribed medication regimen is important because side effects and denial of illness may cause clients to stop taking their medications; this is a common cause of relapse or recurrence of symptoms. Although a close support system is beneficial, it may not always be possible to achieve. It is impossible to create a stress-free environment; clients need to learn better ways to cope with stress. Refraining from any activity that may cause anxiety is too restrictive.

The nurse is caring for a client with Alzheimer's disease who exhibits behaviors associated with hyperorality. To meet the client's need for a safe milieu, the nurse will instruct staff to monitor the client:

For attempts at eating inedible objects

A nurse encourages a client to attend Alcoholics Anonymous (AA) meetings after discharge. What do self-help groups such as AA help their members do?

Foster changes in behavior

During a therapy session, a client is asked to respond to a word with the first word or phrase that comes to mind. What term is commonly used to refer to this technique?

Free association

The parent of a 9 year old boy has been told by the child's teacher that he exhibits symptoms of adhd. Which specific behaviors may the child exhibit for this diagnosis to be made? Select all that apply:

Frequently interrupts or intrudes on others, is easily distracted by outside stimuli, has feelings of restlessness or frequently fidgets with hands or feet, often leaves tasks incomplete

Which functions are registered nurses legally permitted to perform in a mental health hospital? Select all that apply.

Health promotion Case management Treating human responses Health promotion, case management, and treating human responses are all within the legal scope of nursing practice. Registered nurses may use counseling interventions but may not perform psychotherapy; the members of the nursing team permitted to perform psychotherapy are psychiatric/mental health clinical nurse specialists and psychiatric/mental health nurse practitioners. Only those who are legally licensed to prescribe medications, such as psychiatric nurse practitioners, may do so.

A depressed client is admitted to the mental health unit. What factor should the nurse consider most important when evaluating the client's current risk for suicide?

Impending anniversary of the loss of a loved one

A client is found to have a borderline personality disorder. What behavior does the nurse consider is most typical of these clients?

Impulsive

The mother of an 18-year-old man comes to the local mental health center. She is extremely upset because of her son's behavior since the young man returned from his freshman year at college. He takes his brother's clothing, comes in at all hours, and refuses to get a job. Sometimes he is happy and outgoing, but at other times he is withdrawn. The mother asks why her son is like this. While contemplating this situation, the nurse considers that adolescents are usually:

Impulsive and self-centered

A 4-year-old child is found to have attention deficit-hyperactivity disorder (ADHD). What information about the child's behavior should the nurse expect when obtaining a health history from the parents? (Select all that apply.)

Impulsiveness Excessive talking Playing video games for hours on end Failure to follow through or finish tasks

A nurse reminds a client that it is time for group therapy. The client responds by shouting, "You're always telling me what to do, just like my father!" What defense mechanism is the client using

In transference a client assigns to someone the feelings and attitudes originally associated with an important significant other. In regression a client reverts to past levels of coping to reduce anxiety. In reaction formation a client displays the exact opposite behavior, attitude, or feeling to that which is demonstrated in a given situation. Cognitive distortions are thought patterns that exaggerate reality or are irrational, such as black-and-white thinking or overgeneralization.

A client with a history of aggressive, violent behavior is admitted to the psychiatric unit involuntarily. The nurse, who understands the need to use deescalation approaches during the preassaultive stage of the violence cycle, monitors the client's behavior closely for progression of signs of impending violence. List these client assessments in order of escalating aggression, from the lowest risk to the highest. 1. Pacing in the hall 2. Increasing tension in facial expression 3. Engaging in verbal abuse toward the nurse 4. Pushing another client while waiting in line to the dining room 5. Having difficulty waiting to take turns during a group project

Increasing tension in facial expression indicates increasing anxiety, but the client is still maintaining self-control. Impulsivity, as demonstrated by the inability to take turns with others, indicates that the client is having some difficulty setting limits on his or her own behavior. When anxiety escalates to the point of hyperactivity and pacing behaviors, the client is attempting to cope with the anxiety and to discharge physical and psychic energy. Engaging in verbal abuse may precipitate physical abuse and is a sign that the client is not able to maintain self-control. The laying on of the hands in an offensive manner is a physical act of aggression.

A client with a history of aggressive, violent behavior is admitted to the psychiatric unit involuntarily. The nurse, who understands the need to use deescalation approaches during the preassaultive stage of the violence cycle, monitors the client's behavior closely for progression of signs of impending violence. List these client behaviors in order of escalating aggression, from the lowest risk to the highest.

Increasing tension in facial expression indicates increasing anxiety, but the client is still maintaining self-control. Impulsivity, as demonstrated by the inability to take turns with others, indicates that the client is having some difficulty setting limits on his or her own behavior. When anxiety escalates to the point of hyperactivity and pacing behaviors, the client is attempting to cope with the anxiety and to discharge physical and psychic energy. Engaging in verbal abuse may precipitate physical abuse and is a sign that the client is not able to maintain self-control. The laying on of the hands in an offensive manner is a physical act of aggression.

A client with catatonic schizophrenia who is in a vegetative state is admitted to the psychiatric hospital. The nurse identifies short- and long-term outcomes in the client's clinical pathway. What is the priority short-term outcome of care that the client should be able to attain? Talking with peers Performing her own activities of daily living Completing unit activities and assignments Ingesting adequate fluid and food with assistance

Ingesting adequate fluid and food with assistance A client in a vegetative state may not eat or drink without assistance; fluids and foods are basic physiologic needs that are necessary to prevent malnutrition and starvation; therefore the intake of adequate fluid and food is a priority short-term goal. The client is in total withdrawal; talking with peers, performing activities of daily living, and completing activities and assignments are not priority outcomes at this time.

The nurse explains to the mother of a preschool child that Erikson identified the developmental conflict of children from 3 to 5 years as:

Initiative versus guilt Initiative versus guilt is the developmental conflict that faces the preschool child; the child will feel guilty if initiative is stifled by others.

A client with diabetes mellitus is able to discuss in detail the diabetic metabolic process while eating a piece of chocolate cake. What defense mechanism does the nurse identify when evaluating this behavior?

Intellectualization is the avoidance of a painful emotion with the use of a rational explanation that removes the event from any personal significance. Projection is the attribution of unacceptable thoughts and feelings to others. Dissociation is a temporary alteration of consciousness or identity used to handle conflict; amnesia is an example. Displacement is the discharge of a pent-up feeling, generally hostility, on an object or person perceived to be weaker than the person who aroused the feelings.

Which goal specific to a client with impaired verbal communication related to a psychological barrier should be documented in the client's clinical record? 1 Freedom from injury 2 Engaging independently in solitary craft activities 3 Identifying the consequences of acting-out behavior 4 Interacting appropriately with others in the therapeutic milieu

Interacting appropriately with others in the therapeutic milieu Interacting appropriately with others in the therapeutic milieu is a goal related to the identified problem and is appropriate and measurable. Freedom from injury is not related to the identified problem; this is true for everyone. Engaging independently in solitary craft activities will not encourage verbal communication. Identifying the consequences of acting-out behavior is inappropriate and not related to the identified problem.

What conflict associated with Erikson's psychosocial stages of development should the nurse remember when caring for a client 30 years of age?

Intimacy versus isolation The major task of the young adult is to develop close, sharing relationships that may include a sexual partnership; the person develops a sense of belonging and avoids isolation. During the integrity-versus-despair stage the adjusted older adult can look back with satisfaction and acceptance of life and resolve the inevitability of death; failure at these tasks results in despair. The middle school-aged child gains a sense of competence and self-assurance as social interactions and academic pursuits are mastered; failure in these tasks leads to feelings of inferiority. During adolescence the individual develops a sense of self, self-esteem, and emotional stability; failure to establish self-identity results in a lack of self-confidence and difficulty with occupational choices.

What conflict associated with Erikson's psychosocial stages of development should the nurse remember when caring for a client 30 years of age?

Intimacy versus isolation The major task of the young adult is to develop close, sharing relationships that may include a sexual partnership; the person develops a sense of belonging and avoids isolation. During the integrity-versus-despair stage the adjusted older adult can look back with satisfaction and acceptance of life and resolve the inevitability of death; failure at these tasks results in despair. The middle school-aged child gains a sense of competence and self-assurance as social interactions and academic pursuits are mastered; failure in these tasks leads to feelings of inferiority. During adolescence the individual develops a sense of self, self-esteem, and emotional stability; failure to establish self-identity results in a lack of self-confidence and difficulty with occupational choices.

A client with schizophrenia, paranoid type, is delusional, withdrawn, and negativistic. The nurse should plan to: Invite the client to play a game of cards or board game. Explain to the client the benefits of joining a group activity. Encourage the client to become involved in group activities. Mention to the client that the psychiatrist has ordered increased activity.

Invite the client to play a game of cards or board game. Activities that require limited interpersonal contact are less threatening. Individuals with schizophrenia, paranoid type, usually do not respond to an authoritarian approach because they do not trust others, particularly those who act in an aggressive manner. Group activities require interaction with other people, which is threatening to individuals with paranoid feelings.

When answering questions from the family of a client with Alzheimer disease the nurse explains that the disease:

Is a slow, relentless deterioration of the mind

Which information does the nurse include in the teaching plan for the client who is prescribed sumatriptan for migraine headache?

Is contraindicated in people with coronary artery disease

A client who had to be cut out of a car after a motor vehicle collision has no visible physical effects from the ordeal. The client responds to the emergency department nurse's questions factually in a soft voice with a composed manner. This behavior may indicate that the client:

Is controlling the expression of feelings

A client has become increasingly depressed, and the practitioner prescribes an antidepressant. After 20 days of therapy, the client returns to the clinic. The client appears relaxed and smiles at the nurse. The most significant conclusion that the nurse can draw from this behavior is that the client:

Is responding to the antidepressant therapy

Survivors of a major earthquake are being interviewed on admission to the hospital. The nurse notes that they exhibit a flattened affect, make minimal eye contact, and speak in a monotone. These behaviors are indicative of the defense mechanism known as:

Isolation Isolation is the separation of thought or memory from feeling.

Survivors of a major earthquake are being interviewed on admission to the hospital. The nurse notes that they exhibit a flattened affect, make minimal eye contact, and speak in a monotone. These behaviors are indicative of the defense mechanism known as:

Isolation is the separation of thought or memory from feeling. Splitting is the polarization of positive and negative feelings. Introjection is the integration of the beliefs and values of another into one's own ego. Compensation is making up for a real or imagined lack in one area by overemphasizing another.

When do the assessment process begins and end's (pg.94)?

It begins with the client's admission to the facility or service and ends only after his or her relationship with the health cared system if over.

A client with a history of depression tells the nurse about planning to retire from work next year. What common dynamic about retirement should the nurse consider when interacting with this client?

It is a developmental task of significance.

A client tells a nurse, "I have been having trouble sleeping and feel wide awake as soon as I get into bed." Which strategies should the nurse teach the client that will promote sleep? (Select all that apply.)

Leaving the bedroom when unable to sleep Exercising in the afternoon rather than in the evening Counting backward from 100 to 0 when his mind is racing

A client with schizophrenia says to the nurse, "I've been here 5 days. There are five players on a basketball team. I like to play the piano." How should the nurse document this cognitive disorder? Word salad Loose association Thought blocking Delusional thinking

Loose association These ideas are not well connected and there is no clear train of thought. This is an example of loose association. Word salad is incoherent expressions containing jumbled words. This client's thoughts are coherent but not connected. Thought blocking occurs when the client loses the train of thinking and ideas are not completed. Each of the client's thoughts is complete but not linked to the next thought. These statements are reality based and not reflective of delusional thinking

A nurse is caring for an adolescent client with the diagnosis of schizophrenia, undifferentiated type. Which signs and symptoms should the nurse expect the client to experience?

Loosened associations and hallucinations Loosened associations and hallucinations are the primary behaviors associated with a thought disorder such as schizophrenia. Paranoid delusions and hypervigilance are more common in paranoid-type schizophrenia than in the undifferentiated type

The nurse anticipates that the medication that will be used to prevent symptoms of withdrawal in clients with a long history of alcohol abuse is:

Lorazepam (Ativan) is most effective in preventing the signs and symptoms associated with withdrawal from alcohol. It depresses the central nervous system by potentiating γ-aminobutyric acid, an inhibitory neurotransmitter. Phenobarbital (Luminal) is used to prevent withdrawal symptoms associated with barbiturate use. Chlorpromazine (Thorazine), an antipsychotic medication, is not used for alcohol withdrawal. Methadone hydrochloride (Methadone) is used to prevent withdrawal symptoms associated with opioid use.

A client who is going to be discharged has been receiving risperidone (Risperdal) 3 mg three times a day. The nurse should teach the client that the medication:

May cause sedation if taken concurrently with alcohol Risperidone (Risperdal) potentiates the action of alcohol and can cause oversedation if the drug and alcohol are taken together. This medication should be taken consistently to prevent recurrence of symptoms and maintain a therapeutic blood drug level. Medications should be taken as prescribed; taking them all at one time may interrupt the maintenance of a constant therapeutic blood level.

A nurse is assigned to lead a relaxation group. Which techniques should the nurse incorporate? Select all that apply.

Meditation Mental imagery Deep-breathing exercises Meditation lowers heart and blood pressure rates, decreases levels of adrenal corticosteroids, improves mental alertness, and increases a sense of calmness and peace. Imagery is the internal experience of memories, dreams, fantasies, and visions that serves as a bridge connecting the body, mind, and spirit; its distractive ability decreases adrenal corticosteroids, promotes muscle relaxation, and increases a sense of calmness and peace. Deep breathing increases oxygenation and releases tension in the muscles of the neck, shoulders, and torso. Token economy is a behavioral theory that acknowledges acceptable behavior with a reward (token) that can be redeemed for something that has a perceived value (e.g., a desirable activity). Operant conditioning, a behavioral therapy, is the learning of a particular type of behavior followed by a reward.

An obviously upset client comes to the mental health clinic and, after pushing ahead of the other clients, states, "I had an argument with my daughter, and now I'm tense, and worried, and angry." What level of anxiety does the nurse determine that the client is experiencing?

Moderate The client is focused on one part of reality but is unable to grasp the total picture; this situation reflects a moderate level of anxiety.

A client is receiving doxepin (Sinequan). For which most dangerous side effect of tricyclic antidepressants should a nurse monitor the client?

Mydriasis Mydriatic action causes dilated pupils, which can precipitate an acute attack of glaucoma, resulting in blindness.

Abuse and neglect

Neglect is not meeting a child's basic needs for food, clothing, shelter, love, and belonging.

Parent-child conflicts

No child or parent escapes childhood without conflict.

Which tool is used to standardize and measure nursing treatments?

Nursing Interventions Classification (NIC)

At a staff meeting, the question of a staff nurse's returning to work after completing a drug rehabilitation program is discussed. What is the most therapeutic way for the staff to handle the nurse's return?

Offering the nurse support in a straightforward manner

nfant regurgitates and rechews food.

Often disappears as the child grows older

Olanzapine (Zyprexa) is prescribed for a client with bipolar disorder, manic episode. What cautionary advice should the nurse give the client?

Olanzapine (Zyprexa), a thienobenzodiazepine, can cause orthostatic hypotension. Blurred, not double, vision may occur. Decreased salivation is an effect of Olanzapine. It may also cause nausea and other gastrointestinal upsets and should be taken with fluid or food.

The nurse should first discuss terminating the nurse-client relationship with a client during the:

Orientation phase, when a contract is established

A nurse in the mental health clinic is counseling a client with the diagnosis of depression. During the counseling session the client says, "Things always seem the same. They never change." The nurse suspects that the client is feeling hopeless. For what indication of hopelessness should the nurse assess the client?

Outbursts of anger

A client with schizophrenia is demonstrating waxy flexibility. Which intervention is the best way to manage the possible outcome of this behavior? Providing thickened liquids to minimize the risk of aspiration Documenting intake and output each shift to monitor hydration Reinforcing appropriate social boundaries through staff role modeling Passive range-of-motion exercises three times a day for effective joint health

Passive range-of-motion exercises three times a day for effective joint health Waxy flexibility is an excessive and extended maintenance of posture that can lead to a variety of problems, including joint trauma. Passive range-of-motion exercises focus on the effective management of joint mechanics. Although aspiration precautions, documentation of intake and output, and staff role modeling may address issues experienced by a client with schizophrenia, passive range-of-motion exercises address waxy flexibility.

A psychiatric unit uses a behavioral approach to determine a client's level of privileges. Which factor should a nurse use to determine whether an increase in privileges is warranted?

Performing hygiene activities independently Independent performance of hygiene activities is evidence of the client's ability to act responsibly. Although an improvement in short-term memory is good, it is not related to the critical element of behavioral therapy. Verbalizations without actions do not show the improvement in behavior sought in behavioral therapy.

A male client with a history of schizophrenia was admitted to the mental health facility after he was found on the street in a confused state and was uncooperative when approached by the police. One of the first assessments that should be performed on this client upon admission is a _____ assessment.

Physical

An older depressed person at an independent living facility constantly complains about her health problems to anyone who will listen. One day the client says, "I'm not going to any more activities. All these old crabby people do is talk about their problems." What defense mechanism does the nurse conclude that the client is using?

Projection The client is assigning to others those feelings and emotions that are unacceptable to him- or herself.

The nurse manager of a psychiatric unit informs the primary nurse that a client will be admitted to the unit within an hour. The client's admission diagnosis is paranoid schizophrenia. What classic clinical findings should the nurse anticipate? Select all that apply. Mutism Posturing Flat affect Extreme negativism Prominent delusions Auditory hallucinations

Prominent delusions Auditory hallucinations Prominent delusions are the essential feature of paranoid schizophrenia; delusions are typically persecutory, grandiose, or both, but delusions with other themes, such as jealousy, religiosity, or somatization, also may occur. Auditory hallucinations are a characteristic associated with paranoid schizophrenia; usually they are related to the delusional theme. Mutism is associated more commonly with the subtype of catatonic schizophrenia. Psychomotor retardation and posturing are associated with catatonic schizophrenia. A flat affect is associated more commonly with the subtype of disorganized schizophrenia. Extreme negativism is associated more commonly with the subtype of catatonic schizophrenia.

When a diagnosis of child abuse is established, the priority of nursing care is:

Protecting the total well-being of the child Management of the abused child places protection of the child's total being above consideration of parents' rights or wishes. Protecting the child, not promotion of parental attachment, is the priority at this time. Supervision may be necessary, but it is only part of maintaining the child's well-being. Teaching methods of discipline is not appropriate at this time.

What should the nurse do when a client with the diagnosis of schizophrenia talks about being controlled by others? Express disbelief about the client's delusion. Divert the client's attention to unit activities. React to the feeling tone of the client's delusion. Respond to the verbal content of the client's delusion

React to the feeling tone of the client's delusion. Reacting to the feeling tone of the client's delusion helps the client explore underlying feelings and allows the client to see the message that his verbalizations are communicating. Expressing disbelief about the client's delusion denies the client's feelings rather than accepting and working with them. Attempting to divert the client rather than accepting and working with him denies the client's feelings. Responding to the verbal content of the client's delusion focuses on the delusion itself rather than on the feeling that is causing the delusion.

A 30-year-old female client asks the nurse to change her room, stating that she hates her roommate and can't stand to be in the same room with her. Just as she finishes speaking, her roommate enters and the client tells her she missed her and has been all over the unit looking for her. The nurse recognizes that the client is using:

Reaction formation

What should a nurse ensure when creating an environment that is conducive to psychological safety?

Realistic limits are set. Realistic limits and controls provide a degree of security that adds to emotional safety by limiting choices, reducing the need for self-regulation, and decreasing the need for decision-making. Passive acceptance is not conducive to psychological safety and often signifies a degree of resignation. It is impossible to meet all of a client's physical needs in any situation. An orderly physical environment bears little relationship to psychological safety.

A client with a conversion disorder is experiencing paralysis of a leg. The nurse can expect this client to:

Recover use of the affected leg but, under stress, to again experience these symptoms

Operational defiant disorder

Recurring pattern of disobedient, hostile behavior toward authority figures

Unsatisfied needs create anxiety that motivates an individual to action. What should the nurse identify as the purpose for this action?

Reducing tension

A 13-year-old boy who recently was suspended from school for consistently bullying other children is brought to the pediatric mental health clinic by his mother. The child is assessed by the psychiatrist and referred to a psychologist for psychological testing. The day after the tests are completed, the mother returns to the clinic and asks the nurse for results of the tests. The nurse should:

Refer the mother to the psychiatrist

A client is to begin lithium carbonate therapy. The nurse should ensure that before the drug's administration the client has baseline:

Renal studies

Encopresis

Repeated, usually voluntary, passage of feces in inappropriate places in a child over 4 years of age with no physical abnormalities

A nurse is caring for a group of children with the diagnosis of autism. Which signs and symptoms are associated with this disorder? (Select all that apply.)

Repetitive activities Self-injurious behaviors Lack of communication with others

One of the major goals of therapy for adolescents with chemical dependency is:

Replacing the use of the chemical with effective coping skills

Incidences of child molestation often are revealed years later when the victim is an adult. Which defense mechanism reflects this situation?

Repression Repression is a coping mechanism in which unacceptable feelings are kept out of conscious awareness; later, under stress or anxiety, thoughts or feelings surface and come into one's conscious awareness. Isolation is the separation of a thought from a feeling tone. Regression is the use of an unconscious coping mechanism through which a person avoids anxiety by returning to an earlier, more satisfying, or comfortable time in life. Introjection is the integration of the beliefs and values of another into one's own ego structure.

After an automobile collision involving a fatality and a subsequent arrest for speeding, a client has amnesia regarding the events surrounding the accident. Which defense mechanism is being used by the client?

Repression is coping with overwhelming emotions by blocking awareness or memory of the stressful event. Projection is attributing one's own unacceptable feelings and thoughts to others. Suppression is consciously keeping unacceptable feelings and thoughts out of awareness. Rationalization is the attempt to mask unacceptable feelings or behaviors by providing excuses and explanations.

A nurse is caring for an older adult with the diagnosis of dementia. Which manifestations are expected in this client? (Select all that apply.)

Resistance to change Inability to recognize familiar objects Inability to concentrate on new activities or interests Tendency to dwell on the past and ignore the present

A nurse begins a therapeutic relationship with a client with the diagnosis of schizotypal personality disorder. What is the best initial nursing action?

Respecting the client's need for social isolation

A 3-year-old child is found to have a pervasive developmental disorder not otherwise specified (autistic disorder). What should the nurse consider most unusual for the child to demonstrate?

Responsiveness민감성 to the parents

Following an argument with his daughter, a father takes away her phone privileges. He later feels guilty about the argument and asks her if she would like to go shopping for new clothes. What is this defense mechanism known as?

Restitution

Doxepin (Sinequan) is prescribed for a 74-year-old man for treatment of a depressive episode that has not responded to several other medications. The nurse in the outpatient clinic reviews with the client the side effects of doxepin. The identification of which side effects by the client as needing to be reported to the health care provider allows the nurse to conclude that the teaching has been effective? (Select all that apply.)

Retention of urine Thoughts of suicide

The development of motor, language, and social problems and loss of previous skills that occurs between 5 months and 4 years of age. Head growth declines, hand movements resemble hand wringing, and loss of social interest and severe speech impairments occur.

Rett syndrome

What would be some psychiatric assessment examples of " coping responses, physical status" (pg.95)?

Review of function in each body system, physical assessment, diet history, sleep patterns, exposure to toxic substances, activities of daily living

A child is found to have attention deficit-hyperactivity disorder (ADHD). What strategy should the nurse teach the parents to help them cope with this disorder?

Reward appropriate conduct

Antipsychotic drugs can cause extrapyramidal side effects. Which responses should the nurse document as indicating pseudoparkinsonism? (Select all that apply.)

Rigidity Tremors Bradykinesia

What is a constructive but lengthy method of confronting the stress of adolescence and preventing a negative and unhealthy developmental outcome?

Role experimentation

A nurse enters the room of an agitated, angry client to administer the prescribed antipsychotic medication. The client shouts, "Get out of here!" The nurse's best approach is to:

Say, "I'll be back in 15 minutes, and then we can talk." Saying, "I'll be back in 15 minutes, and then we can talk" allows the agitated, angry client time to regain self-control; telling the client that the nurse will return will decrease possible guilt feelings and implies to the client that the nurse cares enough to come back. Getting assistance and giving the medication by way of injection does not respect the client's feelings; it may decrease trust and increase feelings of anger, helplessness, and hopelessness. An agitated, angry client will not be able to accept a logical explanation. Continued insistence may provoke increased anger and further loss of control.

An older client whose family has been visiting on the psychiatric unit is visibly angry and says to the nurse, "My daughter-in-law says they can't take me home until the doctor lets me go. She doesn't understand how important this is to me; she's not from our culture." What should the nurse do?

Say, "You feel she doesn't want you at home."

A client with the diagnosis of schizophrenia watches the nurse pour juice for the morning medication from an almost-empty pitcher and screams, "That juice is no good! It's poisoned." What is the most therapeutic response by the nurse? Assure the client, "The juice is not poisoned." Pour the client a glass of juice from a full pitcher. Take a drink of the juice to show the client that it is safe. Say, "You sound frightened. Is there something else I can give you to take your medication with?

Say, "You sound frightened. Is there something else I can give you to take your medication with? The response "You sound frightened" reflects the client's feelings and avoids focusing on the delusion; following up with "Is there something else I can give you to take your medication with?" encourages the client to take the medication. The response "The juice is not poisoned" will not change the client's feelings because the belief is real to the client. Pouring the client a glass of juice from a full pitcher will not change the client's feelings because the other pitcher also may be perceived as poisoned. Taking a drink of the juice to show the client that it is safe will not change the client's feelings; the client will believe that the nurse was not really drinking the juice.

A 12-year-old child who has a history of school failure and destructive acting out is admitted to a child psychiatric unit with the diagnosis of conduct disorder. The youngest of three children, the child is identified by both the parents and the siblings as the family problem. The nurse recognizes the family's pattern of relating to the child as:

Scapegoating 심리) 전가(轉嫁) ((고통을 준 본인을 벌할 수 없을 경우 다른 사람을 공격하는 현상)) When all members of a family blame one member for all their problems, scapegoating is occurring.

An adult with the diagnosis of schizophrenia is admitted to the psychiatric hospital. The client is ungroomed, appears to be hearing voices, is withdrawn, and has not spoken to anyone for several days. What should the nurse do during the first few hospital days? See that the client bathes and changes clothes daily. Wait and see whether the client approaches the staff. Conduct an admission assessment interview with the client. Seek out the client frequently to spend short periods of time together.

Seek out the client frequently to spend short periods of time together. Seeking out the client frequently to spend short periods of time together will help the nurse establish trust without unduly increasing anxiety. Seeing that the client bathes and changes clothes daily is not the priority unless the client is extremely dirty; this client is ungroomed, not dirty. A withdrawn client will usually not approach anyone. The client's history reveals a failure to speak.

At times a client's anxiety level is so high that it blocks attempts at communication and the nurse is unsure of what is being said. To clarify understanding, the nurse says, "Let's see whether we mean the same thing." What communication technique is being used by the nurse?

Seeking consensual validation

A 30 y/o man is brought to the mental health inpatient unit with the diagnosis of schizophrenia. His clothes are dirty, his hair is uncombed, he has not shaved for several days, and his teeth are chipped, which several cavities evident. He is having hallucinations and delusions. What is the priority nursing diagnosis at this time?

Self-care deficit: bathing, hygiene

An individual whose employment has been terminated because his company has been acquired by another company is brought by a family member to the mental health clinic because of extreme depression. While talking with the nurse the client says, "I'm a useless, worthless person. No wonder I lost my job." What type of delusion does the nurse identify?

Self-deprecation(자기비난 비하) The client's statement is self-derogatory and reflects a low self-appraisal.

An 8-year-old child with a terminal illness is demanding of the staff. The child asks for many privileges that other children on the unit do not have. The staff members know that the child does not have long to live. The nurse can best help the staff members cope with the child's demands by encouraging them to:

Set reasonable limits to help the child feel more secure and content. Reasonable limits are necessary because they provide security and help keep the child's behavior within acceptable bounds. Relationships, not special privileges, should provide the necessary security. Providing treats is an unrealistic approach that allows the child to manipulate the situation.

Group therapy, which assists in relieving emotional distress and encourages psychological and behavioral changes, was developed following World War II as a result of a:

Shortage of psychiatrists

A client with a diagnosis of borderline personality disorder (BPD) has negative feelings toward the other clients on the unit and considers them all "bad." Which defense is the client using when identifying the other clients thusly?

Splitting

On the afternoon of admission to a psychiatric unit, an adolescent boy with the diagnosis of schizophrenia exposes his genitals to a female nurse. What should the nurse's immediate therapeutic response be? Ignoring the client at this time Stating that this behavior is unacceptable Moving him to his room for a short time-out Telling the client to come to the office later to discuss the behavior

Stating that this behavior is unacceptable When clients enter a new milieu, limits should be set on unacceptable behavior and acceptable behavior should be reinforced. Neither clients nor unacceptable behavior should ever be ignored. Moving the client to his room for a short time-out is punishment. Unacceptable attention-getting behavior must be addressed immediately; also, the focus should be on appropriate behavior

A client comes to a mental health center with severe anxiety, evidenced by crying, hand-wringing, and pacing. What should the first nursing intervention be?

Staying physically close to the client

A withdrawn client refuses to get out of bed and becomes upset when asked to do so. What nursing action is most therapeutic?

Staying with the client until the client calms down

Which client characteristic is an initial concern for the nurse when caring for a client with the diagnosis of paranoid schizophrenia? Continual pacing Suspicious feelings Inability to socialize with others Disturbed relationship with the family

Suspicious feelings The nurse must consider the client's suspicious feelings and establish basic trust to promote a therapeutic milieu. Continual pacing is not a problem because the nurse can walk back and forth with the client. Inability to socialize with others and disturbed relationship with the family may be of long-range importance but have little influence on the nurse-client relationship at this time.

A client with schizophrenia is taking benztropine (Cogentin) in conjunction with an antipsychotic. The client tells a nurse, "Sometimes I forget to take the Cogentin." What should the nurse teach the client to do if this happens again?

Take a dose as soon as possible, up to 2 hours before the next dose.

A client has been taking prescribed risperidone (Risperdal) 3 mg twice a day for the past 8 days. A friend brings the client to the outpatient clinic. The client reports tremors, shortness of breath, a fever, and sweating. What should the nurse do?

Take the client's vital signs and arrange for immediate transfer to a hospital These clinical manifestations signal the presence of neuroleptic malignant syndrome; the cardinal sign of this condition is a high body temperature. Therefore the nurse first should document the hyperthermia and then arrange for immediate hospitalization.

A client has been taking prescribed risperidone (Risperdal) 3 mg twice a day for the past 8 days. A friend brings the client to the outpatient clinic. The client reports tremors, shortness of breath, a fever, and sweating. What should the nurse do?

Take the client's vital signs and arrange for immediate transfer to a hospital. These clinical manifestations signal the presence of neuroleptic malignant syndrome; the cardinal sign of this condition is a high body temperature. Therefore the nurse first should document the hyperthermia and then arrange for immediate hospitalization. Unless the client is experiencing impaired ventilation, it is important to complete a focused assessment before transfer. The care needed can be provided in an emergency department or medical unit, not a psychiatric unit. Neuroleptic malignant syndrome may occur without an overdose; this syndrome can occur when a high-potency antipsychotic drug is prescribed, with typical onset within 3 to 9 days after initiation of the medication. Benztropine (Cogentin) will have little or no effect on neuroleptic malignant syndrome.

What is an initial client objective in relation to anger management?

Taking responsibility for the hostile 적대적인 behavior

A hyperactive 9-year-old child with a history of attention deficit-hyperactivity disorder is admitted for observation after a motor vehicle collision. On what should nursing actions be focused when the nurse is teaching about personal safety?

Talking with the child about the importance of using a seat belt

A client with chronic undifferentiated schizophrenia is receiving an antipsychotic medication. For which potentially irreversible extrapyramidal side effect should a nurse monitor the client?

Tardive dyskinesia Tardive dyskinesia occurs as a late and persistent extrapyramidal complication of long-term antipsychotic therapy. It is most often manifested by abnormal movements of the lips, tongue, and mouth.

A client with chronic undifferentiated schizophrenia is receiving an antipsychotic medication. For which potentially irreversible extrapyramidal side effect should a nurse monitor the client? Torticollis Oculogyric crisis Tardive dyskinesia Pseudoparkinsonism

Tardive dyskinesia Tardive dyskinesia occurs as a late and persistent extrapyramidal complication of long-term antipsychotic therapy. It is most often manifested by abnormal movements of the lips, tongue, and mouth. The other side effects are reversible with administration of an anticholinergic (e.g., benztropine [Cogentin]) or an antihistamine (e.g., diphenhydramine [Benadryl]) or cessation of the medication.

A client with the diagnosis of schizophrenia is given one of the antipsychotic drugs. The nurse understands that antipsychotic drugs can cause extrapyramidal side effects. Which effect is cause for the greatest concern? Akathisia Tardive dyskinesia Parkinsonian syndrome Acute dystonic reaction

Tardive dyskinesia Tardive dyskinesia, an extrapyramidal response characterized by vermicular movements and protrusion of the tongue, chewing and puckering movements of the mouth, and puffing of the cheeks, is often irreversible, even when the antipsychotic medication is withdrawn. Akathisia, motor restlessness, usually can be treated with antiparkinsonian or anticholinergic drugs while the antipsychotic medication is continued. Parkinsonian syndrome (a disorder featuring signs and symptoms of Parkinson disease such as resting tremors, muscle weakness, reduced movement, and festinating gait) can usually be treated with antiparkinsonian or anticholinergic drugs while the antipsychotic medication is continued. Dystonia, impairment of muscle tonus, can usually be treated with antiparkinsonian or anticholinergic drugs while the antipsychotic medication is continued.

A client with the diagnosis of schizophrenia is given one of the antipsychotic drugs. The nurse understands that antipsychotic drugs can cause extrapyramidal side effects. Which effect is cause for the greatest concern? 1 Akathisia 2 Tardive dyskinesia 3 Parkinsonian syndrome 4 Acute dystonic reaction

Tardive dyskinesia, an extrapyramidal response characterized by vermicular movements and protrusion of the tongue, chewing and puckering movements of the mouth, and puffing of the cheeks, is often irreversible, even when the antipsychotic medication is withdrawn. Akathisia, motor restlessness, usually can be treated with antiparkinsonian or anticholinergic drugs while the antipsychotic medication is continued. Parkinsonian syndrome (a disorder featuring signs and symptoms of Parkinson disease such as resting tremors, muscle weakness, reduced movement, and festinating gait) can usually be treated with antiparkinsonian or anticholinergic drugs while the antipsychotic medication is continued. Dystonia, impairment of muscle tonus, can usually be treated with antiparkinsonian or anticholinergic drugs while the antipsychotic medication is continued.

A nurse on the psychiatric unit of the hospital has been assigned four clients for the shift. The assignment includes an 84-year-old client who is severely depressed, a 73-year-old client who is being discharged, a 53-year-old client who was admitted for lithium toxicity, and a 48-year-old client who has panic attacks. Which client should the nurse evaluate first after receiving report?

The 53-year-old client should be evaluated first because of the severity of adaptations associated with lithium toxicity.

What is the purpose of the outcome during the planning phase for a mentally ill patient (pg.93)?

identifying behaviors that would indicate the problems is solved. ( used to monitor the client's progress)

A recently hired nurse is caring for several clients on a mental health unit at a local community hospital. The nurse manager is evaluating the nurse's performance. What situation indicates that the nurse-client boundaries of the recently hired nurse are appropriate?

The nurse shares with the entire treatment team vital information the client disclosed in a private session.

Several studies have demonstrated that significant ____ changes occur in people who displayed hostile or negative behaviors. A) Attitudinal B) Behavioral C) Gastrointestinal D) Immune-mediated

immune-mediated

is a pattern of excessive emotional expression accompanied by attention seeking behavior. May be flashy or dramatic in style of dress, mannerisms, and speech in order to draw attention to themselves. They are emotionally shallow and often live in a romantic fantasy world.

histrionic

According to Erikson, a child's increased vulnerability to anxiety in response to separation or pending separation from significant others results from failure to complete a developmental stage. What does the nurse call this stage?

Trust Without the development of trust, the child has little confidence that the significant other will return; separation is considered abandonment by the child. Without identity, the individual will have a problem forming a social role and a sense of self; this results in identity diffusion and confusion. Without initiative, the individual will experience the development of guilt and feelings of inadequacy. Without autonomy, the individual has little self-confidence, develops a deep sense of shame and doubt, and learns to expect defeat.

A secretary in a home health agency gossips about coworkers and then writes them notes to tell them how valuable they are to the organization and how much she likes working with them. What defense mechanism is being used by the secretary?

Undoing Undoing is atonement for or an attempt to dissipate unacceptable acts or wishes.

A client's hands are raw and bloody from a ritual involving frequent handwashing. Which defense mechanism does the nurse identify?

Undoing is an act that partially negates a previous one; the client is using this primitive defense mechanism to reduce anxiety. Clients who wash their hands compulsively may be having thoughts that they consider "dirty." Projection is the attribution of one's thoughts or impulses to another. Introjection is treating something outside the self as if it is actually inside the self. Suppression is a process that is often listed as a defense mechanism, but it is actually a conscious, intentional exclusion of material from one's awareness.

When silence is used in therapeutic communication, clients should feel:

Unhurried to answer

A nurse is in the process of developing a therapeutic relationship with a client who has an addiction problem. What client communication permits the nurse to conclude that they are making progress in the working stage of the relationship? (Select all that apply.)

Verbalizes difficulty identifying personal strengths Acknowledges the effects of the addiction on the family Addresses how the addiction has contributed to family distress

Certain questions are applicable in determining nursing negligence. (Select all that apply.)

Was reasonable care provided?" "Was there a breach of nursing duty?" "Was there an act of omission that resulted in harm? "Except for the nurse's action, would the injury have occurred?"

What is the primary concern for a nurse caring for a client who is grossly impaired by stimulants?

What is the primary concern for a nurse caring for a client who is grossly impaired by stimulants?

Mental Health Problems of Children

When a child demonstrates absence of growth, an inability or refusal to change, or failure to achieve the developmental tasks of his or her age-group, mental health assistance should be sought.

Humanistic theories are important to health care because these theories serve as the foundation for the concept of:

holistic care

A, D, E, F

When presenting a workshop on adolescent suicide, a community health nurse identifies which risk factors? Select all that apply. A: Victim of family violence B: Limited or strained family finances C: Member of a single-parent household D: Dependence on alcohol, drugs, or both E: Uncertainty related to sexual orientation F: Repeated demonstration of poor impulse control

Somatoform disorder

When stress is relieved, the child returns to a healthy level of functioning.

B

Which intervention will the nurse implement when assisting a child with a history of aggressive behavior to regain control in the triggering(v. 引起) phase of an assault cycle? A: Discuss alternative behaviors to substitute for aggression. B: Provide the child with a quiet, low-stimulus environment. C: Speak to the child in a calm but firm manner. D: Administer medication as needed (PRN) to facilitate de-escalation

A client with schizophrenia is actively psychotic, and a new medication regimen is prescribed. A student nurse asks the primary nurse, "Which of the medications will be the most helpful against the psychotic signs and symptoms?" What response should the nurse give? Citalopram (Celexa) Ziprasidone (Geodon) Benztropine (Cogentin) Acetaminophen with hydrocodone (Lortab)

Ziprasidone (Geodon) Ziprasidone (Geodon) is a neuroleptic, which will reduce psychosis by affecting the action of both dopamine and serotonin. Citalopram (Celexa) is a selective serotonin reuptake inhibitor antidepressant. Benztropine (Cogentin) is an anticholinergic. Acetaminophen with hydrocodone (Lortab) is an analgesic/opioid.

A client with schizophrenia is actively psychotic, and a new medication regimen is prescribed. A student nurse asks the nurse, "Which of the medications will be the most helpful against the psychotic signs and symptoms?" What response should the nurse give?

Ziprasidone (Geodon) is a neuroleptic, which will reduce psychosis by affecting the action of both dopamine and serotonin. Citalopram (Celexa) is a selective serotonin reuptake inhibitor antidepressant. Benztropine (Cogentin) is an anticholinergic. Acetaminophen with hydrocodone (Lortab) is an analgesic/opioid.

attention-deficit/hyperactivity disorder (ADHD)

a psychological disorder marked by extreme inattention and/or hyperactivity and impulsivity

A 10 year old male client with autism experiences loneliness and social anxiety as a result of his disease. Which CAM therapy will the nurse suggest to help this client most with his feelings?

animal-assisted therapy

An individual with an __________ personality disorder relies on deceit and manipulation to get his or her way.

antisocial

Loss of ability to perform every day actions, activities

apraxia

Theories that view depression as a group of learned responses are called _____ theories

behavioral

A client response to the termination phase of therapeutic relationship is withdrawal. Thre response most often is manifested by client behavior is

being absent from appointments

A mental health care provider who is aware of her cultural views and attitudes towards other cultures and who strives to understand, communicate, and effectively work with clients of other cultures is considered

culturally competent

The day after the birth of their baby, the parents are upset to learn that the baby has a heart defect. At this time it is most helpful for the nurse to do what? -Have the parents talk with other parents. -Explain the diagnosis in a variety of ways. -Encourage the expression of their feelings. -Assure the parents that surgery will correct the problem.

encourage the expression of their feelings

A client in the outpatient clinic is denying that he is addicted to alcohol. He tells the nurse that he is not an alcoholic and that it is his nagging wife who causes him to drink. What is the most therapeutic response by the nurse? -"I don't think that your wife is the problem." -"Everyone is responsible for his own actions." -"Perhaps you should have marriage counseling." -"Why do you think that your wife is the cause of your problems?"

everyone is responsible for his own actions

Before the diagnosis of mental retardation can be made, which factors must be present?

inability to communicate effectively, poor adaptation to social situations, iq less than 70, inability to care for self-appropriate to age, maladaptive coping skills

Growth

increase in physical size

The inability to fall asleep or stay asleep is called: A) Insomnia B) Narcolepsy C) Hypersomnia D) Nocturnal myoclonus

insomnia

A nurse administers antidepressant medication to a client in an assisted-living facility. This is an example of which phase of the nursing process?

intervention

enuresis

involuntary urination of a child 5 years or older

learning disorders

is diagnosed when a child with normal intelligence routinely falls below the result of other children in the same age and grade group on standard reading,math or written test

Pica

is persistent eating of nonfood items for longer than 1 month.

A mother brings her 16-month-old daughter to the well-child clinic for a checkup. She is upset and reports, "My child refuses to eat at mealtimes. It's a battle between us." What is the best response by the nurse? -"How often do you offer her food?" -"What's her daily eating schedule?" -"Does the doctor know that she refuses food?" -"It may be helpful to keep a daily diary of the food she eats."

it may be helpful to keep a daily diary of the food she eats

Disorder is characterized by a pattern of grandiosity and the need to be admired. They believe they are special, unique, or extra important. Unrealistic inflated beliefs about their accomplishments and they become extremely angry if criticized or outshone by others. Fantasize about unlimited money, power, or love. They take advantage of exploit others without guilt or remorse. Men are more likely to have narcissistic personality disorders.

narcissistic

haldol

photo sensitivity , stay in low sunlight

The extreme form of negative stereotyping. When all assumes that all members of a culture behave in the traditional manner.

prejudice

communication disorders

problems with expression receiving messages the pronunciation of words and stuttering

A nurse uses therapeutic communication techniques in order to achieve desired client outcomes. Which communication technique is a part of therapeutic communication? -asking for explanations -showing sympathy to the pt -asking personal questions to the pt -Providing relevant information to the pt

providing relevant information to the client

The nurse asks questions to an older client about past experiences and listens attentively. Which therapeutic communication strategy is involved when the older client is recalling the past? -touch -reminiscence -reality orientation -validation therapy

reminiscence

are times of partial or complete disappearance of symptoms the course for chronic mental health problems follows this up-and-down pattern.

remissions

Encopresis

repeated usually involuntary passage of feces in inappropriate places in a child over 4 years of age with no physical abnormalities

Following an argument with his daughter, a father takes away her phone privileges. He later feels guilty about the argument and asks her if she would like to go shopping for new clothes. Which defense mechanism is this?

restitution

Detachment from social relationships; emotional expression is restricted. They lack the willingness or desire to become involved in close relationships, they are society's "loners" who prefer solitary activities and their own company. Emotionally restricted and unable to take pleasure in activities, friendships, or social relationships. Individuals communicate emotional detachment. Coldness and lack of concern for others. Sexual experiences hold little interest. More common in men and families with an already diagnosed member.

schizoid

Acute opioid overdose may cause

severe respiratory depression, pinpoint pupils, and stupor or coma.

Schizophrenia is a complex syndrome of maladaptive thoughts and behaviors that affects human functioning in the emotional, spiritual, intellectual, and __________ areas of an individual's life.

social

The nurse is aware that in addition to assessment, one of the first goals of therapy for the client with a dissociative disorder is: A) Revisiting of past traumas B) Pharmacological therapy C) Stabilization D) Family therapy

stabilization

a simplified or standardized belief or concept regarding people who belong to another culture

stereotype

During the recoil stage of recovery from violence, the major characteristic is: _____________

struggle to adapt

Conscience and a sense of right and wrong are expressed in the ____________, which acts to counterbalance the id's desire for immediate gratification.

superego

In order to provide ideal therapeutic communication to clients, a health care facility provides interpreter services. Which statement regarding an interpreter is correct? -Interpreters can be relatives or friends of the client as well. -The interpreter should be able to make literal, word-for-word translations. -The interpreter should be able to interpret not only the language but also the culture. -The interpreter should be available as long as the health care provider is caring for the client.

the interpreter should be able to interpret not only the language but also the culture

During the preparation phase of a therapeutic relationship with a client, what is the main task to be completed by the nurse?

to gather and review all possible information regarding the client

A 16-year-old high school student is referred to a community health center by a local hotline because of the fear of having contracted herpes. The teenager is upset and shares this information with the community health center nurse. What should the nurse's initial response be? -let me get a brief health hx now -try not to worry until you know whether you have herpes -you sound worried. let me make arrangements to have you examined. herpes has received too much attention in the media; let's be realistic.

you sound worried. let me make arrangements to have you examined


Related study sets

Saunders reproductive system review

View Set

Ch 47 Gastric and Duodenal Disorders

View Set

MCQs in ELECTRICITY AND MAGNETISM

View Set

COM1000 everything everywhere, Public Speaking Chapter 8, COMM chps 1,2,4,6,10, Public Speaking Chapter 10

View Set

Intro to Biology Chapter 3 questions

View Set